You are on page 1of 797

A and B are in a line to purchase tickets. How many people are in the line?

(1) There are 15 people behind A and 15 people in front of B.


(2) There are 5 people between A and B.

(algebra, medium)

(T) Suppose there are n people in the line and A is the ath place and B is the
bth place in the line. (1) says that a= n – 15 and b = 16. (2) says that
a = b + 6 (if A is in front of B) or a = b – 6 (if A is behind B).
Thus n = a +15 has two possible values:
n could be either b + 21 = 37 or b + 9= 25. NOT SUFF

1
a , b , and c are integers and a < b < c. S is the set of all integers from a to b,
inclusive. Q is the set of all integers from b to c, inclusive. The median of Set S
is (3/4) b. The median of set Q is (7/8) c. If R is the set of all integers from a to
c , inclusive, what fraction of c is the median of set R?

3/8 1/2 11/16 5/7 3/4

(statistics,hard)

Note that for a set of consecutive integers, the median is the the average of the
first and the last integer
Median of S =(a+b)/2 therefore a=b/2
Median of Q=(b+c)/2 therefore b= (3/4)c
Thus a= (3/8)c
Median of R = (a+c)/2 = (11/16)c

2
A basket contains 5 apples, of which 1 is spoiled and the rest are good. If
Henry is to select 2 apples from the basket simultaneously, what is the
probability that the two apples selected will include the spoiled apple?

1/5 3/10 2/5 1/2 3/5

(probability, medium)

Since the ratio of the apples chosen to the total number of apples is 2 to 5, the
probability that the two apples selected will include the spoiled apple is 2/5.

Other ways of arriving at the same result:

Pr (first apple is spoiled) + Pr (second apple is spoiled) = 1/5 + (4/5)(1/4) =2/5


(for the second apple to be spoiled, the first must be one of the 4 good apples)

1 –Pr (both are good)= 1 – (4/5) (3/4) = 2/5

1/5 3/10 2/5 1/2 3/5

3
A boat traveled upstream a distance of 90 miles at an average speed of (v-3)
miles per hour and then traveled the same distance downstream at an average
speed of (v+3) miles per hour. If the trip upstream took half hour longer than
the trip downstream, how many hours did it take the boat to travel
downstream?

2.5 2.4 2.3 2.2 2.1

(movement, hard)

Remember that distance = rate x time.


Thus the trip upstream took 90/(v-3) hours and the trip downstream took
90/(v+3) hours.

Thus 90  90  1  90(v  3)  2  90(v  3)  2  (v  3)(v  3)


v 3 v3 2

4(3)(90)  v 2  9  v 2  1089  v  33

(As 302  900  1089 and only numbers with units digit 3 and 7 have squares
that have units digit 9, it is clear that v=33.)

Since the time taken downstream is 90 , the correct answer is


v3
90/36=45/18=5/2=2.5 hours.

Another way to solve for v: 180/(v-3) = 180/(v+3) + 1. Take advantage of


the fact that in the GMAT, velocity is usually an integer: lLook for two factors of
180 that differ by 6 and whose pairs differ by 1: 30 (30 × 6) and 36 ( 36 × 5).
Thus v=33

2.5 2.4 2.3 2.2 2.1

4
A bookstore that sells used books sells each of its paperback books for a certain
price and each of its hardcover books for a certain price. If Joe, Mary and Paul
bought books in this store, how much did Mary pay for one paperback book and
one hardcover book?

(1) Joe bought 2 paperback books and 3 hardcover books and paid $12.50.
(2) Paul bought 4 paperback books and 6 hardcover books and paid $25.00.

(algebra, medium)

Suppose that p is the price of each paperback book and h is the price of each
hardcover book. Mary bought one of each, so we need the value of p+h.

(1) says that 2p + 3h = 12.5. Clearly not sufficient: if p=h, p+h=5, but if h=2p,
p=5/4 and h=5/2, making p+h=15/4

(2) says that 4p + 6h = 25, an equation that is equivalent to that given by (1).
Thus (2) is not sufficient, not even in conjunction with (1).

(T) NOT SUFF

5
A box contains 10 light bulbs, fewer than half of which are defective. Two bulbs
are to be drawn simultaneously from the box. If n of the bulbs in box are
defective, what is the value of n?

(1) The probability that the two bulbs to be drawn will be defective is 1/15.
(2) The probability that one of the bulbs to be drawn will be defective and
the other will not be defective is 7/15.

(probability, hard)

(1) The greater the value of n ( a non-negative integer), the higher will be
the probability that both drawn bulbs are defective. Thus, as (1) gives
the exact probability, we can determine the value of n:

n/10 x (n-1)/9 = n(n – 1) /90 = 1/15.


Therefore n(n – 1) = 6 and n= 3 SUFF

(2) The probability that only the first is defective is n/10 x (10 – n)/9. The
proability that only the second is defective is (10 – n)/10 x n/9, the
same. Thus (2) tells us that 2n (10 – n )/90 =7/15
n(10 – n) = 21. Since 21 = 3 x 7, n could be 3 or 7. However, as it is
given that n < 5, n must be 3 SUFF

6
A can manufacturer has 5 identical machines, each of which produces cans at
the same constant rate. How many cans will all 5 machines running
simultaneously produce in z hours ?

(1) Running simultaneously, 3 of the machines produce 72,000 cans in 2z


hours.
(2) Running simultaneously, 2 of the machines produce 24,000 cans in z
hours.

(combined work, hard)

Note that the number of can produced is directly prortional to the number of
machines working and to the number of hours the machines work.

(1) If 3/5 of the 5 machines produce 72,000 cans in twice the z hours, all 5
machines running simultaneously produce 72,000(5/3)/2 cans in z
hours. SUFF
(2) If 2/5 of the 5 machines produce 24,000 cans in z hours, all 5 machines
running simultaneously produce 24,000(5/2) cans in z hours. SUFF

7
A car traveling at a certain constant speed takes 2 seconds longer to travel 1
kilometer than it would take to travel 1 kilometer at 75 kilometers per hour. At
what speed, in kilometers per hour, is the car traveling?

71.5 72 72.5 73 73.5

(movement,hard)

It would take 1/75 of an hour to travel 1 kilometer at 75 kilometers per hour,


and 1/75 of a hour is 60/75= 4/5 of a minute = 48 seconds. Thus the car will
take 50 seconds (5/6 of a minute) to travel 1 kilometer. In 1 minute, this car
would travel 6/5 of a kilometer, and in 60 minutes , 72 kilometers.

Also note that for a constant distance, tava = tbvb, so va = =

71.5 72 72.5 73 73.5

8
A cash register in a certain clothing store is the same distance from two
dressing rooms in the store. If the distance between the two dressing rooms is
16 feet, which of the following could be the distance between the cash register
and either dressing room?

I. 6 feet
II. 12 feet
III. 24 feet

I only II only III only I and II II and III

(geometry, medium)

The placement of the cash registers can be represented by the three vertices of
an isosceles triangle. We know that AC= 16, and since the length of any side of
a triangle must be less than the sum of the other two sides, the lengths of the
other two sides must be greater than 8 feet.

I only II only III only I and II II and III


9
According to the directions on a can of frozen orange juice concentrate, 1 can
of concentrate is to be mixed with 3 cans of water to make orange juice. How
many 12-ounce cans of the concentrate are required to prepare 200 6-ounce
servings of orange juice?

25 34 50 67 100

(ratios, medium)

One can of concentrate generates 4 12-ounce cans of juice. Thus x cans of


concentrate generates 4(12)x ounces of juice. As we need 200 6-ounce
servings, we can solve (4)(12) x  200(6) . Dividing, we see that 2 x  50 and thus
25 cans of concentrate are needed.

25 34 50 67 100
10
A certain basket contains 10 apples, 7 of which are red and three of which are
green. If 3 different apples are to be selected at random from the basket, what
is the probability that 2 of the apples selected will be red and 1 will be green?

7/40 7/20 49/100 21/40 7/10

(probability, hard)

Recall the formula for the number of unordered subsets of size k of a set of size
n! n(n  1)...(n  k  1)
n. n Ck   . The required probability is 7 C2 3 C1
(n  k )!k! k!
divided by 10 C3 . Thus the answer is 21(3)/120= 21/40

7/40 7/20 49/100 21/40 7/10

11
A certain bank charges a maintenance fee on a standard checking account each
month that the balance falls below $1000 at any time during the month. Did the
bank charge a maintenance fee on Sue's standard checking account last
month?

(1) At the beginning of last month, Sue's account balance was $1500.00
(2) During last month, a total of $2000.00 was withdrawn from Sue's
checking account.

(algebra, medium)

(T) No information is given about possible deposits. NOT SUFF

12
A certain business produced x rakes each month from November through
February and shipped x/2 rakes at the beginning of each month from March
through October. The business paid no storage costs for the rakes from
November through February, but it paid storage costs of $0.10 per rake each
month from March through October for the rakes that had not been shipped. In
terms of x, what was the total storage cost, in dollars, that the business paid
for the rakes for the 12 months from November through October?

0.40x 1.20x 1.40x 1.60x 3.20x

(algebra, medium)

From November through February, 4x rakes are produced. So, as x/2 rakes are
shipped at the beginning of each of 8 months starting with March. The business
is charged $0.10 for every rake-month. The sum of the rake months is x/2
multiplied by (1+2+3+...+7)=1.4x

0.40x 1.20x 1.40x 1.60x 3.20x

13
A certain candy manufacturer reduced the weight of candy bar M by 20 percent
but left the price unchanged. What was the resulting percent increase in the
price per ounce of candy bar M?

5% 10% 15% 20% 25%

(percents, medium)

Suppose that the original weight in ounces and price of candy bar M were w
and p. This the original price per ounce was p/w and the new price per ounce
was p/0.8w = 5/4 (p/w), 25% higher than the original.

5% 10% 15% 20% 25%


14
A certain car averages 25 miles per gallon of gasoline when driven in the city
and 40 miles per gallon when driven on the highway. According to these rates,
which of the following is closest to the number of miles per gallon that the car
averages when it is driven 10 miles in the city and then 50 miles on the
highway?

28 30 33 36 38

(ratios, medium)

The car is driven a total of 60 miles and uses 10/25 + 50/40 = 0.4 + 1.25
=1.65 ≈ 5/3 gallons. Dividing the number of miles by the number of gallons we
get about 36 miles per gallon, or to be exact, 3600/165 miles per gallon :

Alternatively, use fractions: For the 10 miles driven in the city, 10/25 = 2/5 of a
gallon of gas was used. For the 50 miles driven on the highway, 50/40 =5/4
gallons of gas was used. Thus a total of 33/20 gallons were used, at rate of 60
÷ (33/20) = 20×60/33 =400/11 miles per gallon.
Note that 400/11 = 3600/99 ≈ 36

28 30 33 36 38

15
A certain characteristic in a large poplulation has a distribution that is
symmetric about the mean m. If 68 percent of the distribution lies within one
standard deviation d of the mean, what percent of the distribution is less than
m + d?

16% 32% 48% 84% 92%

(statistics, medium)

As the distribution that is symmetric about the mean m, the percent that is not
within one standard deviation about m (32%), can be divided into two equal
parts, that above m + d and that below m – d . Thus 16% of the values are
above m + d, which means that 84% are below m + d.

16% 32% 48% 84% 92%

16
A certain circular area has its center at point P and radius 4, and points X and Y
lie in the same plane as the circular area. Does point Y lie outside the circular
area?

(1) The distance between point P and point X is 4.5.


(2) The distance between point X and point Y is 9.

(geometry, hard)

(1) Tells us nothing about Y. X is 0.5 units outside the circle. NOT SUFF
(2) Nothing is said about point P. NOT SUFF
(T) Since the diameter of the circle is 8, Y must be at least 0.5 units outside
the circle. SUFF

17
A certain city with a population of 132,000 is to be divided into 11 districts, and
no district is to have a population that is more than 10% greater than the
population of any other district. What is the minimum possible population that
the least populated district could have?

10,700 10,800 10,900 11,000 11,100

(percents, hard)

Remember that the sum of the population of the 11 districts must be 132,000,
so to mimimize the population of the least populated district, we need to
maximize that of the 10 other districts. Letting the population of the least
populated district be x thousand, the population each of the other 10 could be
as large as 1.1x. Thus to mimimize x, solve x+10(1.1x)=132. Thus 12x=132
and x=11

10,700 10,800 10,900 11,000 11,100

18
A certain cloth with a diameter of 20 inches is placed in a circular tray with a
diameter of 24 inches. What fraction of the tray’s surface is not covered by the
cloth?
1/6 1/5 11/36 25/36 5/6

(geometry, medium)

The tray’s surface area is = 144 , while that of the cloth is 100
(144 - 100 ) /144 = 11/36. Also, note that the area of a circle is directly
proportional to the square of its radius. Thus the ratio of the areas is the square
of the ratio of their radii, which is 10:12 = 5:6. The ratio of the areas, then, is
25/36, so 11/36 of the tray’s surface is not covered by the cloth.

1/6 1/5 11/36 25/36 5/6

19
A certain club has 20 members. What is the ratio of the number of 5-member
committees that can be formed from the members of the club to the number of
4-member committees that can be formed from the members of the club?

16 to 1 15 to 1 16 to 5 15 to 6 5 to 4

(combinatronics, medium)

We are asked for the ratio of 20 C5 : 20 C4 , which is equal to


20  19  18  17  16
divided by 20  19  18  17 = 16 to 5. .
5! 4!

16 to 1 15 to 1 16 to 5 15 to 6 5 to 4

20
A certain company assigns employees to offices in such a way that some of the
offices can be empty and more than one employee can be assigned to an
office. In how many ways can the company assign 3 employees to 2 different
offices?

5 6 7 8 9

(combinatronics, medium)

Suppose that the offices are A and B. Each of the 3 employees can be assigned
to either office, so there 2 x 2 x 2 = 8 ways the company can assign the 3
employees.
It would be more time-consuming to consider 2 cases:
Case I- all 3 employees go to the same office- 2 ways
Case II- 1 employee goes to one office, the other 2 go to another. Choose
which employee is alone (3 ways) and then which office she goes to (2 ways)-
2 x 3 = 6 ways
Total 8 ways

5 6 7 8 9

21
A certain company charges $6 per package to ship packages weighing less than
2 pounds each. For a package weighing 2 pounds or more, the company
charges an initial fee of $6 plus $2 per pound. If the company charged $38 to
ship a certain package, which of the following was the weight of the package,
in pounds.

16 17 19 20 22

(algebra, medium)

If a package weighs x pounds, the amount charged is 6 + 2x. For the amount
charged to be $38, 2x + 6 = 38, so x=16

16 17 19 20 22

22
A certain company divides its total advertising budget into television, radio,
newspaper, and magazine budgets in the ratio 8 : 7 : 3 : 2, respectively. How
many dollars are in the radio budget?

(1) The television budget is $18,750 more than the newspaper budget.
(2) The magazine budget is $7,500

(ratios, medium)

For some positive number x, the television (t), radio (r), newspaper (n) and
magazine (m) budgets are 8x, 7x, 5x and 2x. Any information that allows us to
find the value of x will permit us to determine how many dollars are in the radio
budget.

(1) t – n =18,750 8x – 3x = 5x = 18,750 SUFF


(2) m = 2x = 7,500 SUFF

23
A certain company expects quarterly earnings of $0.80 per share of stock, half
of which will be distributed as dividends to shareholders while the rest will be
used for research and development. If earnings are greater than expected,
shareholders will receive an additional $0.04 per share for each additional $0.10
of per share earnings. If quarterly earnings are $1.10 per share, what will be
the dividend paid to a person who owns 200 shares of the company’s stock?

$92 $96 $104 $120 $240

(ratios, medium)

If earnings were $0.80 per share, $0.40 per share would be paid as dividends

As earnings per share are actually $1.10 = $0.80 + 3($0.10), dividends per
share will be $0.40 + 3($0.04) =$0.52 . For 200 shares, the dividend paid will
be $104.

$92 $96 $104 $120 $240

24
A certain economics report defines a middle-income family as a family whose
income is at least half, but no more than twice, the median family income.
According to this report, is a family whose income is $73,000 considered a
middle-income family?

(1) The median family income is $37,152


(2) The mimimum income of a middle-class family is $18,576

(inequalities, medium)

(1) SUFF
(2) From (2), we can deduce (1). Half the median income is $18,576,
according to the definition. SUFF

25
A certain farmer pays $30 per acre per month to rent farmland. How much
does the farmer pay per month to rent a rectangular plot of farmland that is
360 feet by 605 feet? (43,560 square feet = 1 acre)

$5,330 $3,630 $1,350 $360 $150

(ratios, medium)

The number of square feet is 360(605), so the number of acres rented is


360(605)/(43560) = 5 acres, or 5(30) = 150 dollars per month.

$5,330 $3,630 $1,350 $360 $150

26
A certain group of car dealerships agreed to donate x dollars to a Red Cross
chapter for each car sold during a 30-day period. What was the total amount
that was expected to be donated?

(1) A total of 500 cars were expected to be sold.


(2) 60 more cars were sold than expected, so that the total amount
actually donated was $28,000.

(algebra, medium)

Suppose that the number that were expected to be sold is n. We are asked
about the value of xn .
(1) n = 500. Without knowing the value of x, we cannot answer the
question. NOT SUFF
(2) Suppose that the number that were expected to be sold is n.
Then (n + 60)x = 28,000. xn = 28,000 – 60x. Without knowing the
value of x, we cannot answer the question. NOT SUFF
(T) 560x = 28000, so x= 28000/560. As we know the value of x and the
value of n, we can answer the question. SUFF

27
A certain jar contains only b black marbles, w white marbles and r red marbles.
If a marble is picked at random from the jar, is the probability that the marble
chosen will be red greater than the probability that the marble chosen will be
white?

(1) r/(b + w) > w/(b + r)


(2) b-w>r

(probability, hard)

The probability of getting a red marble will be greater than that of getting a
white marble if and only if there are more reds than whites, that is, if r>w.
............................................................................................................
(1) The ratio of the number of reds to the number of non-reds is greater
than the ratio of the number of whites to the number of non-whites. This
indicates that there are indeed more reds than whites.
One way to see that this must be so is to simplify the inequality given in
(1) by using T (total) = r+w+b:
r w
  r (T  w)  w(T  r )  rT  wT  r  w SUFF
T r T w
....................................................................................................................
(2) b-w > r , in other terms b > r+w. This means that there are more black
marbles than red and white combined. Thus more than half of the
marbles are black.
However, we have no means of comparing r and w. NOT SUFF

28
A certain kennel will house 24 dogs for 7 days. Each dog requires 10 ounces of
dog food per day. If the kennel purchases dog food in cases of 30 cans each
and if each can holds 8 ounces of dog food, how many cases will the kennel
need to feed all of the dogs for 7 days?

5 6 7 8 9

(ratios, medium)

The number of ounces of food needed is 24 x 7 x 10

Each case contains 30 x 8 ounces of food.

Dividing, we get (24 x 7 x 10)/ 30 x 8 = 7

5 6 7 8 9

29
A certain law firm consists of 4 senior partners and 6 junior partners. How
many different groups of 3 partners can be formed in which at least one
member of the group is a senior partner? (Two groups are considered different
if at least one group member is different.)

48 100 120 288 600

(combinatronics, hard)

Rather than consider the four cases in which at least one senior partner is
chosen, it is faster to count the number of groups that do not include a senior
member, 6 C3 = 6 x 5 x 4 / 3! = 20, and subtract this from the total number of
groups, 10 C3 = 10 x 9 x 8/ 3 x 2 x 1 = 120.

48 100 120 288 600

30
A certain library assesses a fine for overdue books as follows. On the first day
the book is overdue, the total fine is $0.10. For each additional day that the
book is overdue, the total fine is either increased by $0.30 or doubled,
whichever results in the lesser amount. What is the total fine for a book on the
fourth day it is overdue?

$0.60 $0.70 $0.80 $0.90 $1.00

(algebra, medium)

The fine in cents on the first day is 10. On the second, it is the lesser
of 10+30 and 10(2) , i.e. 20. On the third, it is the lesser of 20+30 and
20(2), i.e. 40. On the fourth, it is the lesser of 40+30 and 40(2), i.e.
70.

$0.60 $0.70 $0.80 $0.90 $1.00

31
A certain list consists of 5 different integers. Is the average (arithmetic mean)
of the two greatest integers greater than 70?

(1) The median of the integers in the list is 70.


(2) The average of the integers in the list is 70.

(statistics, hard)

(1) The median of the integers is 70, so 70 is the third greatest integer. The
two integers greater than the median are both greater than 70, so the
average of these two integers will be greater than 70. SUFF

(2) If the average of the integers in the list is 70, we can think of the list as
two separate groups of integers, one that consists of the two greatest
integers and the other consisting of the three smallest integers. The
average of the first group is greater than the average of the second. As
the average of all of the integers is 70, it must be that the average of the
first group must be greater than 70. SUFF

Alternatively, if the average of the two greatest is not greater than 70, the sum
of these two integers is at most 140. As the sum of all five integers is
70(5)=350, the sum of the three smallest is at least 210, so the average of the
three smallest is at least 70. However, the average of the three smallest must
be less than that of the two greatest, as all the integers are different. Thus the
average of the two greatest must be greater than 70.
Some prefer to think as follows: if you remove the smallest element from a set
of numbers, the average of the remaining elements will be higher than that of
the numbers of the original set, 70. Remove the smallest of the remaining
elements, and the average of the rest will rise.

32
A certain list consists of several different integers. Is the product of all the
integers in the list positive?

(1) The product of the greatest and smallest of the integers in the list is
positive.
(2) There is an even number of integers in the list.

(algebra, medium)

In other words, the question is asking whether the number of negative integers
in the list is an even number.

(1) We know from (1) that either all the integers are positive, in which case,
the number of negative integers is indeed an even number (0 is an even
number), or all the integers are negative, in which case the number of
negative integers is equal to the number of integers in the set, a number
that may be odd or even. NOT SUFF.

(2) Alone, this tells us nothing about the number of negative integers in the
set. NOT SUFF

(T) We know that the number of negative integers in the set must be an
even number, either 0 (in the case that all of the integers are positive) or
the number of elements in the set, an even number, according to (2).
SUFF

33
A certain list of 100 data has a mean of 6 and a standard deviation of D, where
D is positive. Which of the following pairs of data, when added to the list, must
result in a list of 102 data with standard deviation less than D?

-6 and 0 0 and 0 0 and 6 0 and 12 6 and 6

(statistics, medium)

The standard deviation is a measure of how far the elements are from the
mean. If to a set with at least two distinct numbers is added an element equal
to the mean of that set, the standard deviation of the new set be less than that
of the old set. We are told that the standard deviation the list of 100 data is
positive, so if 6, the mean of the list of 100 data, is added twice, the resulting
list of 102 data will have a standard deviation that is less than D.

-6 and 0 0 and 0 0 and 6 0 and 12 6 and 6

34
A certain manufacturer of cake, muffin, and bread mixes has 100 buyers, of
whom 50 purchase cake mix, 40 purchase muffin mix, and 20 purchase both
cake mix and muffin mix. If a buyer is to be selected at random from the 100
buyers, what is the probability that the buyer selected will be one who
purchases neither cake mix nor muffin mix?

A. 1/10
B. 3/10
C. 1/2
D. 7/10
E. 9/10

35
A certain meter records voltage between 0 volts and 10 volts, inclusive. If the
average value of 3 recordings on the meter was 8 volts. What is the smallest
possible recording, in volts ?

2 3 4 5 6

(algebra, medium)

We know that the sum of the three recordings must be 3 x 8= 24. To minimize
the value of one recording, find the maximum value of the sum of the other
two recordings: 10+10=20. Thus, one recording could be as low as 24-20= 4

2 3 4 5 6

36
A certain movie depicted product A in 21 scenes, product B in 7 scenes, product
C in 4 scenes, and product D in 3 scenes. The four product manufacturers paid
amounts proportional to the number of scenes in which their product was
depicted in the movie. If each manufacturer paid x dollars per scene, how much
did the manufacturer of product D pay for this advertising?

(1) The manufacturers of product A and B together paid a total of $560,000


for this advertising.
(2) The manufacturer of product B paid $60,000 more for this advertising
than the manufacturer of product C paid.

(ratios, medium)

We need to find the value of 3x. Any means of finding the value of x would be
sufficient.

(1) 21x + 7x= 28x =560,000. 3x= 3(560,000)/28 SUFF


(2) 7x – 4x =3x = 60,000 SUFF

37
A certain one-day seminar consisted of a morning session and an afternoon
session. If each of the 128 people attending the seminar attended at least one
of the two sessions, how many of the people attended the morning session
only?

(1) 3/4 of the people attended both sessions.


(2) 7/8 of the people attended the afternoon session.

(sets, medium)

We can say that 128= |morning only| + |afternoon only| + |both morning and
afternoon|
From (1), we can deduce that at most 1/4 of the 128 people attended the
morning session only. As no information is given about the number of people
that attended the afternoon session only, we cannot say how many attended
the morning session only. NOT SUFF

(2) says that 7/8 of the people attended the afternoon session. Thus 1/8 did
not attend the afternoon session. As each of the 128 people attended at least
one of the two sessions, the number that attended the morning session only is
1/8 of 128. SUFF

38
A certain quantity is measured on two different scales, the R-scale and the S-
scale, that are related linearly. Measurements on the R-scale of 6 and 24
correspond to measurements on the S-scale of 30 and 60, respectively. What
measurement on the R-scale corresponds to a measurement of 100 on the S-
scale?

20 36 48. 60 84

(coordinate geometry, medium)

If the S scale were plotted on the y-axis, the relation between S (y) and R (x)
would be a line with a slope of (60-30)/(24-6)= 5/3. Thus if (x, 100) is one this
line, (100-60)/(x – 24) =5/3, so 5(x -24) = 3(40) and x -24 = 24.
Thus x=48.

20 36 48. 60 84

39
A certain restaurant offers 6 kinds of cheese and 2 kinds of fruit for its dessert
platter. If each dessert platter contains an equal number of kinds of cheese and
kinds of fruit, how many different dessert platters could the restaurant offer?

8 12 15 21 27

(combinatronics, medium)

We need to consider not only platters that have two kinds of fruit and two kinds
of cheese , 2 C2 6 C2 = 1 x 15= 15, but also platters that have one kind of fruit
and one kind of cheese (2 x 6=12).

8 12 15 21 27

40
A certain roller coaster has 3 cars, and a passenger is equally likely to ride in
any 1 of the 3 cars each time that passenger rides the roller coaster. If a
certain passenger is to ride the roller coaster 3 times, what is the probability
that the passenger will ride in each of the 3 cars?

0 1/9 2/9 1/3 1

(probability, medium)

For this to happen, the car assigned on the second ride has to be different from
the one assigned on the first and the one assigned on the third has to be the
one car not assigned on either of the first two rides. Thus the required
probability is 2/3 × 1/3 = 2/9. Alternatively, there are 33=27 equally probable
car assignments (aaa,aba,aab,caa,…,ccc), of which 3! involve rides on the 3
different cars: 3!/27 = 2/9.

0 1/9 2/9 1/3 1

41
A certain right triangle has sides of length x, y and z where x < y < z. If the
area of this triangular region is 1, which of the following indicates all of the
possible values of y?

3 2 3 3 2
y 2  y 2 y y
2 3 2 4 3
3
y
4

(geometry, hard)

The smaller the value of x, the larger the value of y. As there is no limit as to
how how close to 0 x is, there is no limit as to how large y can be. Only the first
choice reflects this fact.

More formally, as this is a right triangle, x and y are the lengths of the legs, and
z is the length of the hypotenuse. The area of this triangle is xy/2 < y2 / 2 since
x < y . Thus y2 / 2 > 1 and y > 2

y 2 3
 y 2
2
y
3 3
y
2
2 3 2 4 3
3
y
4

42
A certain state has a sales tax of 2 percent on the purchase price of all
products. In addition, a city within this state imposes its own 0.5 percent sales
tax on the purchase price of all products. If the sales tax on a particular product
purchased in this city was $2.80, what was the purchase price of this product?

40 56 112 137 140

(percents, medium)

A purchase p in this city is levied a total sales tax of 2.5%.


Thus 2.5% of the p is 2.80, so 25% of p = 28 .
Multiplying both sides by 4, p= 112

40 56 112 137 140

43
A certain stock exchange designates each stock with a one , two, or three letter
code where each letter is selected from 26 letters of the alphabet. If the letters
may be repeated and if the same letters used in a different order constitute a
different code, how many different stocks is it possible to uniquely designate
with these codes?

2,951 8,125 15,600 16,302 18,278

(combinatronics, hard)

We need to count one, two and three-letter codes: 26 + 26x26+ 26x26x26.


Note that the units digit of the sum will be 8, an observation that can save
calculating time, but only if you are short of time!

2,951 8,125 15,600 16,302 18,278

44
A certain store sells chairs individually or in sets of 6. The store charges less for
purchasing a set of 6 chairs than for purchasing 6 chairs individually. How much
does the store charge for purchasing a set of 6 chairs?

(1) The charge for purchasing a set of 6 chairs is 10 percent less than the
charge for purchasing the 6 chairs individually.
(2) The charge for purchasing a set of 6 chairs is $20 more than the charge
for purchasing the 5 chairs individually.

(percents, medium)

(1) If the charge for each chair bought individually is x, the charge for the
set is 0.9(6x). However, no information about x is given. NOT SUFF

(2) The set price is 5x + 20, where x is the price of one chair bought
individually. No information is given about x, though. NOT SUFF

(T) We know 0.9(6x)= 5x +20, so x and thus the set price can be
found. SUFF

45
A certain telephone company offers two plans, A and B. Under plan A, the
company charges a total of $0.70 for the first 7 minutes of each call and $0.06
per minute thereafter. Under plan B, the company charges $0.08 per minute of
each call. What is the duration of the call, in minutes, for which the company
charges the same amount under plan A and under plan B?

2 9 14 21 30

(algebra, medium)

Consider an x minute call, where x is greater than 7: Under plan A, 0.7


+0.06(x-7) is charged, and under plan B, 0.08x is charged. Equating the two
charges and solving for x:

0.02x=0.28 , 2x=28, x=14

2 9 14 21 30

46
A certain theater has a total of 884 seats, of which 500 are orchestra seats and
the rest are balcony seats. When tickets for all the seats in the theater are sold,
total revenue from ticket sales is $34,600. What was the theaters total revenue
from ticket sales for last night’s performance?

(1) The price of an orchestra ticket is twice the price of a balcony ticket.
(2) For last night’s performance, tickets for all the balcony seats but only 80
percent of the tickets for the orchestra were sold.

(algebra, medium)

(1) Suppose the price of a balony seat is p dollars. Then 500(2p) + 384(p)
=34,600. p can be found, so the price of each type of ticket can be
found. However, we know nothing about last night’s sales. NOT SUFF
(2) We cannot determine the exact price of each type of ticket.
NOT SUFF
(T) The total revenue, 334p + 0.8(500)2p can be found, as (1) gives us the
value of p. SUFF

47
A certain triangle has two angles measuring 45º and 75º If the side opposite
the 45º angle has length 6, what is the length of the side opposite the 75º
angle?

23 3 3( 3  1) 6 6 3 26 2

(geometry, hard)

Remember the two right isosceles triangles we saw in class!

23 3 3( 3  1) 6 6 3 26 2
CORRECT

48
A child selected a three-digit number, XYZ, where X, Y and Z denote the digits
of the number and X + Y + Z= 10. If no two of the three digits were equal,
what was the three-digit number?

(1) X<Y<Z
(2) The three-digit number selected was even.

(factors and multiples, easy)

(1) XYZ could be 127, 136, 145 or 235 NOT SUFF


(2) XYZ could be 136 or 226 or one of many other possibilities
NOT SUFF
(T) XYZ must be 136

49
A circular jogging track forms the edge of a circular lake that has a diameter of
2 miles. Johanna walked once around the track at the average rate of 3 miles
per hour. If t represents the number of hours it took Johanna to walk
completely around the lake, which of the following is a correct statement?

0.5 < t < 0.75 1.75 < t < 2.0 2.0 < t < 2.5
2.5 < t < 3.5 3 < t < 3.5

(geometry, movement, medium)

Remember that distance = rate x time, so the time Johanna took (in hours) is
the distance she walked (in miles), the circumference of a circle with diameter
2, divided by the rate at which she walked ( in miles per hour)-

So t= 2  3 . As  is slightly more than 3, t is slightly more than 2.

0.5 < t < 0.75 1.75 < t < 2.0 2.0 < t < 2.5
2.5 < t < 3.5 3 < t < 3.5

50
A clock store sold a certain clock to a collector for 20 percent more than the
store had originally paid for the clock. When the collector tried to resell the
clock to the store, the store bought it back at 50 percent of what the collector
had paid. The shop then sold the clock again at a profit of 80 percent on its
buy-back price. If the difference between the clock's original cost to the shop
and the clock's buy-back price was $100, for how much did the shop sell the
clock the second time?

$270 $250 $240 $220 $200

(percents, hard)

original cost = x
collector price = 1.2x
buy back price = 0.6x
resell price = 1.08x
x- 0.6x = 100 => x = 100/0.4 = 250
resell price = 1.08 × 250 = 270

$270 $250 $240 $220 $200

51
A clothing store acquired an item at a cost of x dollars and sold the item for y
dollars. The store’s gross profit from the item was what percent of its cost for
the item?

(1) y – x = 20
(2) xy = 45

(algebra, percents, medium)

We are asked for the value of ((y – x)/x)100%. It is sufficient to know the value
of y/x

(1) NOT SUFF


(2) NOT SUFF
(T) With the two equations, we can say that (y – 20)y =45 or
y2 – 20y – 45 = 0, a quadratic equation that has two solutions of
opposite signs, Since the selling price y must be positive, we can
determine the one possible value of y and thus of x.
SUFF

52
A collection of 36 cards consists of 4 sets of 9 cards each. The 9 cards in each
set are numbered 1 through 9. If one card has been removed from the
collection, what is the number on that card?

(1) The units digit of the sum of the numbers on the remaining 35 cards
is 6.
(2) The sum of the numbers on the remaining 35 cards is 176.

Note that we can find the sum of the entire collection. One need not do so, but
it is 4(1 + 2 + ... + 8 + 9) = 4(9)(5) = 180 (an arthemetic sequence with first
and last terms a and b has a sum of (a+b)n/2, where n is the number of terms.

(1) If we know the units digit of the sum of the remaining 35 cards, we
can determine the units digit of the card removed. Since the number
on each card is a one-digit number. SUFF
(2) SUFF

53
A college admissions officer predicts that 20 percent of the students who are
accepted will not attend the college. According to this prediction, how many
students should be accepted to achieve a planned enrollment of x students?

1.05x 1.1x 1.2x 1.25x 1.8x

(percents, medium)

Suppose that n are accepted. 0.2n will not enroll. Thus 0.8n= x and
n = x/0.8 = x/(4/5) = 5x/4 =1.25x.

1.05x 1.1x 1.2x 1.25x 1.8x

54
A combined total of 55 lightbulbs are stored in two boxes; of these, a total of 7
are broken. If there are exactly two broken bulbs in the first box, what is the
number of bulbs in the second box that are not broken?

(1) In the first box, the number of bulbs that are not broken is 15 times the
number of broken bulbs.

(2) The total number of bulbs in the first box is 9 more than the total
number of bulbs in the second box.

(sets, medium)

BOX 1 BOX 2 TOTAL

BROKEN 2 5 7

NOT BROKEN 48 - x x 48

TOTAL 50 - x x+5 55

We are asked for the value of x.

(1) 48 – x = 15(2) SUFF


(2) 50 – x = (x + 5) + 9 SUFF

55
A company has 2 types of machines, type R and type S. Operating at a constant
rate, a machine of type R does a certain job in 36 hours and a machine of type
S does the job in 18 hours. If the company used the same number of each type
of machine to do the job in 2 hours, how many machines of type R were used?

4 5 6 8 10

(combined work, medium)

Suppose x of each type of machine were used to do the job in 2 hours. Thus
the fraction of the job that all machines do in 1 hour is 1/2. We can write:

x x 1 3x x 1
     x6
36 18 2 36 12 2

4 5 6 8 10

56
A company wants to spend equal amounts of money for the purchase of two
types of computer printers costing $600 and $375 per unit, respectively. What
is the fewest number of computer printers that the company can purchase?

13 12 10 8 5

(ratios, factors and multiples, hard)

If x $600 printers and y $375 printers are to be pruchased, it must be that


x:y=375:600=75:120=15:24=5:8. Thus we know that the company will buy 5k
$600 printers and 8k $375 printers. The number of printers bought, then is 13k,
a multiple of 13. 13 is the smallest multiple of 13.

13 12 10 8 5

57
A company wants to buy printers and computers for a new branch office and
the number of computers can be at most 3 times the number of printers
Computers cost $1500 each and printer cost $300 each what is the greatest
number of computers that a company can buy if it has a total of $9100 to
spend on computers and printers ?

2 3 4 5 6

(ratios, hard)

Ideally, each printer would be accompanied by 3 computers. This bundle


costs $4,800. Two such bundles (a total of 2 printers and 6 computers) would
cost $9,600, $500 more than the limit. 2 printers and 5 computers would cost
$1000 less than the limit.

2 3 4 5 6

58
Shipment Number of defective Total number of
chips in the shipment chips in the
shipment
S1 2 5,000
S2 5 12,000
S3 6 18,000
S4 4 16,000

A computer chip manufacturer expects the ratio of the number of defective


chips to the total number of chips in all future shipments to equal the
corresponding ratio for shipments S1, S2, S3 and S4 combined, as shown in the
following table. What is the expected number of defective chips in a shipment
of 60,000 chips?

14 20 22 24 25

(ratios, medium)

Overall there were 17 defective chips out of a total of 51,000 chips. The ratio of
the number of defective chips to the total number of chips in the shipment is
17/51,000 = 1/3,000. Thus in 60,000 =3,000 x 20 chips, 20 defective chips can
be expected.

14 20 22 24 25

59
A construction company was paid a total of $500,000 for a construction project.
The company’s only costs for the project were for labor and materials. Was the
company’s profit for the project greater than $150,000?

(1) The company’s total cost was three times its cost for materials.
(2) The company’s profit was greater than its cost for labor.

(inequalities, hard)

The profit P = 500 – L – M


The question is whether P> 150 i.e. if L+M < 350

(1) L + M = 3M, i.e. L=2M NOT SUFF

(2) P=500-L-M> L 2L + M < 500 NOT SUFF

(1) and (2) 5M < 500 M< 100 L = 2M < 200 L + M < 300 < 350
SUFF

60
A contractor combined x tons of a gravel mixture that contained 10% gravel G,
by weight, with y tons of a mixture that contained 2% gravel G, by weight, to
produce z tons of a mixture that was 5% gravel, by weight. What is the value
of x?

(1) y = 10
(2) z = 16

(percents, ratios, hard)

Note that the information before the question does not give us the means to
find x, y or z, but does allow us to find the ratios of these variables:

0.1x + 0.02y=0.05(x+y) so y/x is 5/3.


As z=x+y, x/z= x/(y+x)= 3/8

Thus each of (1) and (2) provides sufficient information to find x.

61
A contest will consist of n questions, each of which is to be answered either
"True" or "False". Anyone who answers all n questions correctly will be a
winner. What is the least value of n for which the probability is less than 1/1000
that a person who randomly guesses the answer to each question will be a
winner?

5 10 50 100 1000

(probability, hard)

The probability that a contestant will answer a given question correctly is 1/2.
Therefore the probability that this person will answer n questions in succession
is (1/2)n. If (1/2)n < 10 -3, 2n> 103. Given that 29= 512 and 210= 1024, n must
be greater than or equal to 10.

5 10 50 100 1000

62
According to a survey, 93 percent of teenagers have used a computer to play
games, 89 percent have used a computer to write reports, and 5 percent have
not used a computer for either of these purposes. What percent of the
teenagers in the survey have used a computer both to play games and to write
reports?

82% 87% 89% 92% 95%

(sets, medium)

Suppose that n teenagers were surveyed. 0.93n have used a computer to play
games, 0.89n have used a computer to write reports, and 0.05n percent have
not used a computer for either of these purposes. If y teenagers have used
computers for both purposes, n=0.93n + 0.89n + 0.05n – y, so y=0.87n

82% 87% 89% 92% 95%

63
A craftsman made 126 ornaments and put them all into boxes. If each box
contained either 6 ornaments or 24 ornaments, how many of the boxes
contained 24 ornaments?

(1) Fewer than 4 of the boxes contained 6 ornaments.


(2) More than 3 of the boxes contained 24 ornaments.

(inequalities, factors and multiples, hard)

Suppose there were x boxes containing 6 ornaments and y containing 24


ornaments. 6x + 24y = 126 i.e. x +4y =21. We are asked for the value of y =
21- x) /4. Note that as x and y are integers, x must be 1 greater than a multiple
of 4 (21 is 1 greater than a multiple of 4)

(1) As x < 4, x must be 1, and we can calculate the value of y. SUFF


(2) Note that x = 21- 4y. As y > 3, y can be either 4 or 5. NOT SUFF

64
Adam and Beth each drove from Smallville to Crown City by different routes.
Adam drove at an average speed of 40 miles per hour and completed the trip in
30 minutes. Beth’s route was 5 miles longer, and it took her 20 minutes more
than Adam to complete the trip: How many miles per hour was Beth’s average
speed on this trip?

24 30 48 54 75

(movement, medium)

Adam’s route is 40(1/2) = 20 miles, so Beth’s is 25 miles. As Beth drove


30+20=50 minutes, 5/6 of one hour, her average speed was 25/(5/6) = 30
miles per hour.
24 30 48 54 75

65
A driver completed the first 20 miles of a 40-mile trip at an average speed of 50
miles per hour. At what average speed must the driver complete the remaining
20 miles to achieve an average speed of 60 miles per hour?

65 68 70 75 80

(movement, medium)

To achieve an average speed of 60 miles an hour for the 40 mile trip, the
distance must be covered in 40/60= 2/3 hour. The first 20 miles took
20/50=2/5 hour, so the remaining 20 miles must be covered in 2/3 -2/5 =4/15
hour. Thus the average speed required for the remaining 20 miles is 20÷(4/15)
=75 miles per hour.

65 68 70 75 80

66
A family-size box of cereal contains more cereal and costs more than the
regular-size box of cereal. What is the cost per ounce of the family-size box of
cereal?

(1) The family-size box of cereal contains 10 ounces more than the regular-
size box of cereal.
(2) The family-size box of cereal costs $5.40.

(algebra, ratios, easy)

(T) As no information is given about the number of ounces in a regular box


of cereal, more information is needed. NOT SUFF

67
A farm used two harvesting machines, h and k, to harvest 100 acres of wheat.
Harvesting machine h, working alone at its constant rate harvested 40 acres of
wheat in 8 hours. Then harvesting machine k was brought in, and harvesting
machines h and k, working together at their respective constant rates,
harvested the remaining acres of wheat in 5 hours. Harvesting machine k
harvested how many acres of wheat per hour?

7 8 12 13 15

(combined work, medium)

Machine h harvested 40 acres in 8 hours, so it harvests 5 acres per hour. In the


13 hours it worked, it harvested 65 of the 100 acres. The remaining 35 acres
were harvested in 5 hours by k, at a rate of 7 per hour.

7 8 12 13 15

68
A five-member committee is to be formed from a group of five military officers
and nine civilians. If the committee must include at least two officers and two
civilians, in how many different ways can the committee be chosen?

119 1200 3240 3600 14400

(combinatronics, hard)

Such a committee must consist of either 3 officers and 2 civilians or 2 officers


and 3 civilians.

There are 5C3 x 9C2 committees that can be formed with 3 officers and 2
civilians 10 x 36 = 360

There are 5C2 x 9C3 committees that can be formed with 2 officers and 3
civilians 10 x 84 = 840

Thus there are 1200 different ways the committee can be chosen.

A common mistake is to choose two officers and two civilians and then choose
a fifth person from among the 10 people not chosen. This results in double
counting.

119 1200 3240 3600 14400

69
A folk group wants to have one concert on each of the seven consecutive nights
starting January 1 of next year. One concert is to be held in each of cities A, B,
C, D and E. Two concerts are to be held in city F, but not on consecutive nights.
In how many ways can the group decide on the venues for these seven
concerts?

10 x 5! 14 x 5! 15 x 5! 20 x 5! 21 x 5!

(combinatronics, hard)

Step I: Decide which two non-consecutive concerts will be be held in F:


This can be done in 7 C 2 - 6 = 15 ways. (Exclude the 6 placements in which the
two dates are consecutive.

Step II: Assign venues A, B, C, D and E to the remaining 5 concerts: 5! ways.

Answer: 15 x 5!

10 x 5! 14 x 5! 15 x 5! 20 x 5! 21 x 5!

70
A 4-person task force is to be formed from the 4 men and 3 women who work
in company G's human resources department. If there are to be 2 men and 2
women on this task force, how many task forces can be formed?

14 18 35 56 144

(combinatronics, medium)

There are C2  6 ways to choose the men for the task force and 3 ways to
4

decide which woman will not serve on this task force. Therefore there are 3 x 6
=18 ways to form this task force.

14 18 35 56 144

71
A glass was filled with 10 ounces of water, and 0.01 ounce of the water
evaporated each day during a 20-day period. What percent of the original
amount of water evaporated during this period?

0.002% 0.02% 0.2% 2% 20%

(percents, medium)

Over the 20 days, 0.20 ounces evaporated, and (0.2/10) x 100% = 2%

0.002% 0.02% 0.2% 2% 20%

72
Alice’s take-home pay last year was the same each month, and she saved the
same fraction of her take-home pay each month. The total amount of money
that she had saved at the end of the year was 3 times the amount of that
portion of her monthly take-home pay that she did not save. If all the money
that she saved last year was from her take-home pay, what fraction of her
take-home pay did she save each month?

1/2 1/3 1/4 1/5 1/6

(algebra, hard)

Suppose that she she saved fraction f of her take-home pay p, where 0<f<1.
Then she did not save fraction 1-f of her take-home pay. We are given that the
amount that she had saved at the end of the year, 12fp is three times the
portion of her monthly income that she did not save, (1-f)p. Thus 12fp=3(1-f)p
and f=1/5.

1/2 1/3 1/4 1/5 1/6

73
A jar contains 16 marbles, of which 4 are red, 3 are blue, and the rest are
yellow. If 2 marbles are to be selected at random from the jar, one at a time
without being replaced, what is the probability that the first marble selected will
be red and the second marble selected will be blue?

3/64 1/20 1/16 1/12 1/8

(probability, hard)

Let R1 and B2 be the events that the the first marble drawn is red and the
second marble drawn is blue.

4 3 1
Pr(R1  B2 )  Pr(R1 )  Pr(B2 | R1 )   
16 15 20

3/64 1/20 1/16 1/12 1/8

74
All of the stocks on the over-the-counter market are designated by either a 4-
letter or a 5-letter code that is created by using the 26 letters of the alphabet.
Which of the following gives the maximum number of different stocks that can
be designated with these codes?

2(265) 26(264) 27(264) 26(265) 27(265)

(combinatronics, hard)

We need to consider both 4-letter and 5-letter codes. There are 264 four-letter
codes and 265= 26(264) five-letter codes, so the maximum number of stocks
that can be so designated is 27(264).

2(265) 26(264) 27(264) 26(265) 27(265)

75
A furniture dealer purchased a desk for $150 and then set the selling price
equal to the purchase price plus a markup that was 40 percent of the selling
price. If the dealer sold the desk at the selling price, what was the amount of
the dealer's gross profit from the purchase and the sale of the desk?

$40 $60 $80 $90 $100

(percents, medium)

Suppose the selling price was s dollars.


Then the gross profit was s – 150. As s= 150 + 0.4s, 0.6s=150 and s=250. The
gross profit, therefore, is 250-150=100.

$40 $60 $80 $90 $100

76
A glass was filled with 10 ounces of water, and 0.01 ounce of the water
evaporated each day during a 20-day period. What percent of the original
amount of water evaporated during this period?

0.002% 0.02% 0.2% 2% 20%

(precents, medium)

In all 20/100 =0.2 ounces of the 10 ounces of water evaporated. Since 0.2/10
= 2/100, 2% of the 10 ounces of water evaporated.

0.002% 0.02% 0.2% 2% 20%

77
A grocer has 400 pounds of coffee in stock, 20% of which are decaffeinated. If
the grocer buys another 100 pounds of coffee of which 60% is deffeinated,
what percent, by weight, of the grocer’s stock of coffee is decaffeinated ?

28 % 30 % 32 % 34 % 40 %

(percents, medium)

Of the 500 pounds of coffee in stock, 0.2(400) + 0.6(100) = 140 are


decaffeinated. 140/500 = 280/1000 = 28%

28 % 30 % 32 % 34 % 40 %

78
A grocer stocks oranges in a pile.The bottom layer was rectangular with 3 rows
of 5 oranges each. In the second layer from the bottom, each orange rested on
4 oranges from the bottom layer and in the third layer, each orange rested on 4
oranges from the second layer. Which of the following is the maximum number
of oranges that could have been in third layer?

5 4 3 2 1

(geometry, medium)

The second layer from the bottom could have as many as 2 x 4 = 8 oranges,
and the third layer from the bottom could have as many as 1 x 3 =3 oranges.

5 4 3 2 1

79
A hiker walking at a constant rate of 4 miles per hour is passed by a cyclist
travelling in the same direction along the same path at a constant rate of 20
miles per hour. The cyclist stops to wait for the hiker 5 minutes after passing
her, while the hiker continues to walk at her constant rate. How many minutes
must the cyclist wait until the hiker catches up?

2 2
6 15 20 25 26
3 3

(movement, hard)

Remember that d (distance) =v (velocity) multiplied by t (time).

In the five minutes (1/12 hour) that elapses between the moment when the
cyclist passed the hiker and the moment when the cyclist stopped, the distance
between the two people increased at a rate of 20 – 4 = 16 miles per hour. Thus
the distance between the two people when the cyclist stopped was 16/12 = 4/3
mile. It will take the hiker 4/3 ÷ 4 = 1/3 hour (20 minutes) to cover this
distance.

6
2
15 20 25 26
2
3 3

80
A lawyer charges her clients $200 for the first hour of her time and $150 for
each additional hour. If the lawyer charged her new client $1550 for a certain
number of hours of her time, how much was the average (arithmetic mean)
charge per hour?

$155 $160 $164 $172 $185

(algebra, medium)

Suppose that she charged for x hours, so that she charged a total of 200 +
150( x – 1) = 1550. Thus x – 1 = 9 and x = 10. The average charge per hour,
then, was 1550/10 = $155.

$155 $160 $164 $172 $185

81
Amy’s grade was the 90th percentile of the 80 grades for her class. Of the 100
grades from another class, 19 were higher than Amy’s, and the rest were lower.
If no other grade was the same as Amy’s grade, then Amy’s grade was what
percentile of the grades of the two classes combined?

72nd 80th 81st 85th 92nd

(percents, medium)

Amy did as well as or better than 90% of the 80 students in her class and 100-
19=81 students in the other class. Thus she did as well as or better than
72+81=153 of the 180 students. As 153/180 =17/20=85%, Amy’s grade was in
the 85th percentile of the two classes combined. Note that as the two classes
are fairly equal in size, we expect the percentile to be between roughly halfway
between 81 and 90.

72nd 80th 81st 85th 92nd

82
All points (x,y) that lie below the line l , shown above, satisfy which of the
following inequalities?

y < 2x + 3 y < -2x + 3 y < -x +3


y < x/2 +3 y< -x/2 +3

(coordinate geometry, medium)

Note that the line has a slope of (0-3)/(6-0)= -1/2 and a y intercept of 3, so the
equation of the line is y= -1/2x + 3. All points below the line are given by the
inequality y < -x/2 + 3.

y < 2x + 3 y < -2x + 3 y < -x +3


y < x/2 +3 y< -x/2 +3

83
A lighthouse blinks regularly 5 times a minute. A neighboring lighthouse blinks
regularly 4 times a minute. If they blink simultaneously, after how many
seconds will they blink together again?

20 24 30 60 300

(factors and multiples, medium)

The first lighthouse blinks every 12 seconds, whereas the neighbouring


lighthouse does so every 15 seconds. We see that the least common multiple of
12 and 15 is 60, so they will not blink together until 60 seconds have passed.
You could also reason that, as the greatest common divisor of 4 and 5 is 1,
their blinking will not coincide at any time less than one minute.

20 24 30 60 300

84
A list of measurements in increasing order is 4,5,6,8,10, and x. If the median of
these measurements is 6/7 times their arithmetic mean, what is the value of x?

16 15 14 13 12

(statistics, medium)

As there are 6 measurements, the median is the average of the 3rd and 4th
elements when the elements are listed in increasing order. Thus the median is
7. The arithmetic mean of these measurements is (33+x)/6.
Thus 7=6/7 of (33+x)/6 and so x=16.

16 15 14 13 12

85
Al, Pablo, and Marsha shared the driving on a 1,500-mile trip. Which of the
three drove the greatest distance on the trip?

(1) Al drove 1 hour longer than Pablo but at an average rate of 5 miles per
hour lower than Pablo.

(2) Marsha drove 9 hours and averaged 50 miles per hour.

(movement, hard)

(T) Marsha drove 450 miles, so Al and Pablo drove the remaining 1050
miles. Either Al or Pablo drove the greatest distance, as they drove an
average of 525 miles each If they drove quickly, the additional hour that
Al drove, even at a slightly lower speed, would make him the one who
drove the greatest distance. However, it they drove slowly, the faster
speed at which Pablo drove would offset the shorter duration behind the
wheel. NOT SUFF

Suppose Pablo drove for t hours at a rate of v miles per hour


Then Al drove for t + 1 hours at a rate of v – 5 miles per hour.
Pablo drove farther than Al if vt > (t + 1)(v – 5) = vt + v – 5t – 5, i.e. if v – 5t
<5

86
A manufacturer conducted a survey to determine how many people buy
products P and Q. What fraction of the people surveyed said that they buy
neither product P nor product Q?

(1) 1/3 of the people surveyed said that they buy product P but not product
Q.
(2) 1/2 of the people surveyed said that they buy product Q.

(sets, medium)

If one can find the fraction f of people surveyed that said that they buy at least
one of the two products, 1 – f is the fraction that said that they buy neither
product. f = p + q + b, where p is the fraction that said that they bought P
only, q is the fraction that said that they bought q only, and b is the fraction
that said that they bought P and Q.

(1) p = 1/3. No information is given about q or b. INSUFF


(2) q + b = 1/2. No information is given about p. INSUFF
(T) p + q + b = 1/3 + 1/2. SUFF

87
A manufacturer produced x percent more video cameras in 1994 than in 1993
and y percent more video cameras in 1995 than in 1994. If the manufacturer
produced 1,000 video cameras in 1993, how many video cameras did the
manufacturer produce in 1995?

(1) xy = 20
(2) x + y + xy/100 = 9.2

(percents, algebra, hard)

We know that the manufacturer produced x percent more video cameras in


1994 than it did in 1993, so the number of cameras produced in 1994 is
1000(1+x/100). As the number of cameras produced in 1995 is y% greater
than the number produced in 1994, the number produced in 1995 is
1000(1+x/100)(1+y/100) =1000 (1 + (x+y)/100 + xy/10000)=1000 +
10(x+y)+ xy/10.

(1) Knowing that xy=20 does not allow us to find x+y. Therefore the
number produced in 1995 cannot be determined. NOT SUFF

(2) If x+y +xy/100 = 9.2, 10(x+y)+xy/10 =92, and the number produced in
1995 can be found. SUFF

88
An athlete runs R miles in H hours, then rides a bike Q miles in the same
number of hours. Which of the following represents the average speed, in miles
per hour, for these 2 activities combined?

(R-Q)/H (R-Q)/2H 2(R+Q)/H 2(R+Q)/2H (R+Q)/2H

(movement, easy)

The average speed, in miles per hour, for these 2 activities combined, is the
total distance covered in miles, R + Q , divided by the total number of hours,
2H.

(R-Q)/H (R-Q)/2H 2(R+Q)/H 2(R+Q)/2H (R+Q)/2H

89
An equilateral triangle ABC is inscribed in square ADEF, forming three right
triangles: ADB, ACF and BEC. What is the ratio of the area of triangle BEC to
that of triangle ADB?

4/3 3 2 5/2 5

(geometry, hard)

Since |AB|=|BC|, x2 + 2xy + y2 +x2 = 2y2, so 2x2 +2xy =2x(x+y)= y2


The ratio of the area of triangle BEC to that of triangle ADB is y2/x(x+y) =2

4/3 3 2 5/2 5

90
An integer greater than 1 that is not a prime number is called composite. If
two-digit integer n is greater than 20, is n composite?

(1) The tens digit of n is a factor of the units digit of n.


(2) The tens digit of n is 2.

(factors and multiples, hard)

(1) Suppose the tens digit of n is x.The the units digit of n is kx, where k is a
positive integer. Therefore n = 10x + kx = x (10 + k). Since x is at least
two, x , an integer greater than 1 but less than n, is a divisor of n. Thus
n is a not a prime number. SUFF
(2) n may or may not be a prime number: n could be 22 ( a composite
number) or 23 ( a prime number). NOT SUFF

91
An investment of d dollars at k percent simple annual interest yields $600
interest over a 2-year period. In terms of d, what dollar amount invested at the
same rate will yield $2400 interest over a 3-year period?

2d / 3 3d /4 4d /3 3d /2 8d /3

(interest, ratios, medium)

Remember that at r% simple interest over n year, an investment of I dollars


yields I x r x n dollars.

Let x be the amount invested over a 3-year period.

3xk=2400 But 2dk=600


Thus 3xk=4(2dk) and x=8d/3

Alternatively, to earn 4 times as much interest in 3/2 of the original time, one
needs 4 × 2/3 = 8/3 times the original principal!

2d / 3 3d /4 4d /3 3d /2 8d /3

92
An investor opened a money market account with a single deposit of $6,000 on
December 31, 2001. The interest earned on the account was calculated and
reinvested quarterly. The compounded interest reported for the first three
quarters was $125 , $130 , and $145, respectively. If the investor made no
deposits or withdrawals during the year, approximately what annual rate of
interest must the account earn for the fourth quarter in order for the total
interest earned on the account for the year to be 10 percent of the inital
deposit?

3.1% 9.3% 10.0% 10.5% 12.5%

(interest, hard)

10 percent of the inital deposit is $600, of which $400 was earned in the first
three quarters, and the balance at the end of the 3rd quarter is $6,400. Thus
$200 must be earned in the 4th quarter, so the annual rate of interest r that the
account must earn for the 4th quarter is given by the following:

6400 (1+r/4)=6600, so r = 4 (6600/6400 – 1) = 4 (33/32 – 1) > 4(0.03) =0.12

3.1% 9.3% 10.0% 10.5% 12.5%


93
An investor purchased 20 shares of a certain stock at a price of $45.75 per
share. Late this investor purchased 30 more shares at a price of $46.25 per
share. What is the average (arithmetic mean) price per share that this investor
paid for the 50 shares?

$45.80 $45.95 $46.00 $46.05 $46.20

(ratios, medium)

The total sum of money paid is 20(45.75) + 30(46.25) = 50(45.75) + 30(0.50)


Dividing this sum by 50, we get the average price per share: 45.75 + 0.3 =
46.05. This is just more than the average of the two prices, $46.00.

$45.80 $45.95 $46.00 $46.05 $46.20

94
Ann deposited money into two accounts, A and B. Account A earns 8% simple
interest and B earns 5% simple interest. If there were no other transactions, A
gained how much more interest in the first year than did B in the first year?

(1) Ann invested $200 more in B than she did in A.


(2) The total amount of interest gained by the two accounts in the first
year was $120.

(interest, hard)

Suppose the amounts deposited are a and b. We are asked for the value of D=
0.08a – 0.05b

(1) b= 200 + a, so D = 0.03a – 10 NOT SUFF


(2) 0.08a + 0.05b = 120, so D= 120 – 0.1b NOT SUFF
(T) As we have two linear equations involving a and b, we can solve for both
variables and then answer the question. SUFF

95
{ - 10, -6, -5, -4, -2.5, -1, 0, 2.5, 4, 6, 7, 10}

A number is to be selected at random from the set above. What is the


probability that the number selected will be a solution of the equation
(x – 5)(x + 10)(2x – 5) = 0 ?

1/12 1/6 1/4 1/3 1/2

(proabability,medium)

The solutions of the equation above are x= 5, x= - 10, x= 2.5. As two of these
values are elements of the set of numbers above, 2 of the 12 elements are
solutions of the equation. Therefore, the probability that the number selected
will be a solution of the equation (x – 5)(x + 10)(2x – 5) = 0 is 2/12 = 1/6.
1/12 1/6 1/4 1/3 1/2

96
A parent established a college fund for his daughter. Each year the parent
made a contribution to the fund, and each year he increased his contribution by
a constant amount. If he made a contribution of $800 in the first year, by what
amount did the parent increase his contribution to the fund each year?

(1) The parent’s contribution to the fund in the 18th year was $7600.
(2) The parent’s contribution to the fund in the 7th year was twice what it
was in the 3rd year.

(sequences, hard)

The contribution in the nth year is 800 + (n – 1)x , where x is the amount by
which his contribution increases each year. We are asked to find the value of x.

(1) 800 + (18 – 1)x = 7600 SUFF


(2) 800 + (7 – 1)x =2(800 + (3 -1)x) SUFF

97
A point is arbitrarily selected on a line segment, breaking it into two smaller
segments.What is the probability that the bigger segment is at least twice as
long as the smaller?

1/4 1/3 1/2 2/3 3/4

(algebra, hard)

Think of the number line with the segment between 0 and 1, which point x
chosen as cutting point.
The left segment is at least twice as long as the right if x ≥ 2(1-x) i.e. x ≥ 2/3
Thus the probability that the left segment is at least twice as long as the right is
1/3, as is the probability that the right is at least twice as long as the left. So
the required probability is 1/3 + 1/3 =2/3

1/4 1/3 1/2 2/3 3/4

98
A positive integer n is said to be ―prime-saturated‖ if the product of all the
different positive prime factors of n is less than the square root of n. What is
the greatest two-digit prime-saturated integer?

99 98 97 96 95

(factors and multiples, hard)

For a positive integer n slightly less than 100 to be prime saturated, its only
prime factors should be 2 and 3, as if it has larger prime factors the product of
the prime factors will be greater than 10.
96 = 24 × 4 = 3 × 25 is prime-saturated, but none of the other numbers are:
99 is a multiple of 11, 98 is a multiple of 7 and 2, 97 is a prime number, and 95
is a multiple of 19 and 5.

99 98 97 96 95

99
Are at least 10% of the people in Country X who are 65 years old or older
employed?

(1) In Country X, 11% of the population is 65 years old or older.


(2) In Country X, of the population 65 years old or older, 20% of the men
and 10% of the women are employed.

(algebra, hard)

(1) gives us no information about employment. NOT SUFF


(2) tells us that the percentage of all people 65 years old or order that is
employed is between 10% and 20%, and is thus at least 10%. SUFF

100
Are positive integers p and q both greater than n?

(1) p – q is greater than n.


(2) p<q

(inequalities, hard)

(1) p > n + q, so p is greater than n. However, q may or may not be be


greater than n. NOT SUFF
(2) p – q is negative, but nothing is known about n NOT SUFF

(1) and (2): p – q < 0 but p – q > n, so n <0 and since p and q are positive,
we know that p and q are both greater than n. SUFF

101
A rectangular region has a fence along three sides and a wall along the fourth
side. The fenced side opposite the wall is twice the length of each of the other
two fenced sides. If the area of the rectangular region is 128 square feet, what
is the total length of the fence in feet?

4 8 16 32 64
(algebra, easy)

The lengths of the 3 sides are x, x and 2x, for a total of 4x , and the area of the
rectangular region is 2x2 = 128, so x=8 and 4x = 32.

4 8 16 32 64

102
Are x and y both positive?

(1) 2x -2y = 1
(2) x/y > 1

(inequalities, hard)

(1) tells us that x= y + 1/2, so x and y can be both positive or both


negative. Also y could be negative and x positive. NOT SUFF
(2) x/y > 1 tells us that x > y if y>0 and x<y if y<0. So x and y can be both
positive or both negative. NOT SUFF

(T) From (1), we know that x>y. (2) tells us that x > y if and only if
y >0. Thus x and y are both positive. SUFF

103
A saleswoman’s monthly income has two components, a fixed component of
$1000, and a variable component, which is $C for each set of encyclopaedias
that she sells in that month over a sales target of n sets, where n > 1. How
much did she earn in March?

(1) If she had sold three fewer sets than she actually did, her March income
would have been $600 less.
(2) If she had sold 8 sets of encyclopaedias, her income in March would
have been over $4000.

(inequalities, hard)

We see that the saleswoman’s income in a given month on sales for that month
of x sets is (in dollars) 1000 if x<n, and 1000 +C(x-n) if she sells x sets, where
x>n.

(1) 600 is equal to either C, 2C or 3C. If 600=C or 600=2C. her income for
that month was $1600. However, if 600=3C, her income could be $1600
or more. NOT SUFF

(2) Clearly, n<8, so we can say that 1000+C(8-n)>4000


and thus C > 3000/(8-n). We do not know, however, the exact value of
C. Nor do we know how many sets she actually sold. NOT SUFF

(T) From (2), we see that C> 3000/7 > 300, so (1) tells us that
C=600 and her income for the month was $1,600. SUFF

104
A scientist recorded the number of eggs in each of 10 birds’ nests. What was
the standard deviation of the numbers of eggs in the 10 nests?

(1) The average (arithmetic mean) number of eggs for the 10 nests was 4.
(2) Each of the 10 nests contained the same number of eggs.

(statistics, medium)

(1) Remember that standard deviation of a set of numbers is the average


distance between each element. No information is given in (1) about
how far apart the elements are. NOT SUFF
(2) Since each element is equal to the mean number of eggs, the standard
deviation is 0. SUFF

105
A set of 15 different integers has a median of 25 and a range of 25. What is the
largest possible integer that could be in the set?

32 37 40 43 50

(statistics, hard)

In a nutshell, since the range is 25, the greater the value of the smallest integer
in the set, the greater the value of the largest integer in the set.

If we look at the integers in increasing order, x1<x2<x3<…<x15, the median is


x8. The range x15-x1=25. Since x15=25+x1 and we want to maximize x15, we
need to find the maximum value of x1. If {x1, x2, x3,…, x8=25} are consecutive
integers x1=25-8+1=18. Thus the largest possible value for x15 is 25+18=43.

32 37 40 43 50

106
A set of data consists of the following 5 numbers: 0,2,4,6, and 8. Which two
numbers, if added to create a set of 7 numbers, will result in a new standard
deviation that is closest to the standard deviation for the 5 original numbers?

-1 and 9 4 and 4 3 and 5 2 and 6 0 and 8

(statistics, hard)

Note that the terms of the original set are on average 2 units from the
arithemetic mean, 4. Thus the addition of elements that are 2 units from 4 (i.e.
2 and 6) will result in a larger set with a very similar standard deviation.

-1 and 9 4 and 4 3 and 5 2 and 6 0 and 8

107
A state legislature had a total of 96 members. The members who did not vote
on a certain bill consisted of 25 who were absent and 3 who abstained. How
many of those voting voted for the bill?

(1) Exactly 1/3 of the total membership of the legislature voted against the
bill.
(2) The number of legislators who voted for the bill was 8 more than the
total number who were absent or abstained.

We need to know how many of the remaining 96 -28 legislators voted for the
bill. Each of these legislators voted either for the bill or against it.

(1) If 1/3 of the 96 voted against the bill, the number who voted for it is 96
– 28 – 1/3(96) SUFF
(2) The number required is 8 more than 25 + 3 SUFF

108
A store purchased 20 coats that each cost an equal amount and then sold each
of the 20 coats at an equal price. What was the store's gross profit on the 20
coats?

(1) If the selling price per coat had been twice as much, the store's gross
profit on the 20 coats would have been $2,440.
(2) If the selling price per coat had been $2 more, the store's gross profit
on the 20 coats would have been $440.

(algebra, hard)

Suppose that the cost of each coat is c dollars and the selling price for each
coat is s > c. We are asked for the value of 20(s - c).

(1) 20 (2s – c) = 2400, so 2s – c = 200 and – c = 200 – 2s. Thus the store’s
gross profit is 20(200 – s). We cannot answer the question without the
value of s. If fact, if s=200, the gross profit would have been 0!
NOT SUFF

(2) 20( s+2 – c) = 440, so 20(s – c) =440 – 40 = 400 SUFF

109
A researcher computed the mean, the median, and the standard deviation for a
set of performance scores. If 5 were to be added to each score, which of these
statistics would change?

mean only median only standard deviation only


mean and median mean and standard deviation

(statistics, medium)

Remember that mean and median are measurements of the central tendancy of
the score. If 5 were to be added to each score, the sum of the scores would
increase, and thus the mean would increase as well. If 5 were added to each
score, each score will rise, including the one score or two scores used to
compute the median.
The standard deviation, in contrast, is a measurement of the average distance
from each score to the mean of the scores. If 5 were added to each score, the
mean would increase by 5 and thus the distance between any score and the
mean would not be changed. Therefore, the standard deviation would not
change.

mean only median only standard deviation only


mean and median mean and standard deviation

110
A school administrator will assign each student in a group of n students to one
of m classrooms. If 3<m<13<n, is it possible to assign each of the n students
to one of the m classrooms so that each classroom has the same number of
students assigned to it?

(1) It is possible to assign each of 3n students to one of m classrooms so


that each classroom has the same number of students assigned to it.
(2) It is possible to assign each of 13n students to one of m classrooms so
that each classroom has the same number of students assigned to it.

(factors and multiples, hard)

It is possible to do so if and only if m is a factor of n. In other words is n/m an


integer?

(1) 3n is a multiple of m. In other words, 3n/m is an integer. We do not


know, however, whether n/m is an integer. For example, if n=14 and m=6,
3n/m is an integer, but n/m is not an integer. On the other hand, if n=18 and
m=6, both 3n/m and n/m are integers. NOT SUFF

(2) 13n is a multiple of m. In other words, 13n/m is an integer. Thus 13n is


a multiple of m. As m is an integer between 3 and 13, 13 is not a multiple of m.
Since 13 is a prime number, n must be a multiple of of m. SUFF

Another way to think about this: suppose that n/m is not an integer. As
3<m<13 and 13 is a prime number, 13n/m is not an integer, either, a
conclusion that contradicts (2).

111
A straight line in the xy-plane has a slope of 2 and a y-intercept of 2. On this
line, what is the x-coordinate of the point whose y-coordinate is 500 ?

249 498 676 823 1,002

(coordinate geometry, medium)

Remember that the equation of a line with slope m and y-intercept b is y=mx+
b. Thus the equation of the line is question is y= 2x + 2. If y=500, x = 249

249 498 676 823 1,002

112
A student worked for 20 days. For each of the amounts shown in the first row
of the table, the second row gives the number of days that the student earned
that amount. What is the median amount of money that the student earned per
day for the 20 days?

96 84 80 70 48

(statistics, medium)

As 20 is an even number, to find the median amount of money, we must find


the mean of 10th and 11th largest of the 20 amounts shown. Each of these is
$84, so that is the median amount of money that the student earned.

96 84 80 70 48

113
A store purchased a Brand C computer for the same amount that it paid for a
Brand D computer and then sold them both at higher prices. The store’s gross
profit on the Brand C computer was what percent greater than its gross profit
on the Brand D computer?

(1) The price at which the store sold the Brand C computer was 15 percent
greater than the price at which the store sold the Brand D computer.
(2) The store’s gross profit on the Brand D computer was $300.

(percents, algebra, hard)

Let p be the price paid for each computer and c and d be the selling price of
the Brand C and Brand D computers respectively. We are asked for the value of
c p
100(  1) . In other words can we find the ratio c – p : d – p ?
dp

(1) c=1.15d. NOT SUFF. A couple of examples: if p=90 and d=100, c=115
and (c - p)/(d - p)=25/10=2.5. However, if p=10 and d=100, c=115 and
(c - p)/(d - p)=95/80.

(2) d-p=300. NOT SUFF, as no information is given about c-p.

(T) Given that d=p+300 and c=1.15d


c-p=1.15p + 1.15(300) - p =0.15p +1.15(300).
Clearly, the numerator depends on the value of p, whereas the
denominator is 300, so the ratio will depend on the value of p.
NOT SUFF

114
At a certain bakery, each roll costs r cents and each doughnut costs d cents. if
Alfredo bought rolls and doughnuts at the bakery, how many cents did he pay
for each roll?

(1) Alfredo paid $5 for 8 rolls and 6 doughnuts.


(2) Alfredo would have paid $10 if he had bought 16 rolls and 12
doughnuts.

(algebra, medium)

We are asked for the value of r.

(1) 8r + 6d = 500 4r + 3d = 250 r = (250 -3d)/ 4 NOT SUFF


(2) Equivalent to (1) NOT SUFF
(T) NOT SUFF

115
At a certain college there are twice as many English majors as history majors
and three times as many English majors as mathematics majors. What is the
ratio of the number of history majors to the number of mathematics majors?

6 to 1 3 to 2 2 to 3 1 to 5 1 to 6

(ratios, medium)

We are given that e = 2h=3m, so h/m= 3/2

6 to 1 3 to 2 2 to 3 1 to 5 1 to 6

116
At a certain company, each employee has a salary grade s that is at least 1 and
at most 5. Each employee receives an hour wage, p , in dollars, determined by
the formula p= 9.50 + 0.25 ( s – 1) . An employee with a salary grade of 5
receives how many more dollars per hour than an employee with a salary grade
of 1 ?

$0.50 $1.00 $1.25 $1.50 $1.75

Note that there is a linear relationship between p and s, and if s and p were
plotted and the x and y axes of the xy-coordinate plane, the slope would be
$0.25. Thus as 5 is 4 higher than 1, p(5) is $0.25(4)= $1.00 higher than p(1).

$0.50 $1.00 $1.25 $1.50 $1.75

117
At a certain food stand, the price of each apple is 40¢ and the price of each
orange is 60¢. Mary selects a total of 10 apples and oranges from the food
stand, and the average (arithmetic mean) price of the 10 pieces of fruit is 56¢.
How many oranges must Mary put back so that the average price of the pieces
of fruit that she keeps is 52¢?

1 2 3 4 5

(algebra, hard)

If she puts back x oranges, the price of the 10 – x pieces of fruit she keeps is
560 – 60x cents. If the average price of the fruit she keeps is 52¢,

560  60 x
 52  560  60 x  520  52 x  40  8 x  x  5
10  x

1 2 3 4 5
118
At a certain refreshment stand, all hot dogs have the same price and all sodas
have the same price. What is the total price of 3 hot dogs and 2 sodas at the
refreshment stand?

(1) The total price of 5 sodas at the stand is less than the total price of 2 hot
dogs.
(2) The total price of 9 hot dogs and 6 sodas at the stand is $21.

(algebra, medium)

We need to find the value of 3h + 2s , where h is the price of each hot dog and
s is the price of each soda.

(1) 5s < 2h NOT SUFF


(2) 9h + 6s = 21. Since 9h + 6s = 3(3h + 2s), we can find the value of 3h +
2s. SUFF

119
At a certain store, each notepad costs $x and each marker $y. $10 is enough to
buy 5 notepads and 3 markers. Is $10 enough to buy 4 notepads and 4
markers?

(1) Each notepad costs less than $1.


(2) $10 is enough to buy 11 notepads.

(inequalities, hard)

We know that $10 is enough to buy 5 notepads and 3 markers. We know that
$10 will be enough for 4 notepads and 4 markers if the price of a marker is no
higher than that of a notepad. If this is not the case, $10 may or may not be
enough. Alternatively, one could rephrase the question as follows: Is the price
of one marker plus the price of one notepad no more than $2.50?

(T) Nothing is said about the price of markers. If the price of a marker is $2
and that of a notepad is $0.90, $10 will not be enough. If the price of a marker
is $1 instead, $10 will be enough. NOT SUFF

120
At a certain supplier, a machine of type A costs $20,000 and a machine of type
B costs $50,000. Each machine can be purchased by making a 20 percent down
payment and repaying the remainder of the cost and the finance charges over a
period of time. If the finance charges are equal to 40 percent of the remainder
of the cost, how much less would 2 machines of type A cost than 1 machine of
type B under this arrangement?

$10,000 $11,200 $12,000 $12,800 $13,200

(percents, medium)

Note that under this arrangement, an article costing p dollars requires a down
payment of 0.2p and subsequent payments totaling 1.4(0.8p) = 1.12p, for a
total payment of 1.32p. Since 2 A´s cost $10,000 less than 1 B, under this
arrangement, they could cost $13,200 less.

$10,000 $11,200 $12,000 $12,800 $13,200


121
At a certain university, the ratio of teachers to students for every course is
always greater than 3:80. At this university, what is the maximum number of
students possible in a course that has 5 teachers?

130 131 132 133 134

(inequalities, medium)

If there are t teachers and s students, we are told that t/s > 3/80, so s< 80t/3.
If t=5, s<400/3=133 1/3. Thus there are at most 133 students.

130 131 132 133 134

122
At a constant rate of flow, it takes 20 minutes to fill a swimming pool if a large
hose is used and 30 minutes if a small hose is used. At these constant rates,
how many minutes will it take to fill the pool when both hoses are used
simultaneously?

10 12 15 25 50

(combined work, medium)

In one hour, the large and small hose would fill 3 and 2 pools
respectively, for a total of 5 pools. Thus in 1/5 of an hour, i.e. 12 minutes, one
pool could be filled by the two hoses.

Alternatively, in one minute, 1/20 + 1/30 = 5/60=1/12 of the pool is


filled, so in 12 minutes, one pool could be filled.

10 12 15 25 50

123
At a dinner party, 5 people are to be seated around a circular table. Two
seating arrangements are considered different only when the positions of the
people are different relative to one another. What is the total number of
different seating arrangements for the group?

5 10 24 36 120

(combinatronics, hard)

There is a one-to-one correspondence between each different seating


arrangement and a seating arrangement in which person A is seated in position
*. Thus we simply need to count the number of ways of arranging the four
remaining people in the four remaining seats, which is 4!=24.

Alternatively, one could suppose for a moment that the seats were
distinguishable. In that case, there would be 5! ways to seat the 5 people.
However, we can rotate the table 4 times and we get a different arrangement
in which the positions of the people are the same relative
to one another. Thus 5 absolute arrangements correspond to the same relative
arrangement. Therefore, to find the number of different relative arrangements,
we need to divide 5! by 5: 5!/5= 4!= 24.

5 10 24 36 120

124
At a garage sale, all of the prices of the items sold were different. If the price of
a radio sold at the garage sale was both the 15th highest price and the 20th
lowest price among the prices of the items sold, how many items were sold at
the garage sale?

33 34 35 36 37

(algebra, medium)

If the price of the radio was the 15th highest and the 20th lowest, there were 14
items whose selling price was higher and 19 items whose selling price was
lower. Therefore, 14 + 19 + 1 = 34 items were sold at the garage sale.

33 34 35 36 37

125
At a special sale, all of the prices of the items were different. If the price of a
radio sold at the sale was both the 15th highest and the 20th lowest price among
the prices of the items sold, how many items were sold at the sale?

33 34 35 36 37

(algebra, medium)

14 items have a higher selling price than the radio, and 19 have a lower selling
price. Therefore, 14 + 19 +1= 34 items were sold, including the radio

33 34 35 36 37

126
At a speed of 50 miles per hour, a certain car uses 1 gallon of gasoline every 30
miles. If the car starts with a full 12 gallon tank of gasoline and travels for 5
hours at 50 miles per hour, the amount of gasoline used would be what fraction
of a full tank?

3/25 11/36 7/12 2/3 25/36

(ratios, medium)

Such a car would travel 250 miles and thus use 250/30 = 25/3 gallons. Thus
the required fraction is (25/3)/12 = 25/36.

3/25 11/36 7/12 2/3 25/36

127
At a two-candidate election for mayor, ¾ of the registered voters cast ballots.
How many registered voters cast ballots for the winning candidate?

(1) 25,000 registered voters did not cast ballots in the election.
(2) Of the registered voters who cast ballots, 55% cast ballots for the
winning candidate.

(percents, medium)

(T) From (1) we can find the number of registered voters v: v/4= 25,000
From (2), we know that the number of registered voters who voted for
the winning candidate is 0.55 (3/4) v SUFF

128
At a two day seminar, 90 percent of those registered attended the seminar on
the first day. What percent of those registered did not attend the seminar on
either day?

(1) A total of 1000 people registered for the two day seminar.
(2) Of those registered, 80 percent attended the seminar on the second day.

(sets, medium)

(1) Clearly insufficient. NOT SUFF


(2) It could be that the 10% of those registered who did not attended on the
first day attended on the second. In this case, 0% did not attend on either day.
Alternatively, perhaps the 10% who failed to attend on the first day also failed
to attend on the second day, in which case 10% did not attend on either day.
NOT SUFF
(T) That 1000 people registered does not preclude either of the two cases
identified in (2) NOT SUFF

129
At Company X senior sales representatives visit the home office once every 30
days, and junior sales representatives visit the home office once every 20 days.
The number of visits that a junior sales representative makes in a 2-year period
is approximately what percent greater than the number of visits that a senior
representative makes in the same period?

10% 25% 33% 50% 67%

(ratios, medium)

The number of visits in a given number of years is approximately inversely


proportional to the number of number of days between consecutive visits. Thus
the junior makes 3/2 as many visits as the senior member (i.e. 50% more)

10% 25% 33% 50% 67%

130
A ten liter mixture of cranberry juice and water contains juice and water in the
ratio of 3 : 2. Five liters of the mixture are removed and replaced with pure
juice and the operation is repeated once more. At the end of the two removals
and replacements, what is the ratio of juice to water in the resulting mixture?

5:3 6:4 8:2 17 : 3 9:1

(ratios, medium)

We start 6 liters of juice and 4 liters of water. The volume is halved, leaving us
with 3 liters of juice and 2 liters of water. 5 liters of juice are added, leaving us
with 8 liters of juice and 2 liters of water. The volume is halved again, leaving
us with 4 liters of juice and 1 liter of water. 5 liters of juice are added, leaving
us with 9 liters of juice and 1 liter of water.

5:3 6:4 8:2 17 : 3 9:1

131
A thin piece of wire 40 meters long is cut into two pieces. One piece is used to
form a circle with radius r, and the other is used to form a square. No wire is
left over. Which of the following represents the total area, in square meters, of
the circular and the square regions in terms of r?

 r2  r2 +10  r2 +(1/4)  2r2


 r2+(40-2  r) 2  r2+(10-0.5  r) 2
(geometry, hard)

If a circle with radius r is made, 2 r meters of wire are needed,


leaving 40 - 2 r of wire to form the square. Each side of the square will need
1/4 of this remaining wire, so each side of the square will have a length in
meters of 10 - r . Thus the combined areas of the circle and the square, is
2
r
r 2  (10  )2 .
2

 r2  r2 +10  r2 +(1/4)  2r2


 r2+(40-2  r) 2  r2+(10-0.5  r) 2

132
At least 100 students at a certain high school study Japanese. If 4 percent of
the students at the school who study French also study Japanese, do more
students at the school study French than Japanese?

(1) 16 students at the school study both French and Japanese


(2) 10% of the students at the school who study Japanese also study
French.

(sets, hard)

(1) Let f be the number of students who study French. The 16 students who
study French and Japanese constitute 4% of the students who study
French. Thus 16=0.04f and f=16 x 25=400. Thus 400 students study
French, but we do not know how this number compares with the number
of students who study Japanese, a number that is at least 100.
NOT SUFF
(2) Suppose that x students study French and Japanese and j students study
Japanese. x=0.1j. But x=0.04f, so 0.1j=0.04f. As we can find the ratio
between j and f, we can answer the question. SUFF

133
A total of 100 customers purchased books at a certain bookstore last week. If
these customers purchased a total of 200 books, how many of the customers
purchased only 1 book each?

(1) None of the customers purchased more than 3 books.


(2) 20 of the customers purchased only 2 books each.

(algebra, medium)

(1) Suppose x students bought 1 book and y both 2 books.


Then 100 – x – y bought 3 books , and x + 2y + 3(100 – x – y) = 200
Therefore 2x + y = 100 and x= (100 – y)/2. As the value of x depends
on the value of y, more information is needed. NOT SUFF
(2) We are given that y=20, but no information is given as to whether
customers bought more than 3 books. NOT SUFF
(T) SUFF

134
At the bakery, Lew spent a total of $6.00 for one kind of cupcake and one kind
of doughnut. How many doughnuts did he buy?

(1) The price of 2 doughnuts was $0.10 less than the price of 3 cupcakes.
(2) The average price of 1 doughnut and 1 cupcake was $0.35.

(algebra, hard)

(1) Without information about the actual prices nor the number of cupcakes
Lew bought, the number of doughnuts cannot be found. NOT SUFF
(2) We know that the sum of the prices of one cupcake and one doughnut is
70 cents, but no information is given about the prices and quantities.
NOT SUFF
(T) The the price of a doughnut is d, that of a cupcake is 70 – d (from (2))
and thus 2d = 3(70 – d) – 10 and thus d=40. The prices of one
doughnut and one cupcake are 40 and 30 cents respectively. If Lew
bought x doughnuts and y cupcakes, 40x + 30y = 600 (i.e 4x + 3y = 60)
x= (60-3y)/4 , where x and y are positive integers. Thus y is a multiple
of 4 such that 3y < 60 (i.e y < 20). If y =4 , x=12. However, if y=16,
x=3. NOT SUFF

135
Amount Charged Percent Bonus
$.01 to $1000 0.5
$1000.01 to $3000 1.0
$3000.01 to $6000 1.5
$6000.01 to $10000 2.0

At the end of each year, a certain credit card company pays a bonus to each of
its credit card holders based on the amount up to $10,000 that the cardholder
charged throughout the year. A 0.5% bonus is paid on the first $1000 charged ,
a 1.0% bonus is paid on the next $2000, and so on, as indicated in the table
above. What is the maximum annual bonus, in dollars?

100 150 200 250 500

(percents, hard)

The maximum annual bonus corresponds to a card holder who charges


$10,000, who receives 1/2 % of $1000, 1% of $2000, 1.5% of $3000 and 2%
of $4000= 5 + 20 + 45 + 80 dollars.

100 150 200 250 500

136
At the end of the first quarter, the share price of a certain mutual fund was 20
percent higher than it was the beginning of the year. At the end of the second
quarter, the share price was 50 percent higher than it was at the beginning of
the year. What was the percent increase in the share price from the end of the
first quarter to the end of the second quarter ?

(percents, medium)

Suppose the share price at the beginning of the year was x. It was 1.2x at the
end of the first quarter and 1.5x at the end of the quarter. The percent increase
in the share price from the end of the first quarter to the end of the second
quarter was 100% x (1.5x – 1.2x)/1.2x = 25%

137
A used-car dealer sold one car at a profit of 25% of the dealer’s purchase price
for that car and sold another car at a loss of 20% of the dealer’s purchase price
for that car. If the dealer sold each car for $20,000, the dealer’s total profit or
loss. in dollars, for these two transactions was

A profit of $1,000 A profit of $2,000 A loss of $1,000


A loss of $2,000 A loss of $3,334

(percents, hard)

For the first car, $20,000 is 25% greater than the purchase price p, so in
thousands of dollars 5p/4 = 20 and p=16. For the second carr, $20,000 is 20%
less than the purchase price q, so in thousands of dollars 4q/5 = 20 andq=25.
p + q is 41, $1,000 greater than the sales revenue.

A profit of $1,000 A profit of $2,000 A loss of $1,000


A loss of $2,000 A loss of $3,334

138
A vending machine is designed to dispense 8 ounces of coffee into a cup. After
a test that recorded the number of ounces of coffee in each of 1,000 cups
dispensed by the vending machine, the 12 listed amounts, in ounces, were
selected from the data: 7.51, 8.22, 7.86, 8.36, 8.09, 7.83, 8.30, 8.01, 7.73,
8.25, 7.96 and 8.53 If the 1,000 recorded amounts have a mean of 8.1 ounces
and a standard deviation of 0.3 ounce, how many of the 12 listed amounts are
within 1.5 standard deviations of the mean?

4 5 6 9 11

(statistics, hard)

In other words, how many of the amounts a satisfy the following inequality:
8.1 – 1.5(0.3) ≤ a ≤ 8.1 + 1.5(0.3) i.e. 7.65 ≤ a ≤ 8.55. All but one of the 12
measurements do.

4 5 6 9 11

139
A total of $60,000 was invested for one year. Part of this amount earned simple
annual interest at the rate of x percent per year, and the rest earned simple
annual interest at the rate of y percent per year. If the total interest earned by
the $60,000 for that year was $4,080, what is the value of x?

(1) x = 3/4y
(2) The ratio of the amount that earned interest at the rate of x percent per
year to the amount that earned interest at the rate of y percent per year
was 3 to 2.

(interest, hard)

Suppose a dollars was invested at x percent per year.


Then 60000 – a was invested at y percent per year and we get
ax/100 + (60,000 –a)y/100 = 4080.

(1) Leaves us with a linear equation with two variables, x and a.


NOT SUFF
(2) Gives us the value of a : 2a = 3(60,000 – a), but we are still left with a
linear equation in two variables, x and y. NOT SUFF
(T) Leaves us with a linear equation involving one variable, x. SUFF

140
A toy store’s gross profit on a computer game was 10 percent of the cost of the
game. If the store increased the selling price of the game from $44 to $46 and
the cost of the game remained the same, then the store’s gross profit on the
game after the price increase was what percent of the cost of the game?

10.5% 11% 12.5% 13% 15%

(percents, medium)

Suppose the cost of the game was c dollars. Then 1.1c = 44 and c = 40
Since 46/40 = 1.15, the store’s gross profit on the game after the price increase
was what 15% of the cost of the game.

10.5% 11% 12.5% 13% 15%

141
The wire that weighs 24 kilograms is cut into two pieces so that one of the
pieces weighs 16 kilograms and is 34 meters long. If the weight of each piece is
proportional to its length, how many meters long is the other piece of wire ?

8 11 13 17 20

(ratios, medium)

The weight of the other piece of wire is 8 kilograms, half that of the first piece.
Thus it will be half as long: 17 meters long.

8 11 13 17 20

142
Before being simplified, the instructions for computing income tax in Country R
were to add 2% of one's annual income to the average (arithmetic mean) of
100 units of Country R's currency and 1% of one's annual income. Which of the
following represents the simplified formula for computing the income tax, in
Country R's currency, for a person in that country whose annual income is I?

50 + (I/200) 50 + (3I/100) 50 + (I/40)


100 + (I/50) 100 + (3I/100)

(percents, medium)

Income tax due on annual income of I is 0.02I + (0.01I+100)/2=


I/50 + I/200 + 50= I/40 + 50

50 + (I/200) 50 + (3I/100) 50 + (I/40)


100 + (I/50) 100 + (3I/100)

143
Between 1990 and 2000, the number of Americans aged 45 to 64 are projected
to increase from 47 million to 61 million, while the number of Americans aged
18 to 24 was projected to decrease from 26 million to 25 million. According to
this projection, which of he following is closest to the ratio of the percent
increase in the 45 to 64 age group to the percent decrease in the 18 to 24 age
group?

5 to 1 6 to 1 8 to 1 10 to 1 14 to 1

(percents, medium)

61 milllion is 14 million more than 47 million, a percent increase of 1400/47 %


25 million is 1 million less than 26 million, a percent decrease of 100/26 %

Thus the required ratio is 1400/47 to 100/26 = (1400(26))/47(100) = 14(26) to


47 i.e. approximately 8 to 1.

5 to 1 6 to 1 8 to 1 10 to 1 14 to 1

144
Bobby bought two shares of stock, which he sold for $96 each. If he had a
profit of 20% on the sale of one of the shares but a loss of 20% on the sale of
the other, then on the sale of both shares combined Bobby had

a profit of $10 a profit of $8 a loss of $8 a loss of $10


neither a profit nor a loss

(percents, hard)

Suppose p1 and p2 are the purchase prices of the two shares. The profit on the
first share, 96- p1 , is 20% of p1 and the loss on the second, p2-96 , is 20% of
p2 . Thus 1.2p1 =96 and p1=80. Also 0.8p2=96 and p2=120. Thus p1 +p2= $200
was paid for the shares that were sold for 2(96)=$192. Thus Bobby had a loss
of $8.

a profit of $10 a profit of $8 a loss of $8 a loss of $10


neither a profit nor a loss

145
Box W and Box V each contain several blue sticks and red sticks, and all of the
red sticks have the same length. The length of each red stick is 18 inches less
than the average length of the sticks in Box W and 6 inches greater than the
average length of the sticks in Box V. What is the average (arithmetic mean)
length, in inches, of the sticks in Box W minus the average length, in inches, of
the sticks in Box V?

3 6 12 18 24

(algebra, hard)

Suppose that the average lengths of the sticks in Box W and Box V are w and v
respectively, and that the length of each red stick is r.

r = w – 18 = v + 6  w – v = 24

3 6 12 18 24

146
Can a certain rectangular sheet of glass be positioned on a rectangular tabletop
so that it covers the entire tabletop and its edges are parallel to the edges of
the tabletop?

(1) The tabletop is 36 inches wide by 60 inches long.


(2) The area of one side of the sheet of glass is 2400 square inches.

(geometry, hard)

Neither statement is sufficient by itself, as information about the dimensions of


both objects is needed.

(T) As the area of the glass is greater than that of the tabletop, the glass
may be able to cover the tabletop. For example, if the sheet of glass is
36 inches wide by 60 inches long, it can cover the tabletop. On the other
hand, if the glass is long and narrow (e.g. 20 x 120), it will not cover the
tabletop.

NOT SUFF

147
Circle C and line k lie in the xy-plane. If circle C is centered at the origin and
has radius 1, does line k intersect circle C?

(1) The x intercept of line k is greater than 1.


(2) The slope of the line k is -1/10.

(coordinate geometry, hard)

(1) is not sufficient, as we have no information about the slope of the line,
nor do we know how much greater than 1 is the x intercept of line k.
(2) If the slope of line k is -1/10, line k may or may not intersect circle C.

(T) A line with a slope of -1/10 that has a y-intercept of -1 has an equation
of y=- x/10 - 1, which has an x-intercept of 10. Such a Line with a x-
intercept that is substantially greater than 10 will not intersect C, but one
with an x-intercept between 1 and 10 will intersect C. NOT SUFF

148
Circular gears P and Q start rotating at the same time at constant speeds. Gear
P makes 10 revolutions per minute, and gear Q makes 40 revolutions per
minute. How many seconds after the gears start rotating will gear Q have made
exactly 6 more revolutions than gear P?

6 8 10 12 15

(movement, ratios, medium)

In one minute, gear Q makes 30 revolutions more than does gear P. Thus gear
Q will make exactly 6= 30/5 revolutions more than will P in 1/5 of a minute
=60/5 seconds.

6 8 10 12 15

149
C(m,n) = m! / [(m-n)! n!] for nonnegative integers m and n, m ≥ n.
If C(5,3) = C(5,x) and x ≠ 3, what is the value of x?

0 1 2 4 5

(functions and sequences, medium)

We are told that C (5 , x ) = C ( 5 , 3 ) = (5 4 × 3) / 3! = 10


C(5,0) = 1, C(5,1) = 5 C(5,2) = (5×4) / 2! = 10
Remember that C(n,k) = C(n,n- k) for all integers k such that 0 ≤ k ≤ n.

0 1 2 4 5

150
Company C has a machine that, working alone at its constant rate, processes
100 units of a certain product in 5 hours. If Company C plans to buy a machine
that will process this product at a certain rate, and if the two machines, working
together at their constant rates, are to process 100 units of this unit in 2 hours,
what should be the constant rate, in units per hour, of the new machine?

50 45 30 25 20

(combined work, medium)

We know that the first machine processes 100 units per 5 hours, i.e. 20 units
per hour. Combined, the two machines are to processes 100 units in 2 hours,
i.e. 50 units per hour. If the second machine processes x units of hour, the two
machines combined process 20 + x units per hour. As 20 + x = 50, x= 30.

50 45 30 25 20

151
Company K has an annual budget for a certain project, and 1/5 of this budget
was spent during the first quarter of the year. If 1/8 of the remainder of the
budget was spent during the second quarter, what fraction of the budget was
left at the end of the second quarter?

7/10 31/40 27/40 3/10 9/40

(ratios, medium)

4/5 of the budget remained after the first quarter, of which 7/8 was not spent
during the second quarter. 4/5 × 7/8 = 28/40 = 7/10

7/10 31/40 27/40 3/10 9/40

152
Company P had 15 percent more employees in December than it had in
January. If Company P had 460 employees in December, how many employees
did it have in January?

391 400 410 423 445

(percents, medium)

If the company had j employees in January, it had 1.15j = 23j/20 employees in


December. Since 23j/20 = 460, j= 400.

391 400 410 423 445

153
Company S produces two kinds of stereos: basic and deluxe. Of the stereos
produced by Company S last month, 2/3 were basic and the rest were deluxe.
If it takes 7/5 as many hours to produce a deluxe stereo as it does to produce a
basic stereo, then the number of hours it took to produce the deluxe stereos
last month was what fraction of the total number of hours it took to produce all
the stereos?

7/17 14/31 7/15 17/35 1/2

(percents, hard)

The chart above is helpful.

7/17 14/31 7/15 17/35 1/2


154
Connie paid a sales tax of 8% on her purchase. If the sales tax had been only
5%, she would have paid $12 less in sales tax on her purchase. What was the
total amount that Connie paid for her purchase including sales tax?

$368 $380 $400 $420 $432

(percents, medium)

Suppose the price before tax was p. We want to find 1.08p


If the sales tax had been only 5%, she would have paid $12 less,
so 0.08p - 0.05p = 12. 0.03p=12.
Therefore 3p=1200 and p=400, and 1.08p= 432

$368 $380 $400 $420 $432

155
Did it take Pei more than 2 hours to walk a distance of 10 miles along a certain
trail? (1 mile= 1.6 km)

(1) Pei walked this distance at an average rate of less than 6.4 km per hour.
(2) On average, it took Pei more than 9 minutes per kilometer to walk this
distance.

(movement, medium)

We can simplify the question in terms of speed. Was Pei’s speed less than 5
miles (i.e. 8 kilometers) per hour? Note that 5 miles = 1.6(5)=8 kilometers.

(1) SUFFICIENT
(2) Her speed in kilometers per hour is less than 60/9 < 8 SUFFICIENT

156
Did one of the 3 members of a certain team sell at least 2 raffle tickets
yesterday?

(1) The 3 members sold a total of 6 raffle tickets yesterday.


(2) No 2 of the members sold the same number of tickets yesterday.

(inequalities, medium)

(1) SUFFICIENT. If none of the 3 members sold at least 2 raffle tickets


yesterday, they sold at most 3 tickets in all, which is not consistent with
the information in (1).

(2) If no two members sold the same number of tickets, then the lowest
number of tickets sold by the most successful member was 2.
SUFFICIENT

157
Division R of Company Q has 1,000 employees. What is the average (arithmetic
mean) annual salary of the employees at Company Q ?

(1) The average annual salary of the employees in Division R is $30,000.


(2) The average annual salary of the employees at Company Q who are not
in Division R is $35,000.

(ratios, medium)

(T) We know that the average annual salary of the employees at Company Q
is between $30,000 and $35,000, but without information on the number
of employees in R, the exact figure cannot be found. NOT SUFF

158
Does the decimal equivalent of P/Q, where P and Q are positive integers,
contain only a finite number of nonzero digits?

(1) P>Q
(2) Q=8

(ratios, medium)

Note that 1/3, 11/3, 4/7, 4/9 have an infinite number of non-zero digits but that
1/5, 13/2 and 7/4 do not. If Q has any prime factors other than 2 and 5, P/Q
may contain an infinite number of non-zero digits. If not, P/Q will be a
terminating decimal.

(1) does not give us the information about Q that we need NOT SUFF

(2) tells us that Q= 8= 23. Whatever the value of integer P, P/Q will either be
an integer or the sum of an integer and an element of {0.125, 0.25, 0.375, 0.5,
0.625, 0.75, 0.875}, so we know that P/Q has a finite number of non-zero
digits, at most 3 to the right of the decimal point. SUFF

159
Does the integer k have a factor p such that 1 < p < k ?

(1) k > 4!
(2) 13! + 2 ≤ k ≤ 13! + 13

(factors and multiples, medium)

In other words, is k not a prime number?

(1) k could be 25, which is not a prime number, or 29, which is a prime
number. NOT SUFF
(2) k = 13! + t where t is an integer from 2 to 13. As 13! is a multiple of
every integer from 2 to 13, 13! is a multiple of t, and so is k. Thus t is a
factor of k. SUFF

160
During a certain season, a team won 80% of its first 100 games and 50 percent
of its remaining games. If the team won 70% of its games for the entire
season, what was the total number of games that the team played?

180 170 156 150 105

(percents, algebra, medium)

If the team played 100 + x games in all, it won 80 + 0.5x of them.


Thus 80 + 0.5x = 0.7 ( 100 + x) −> 10 = 0.2x , so x = 50
In all, 100 + 50 = 150 games were played.

180 170 156 150 105

161
During a 40-mile trip, Marla travelled at an average speed of x miles an hour for
the first y miles of the trip and at an average speed of 1.25x miles an hour for
the remaining 40-y miles of the trip. The time that Marla took to travel the 40
miles what percent of the time it would have taken her if she had travelled at
an average speed of x miles per hour for the whole trip?

(1) x=48
(2) y=20

(movement, hard)

Remember that distance= rate x time.


Time that Marla took: y/x + (40-y)/1.25x= y/x + (32-4y/5)/x= (32+y/5)/x
Time that she would have taken at an average speed of x = 40/x
Required percentage: 100(32+y/5)/40

Hopefully, you realised beforehand that the actual distance is irrevelant, and
that the percentage will depend only on y.

(1) NOT SUFF


(2) SUFF

162
During an experiment, some water was removed from each of 6 water tanks. If
the standard deviation of the volumes of water in the tanks at the beginning of
the experiment was 10 gallons, what was the standard deviation of the volumes
of water in the tanks at the end of the experiment?

(1) For each tank, 30% of the volume of water that was in the tank at
the beginning of the experiment was removed during the experiment.
(2) The average (arithmetic mean) volume of water in the tanks at the end
of the experiment was 63 gallons.

(statistics, hard)

(1) Remember that the standard deviation is the average distance from the
elements to the arithmetic mean of the elements. It stands to reason,
then, that if each element is reduced by 30%, the arithmetic mean will
be reduced by 30%, as will the distances from the elements to the
arithmetic mean. So the standard deviation of the volumes of water at
the end of the experiment will be 30% lower than 10, i.e. 7.

(v1  v ) 2  (v2  v ) 2  ...  (v6  v ) 2


Technically,  
2
where  2 is the
6
variance, the square of the standard deviation. At the end of the
experiment, the sum of the terms in the numerator will be 0.72 times the
sum of the terms in the numerator at the beginning of the experiment
and so the new standard deviation will be 0.7 times the old standard
deviation. SUFF

(2) No information is given about the new volumes. NOT SUFF

163
During a one day sale, a store sold each sweater of a certain type for $30 more
than the store's cost to purchase each sweater. How many of those sweaters
were sold during the sale?

(1) During the sale the total revenue from sale of these sweaters was $270
(2) During the sale, the store sold each of these sweaters at a price that
was 50% greater than the store's cost to purchase each sweater.

(percents, medium)

Suppose that the store purchased each of the n sweaters it bought for $c. It
then sold each sweater for $(c + 30).

(1) n(c + 30) = 270. As we do not know the value of c, we cannot


determine the value of n. NOT SUFF
(2) c + 30 = 1.5c. We can find the value of c, but no information is given to
allow us to find the value of n. NOT SUFF
(T) SUFF

164
During a 10-week summer vacation, was the average (arithmetic mean)
number of books that Carolyn read per week greater than the average number
of books that Jacob read per week?

(1) Twice the average number of books that Carolyn read per week was
greater than 5 less than twice the average number of books that Jacob
read per week.
(2) During the last 5 weeks of the vacation, Carolyn read a total of 3 books
more than Jacob.

(algebra, medium)

(1) Suppose C and J are the average numbers of books that Carolyn and
Jacob read respectively. 2C > 2J - 5, so C > J – 5/2. C may or may not
be greater than J. NOT SUFF
(2) Nothing is said about the first three weeks. NOT SUFF
(T) NOT SUFF

165
During a trip on an expressway, Don drove a total of x miles. His average speed
on a certain 5 mile section of the expressway was 30 miles per hour, and his
average speed for the remainder of the trip was 60 miles per hour. His travel
time for the x mile trip was what percent greater than it would have been if he
had traveled at a constant rate of 60 miles per hour for the entire trip?

8.5% 50% x/12% 60/x% 500/x%

(movment, percents, hard)

If he had travelled at a constant rate of 60 miles an hour, the trip would have
taken x/60 hours. At a constant rate of 60 miles per hour, the time he would
have taken for the 5 mile section that took him 5/30 =1/6 an hour, would have
been only 5/60= 1/12 hour. Thus the 5 mile section added 1/6 -1/12 =1/12
hour to a trip of x/60 hours. As (1/12)/(x/60)= 5/x, his travel time was 500/x
% greater than it would have been at a constant rate of 60 miles per hour.

8.5% 50% x/12% 60/x% 500/x%

166
During a sale, a clothing store sold each shirt at a price of $15 and each
sweater at a price of $25. Did the store sell more sweaters than shirts during
the sale?

(1) The average (arithmetic mean) of the prices of all of the shirts and
sweaters that the store sold during the sale was $21
(2) The total of the prices of all the shirts and sweaters that the store sold
during the sale was $420.

Suppose they sold x shirts and y sweaters, we are asked whether y > x.

(1) 15x + 25y = 21 (x + y) = 21x + 21y


4y = 6x and thus y=3x/2 > x. SUFF

Observe that since the average price is greater than the average of the prices,
more of the expensive garments (i.e. sweaters) were sold than of the cheap
ones (i.e. shirts).

(2) NOT SUFF

167
Each correctional facility in a certain state has the same number of interns, and
each facility has interns of each sex. If Lockwood Correctional Facility has 22%
of the female interns in the state, does it have less than 20% of the male
interns?

(1) Lockwood has between 18% and 22% of the state's male interns.
(2) Lockwood has more female interns than does any other facility in the
state.

(ratios, hard)

Suppose that there are n facilities. Each facility has 100/n % of the interns.

(1) As Lockwood has between 18% and 22%the male interns and 22% of
the female interns, it has between 18% and 22% of all residents.

Therefore 18 < 100/n < 22, and since n is an integer, n=5.


Suppose there are m male interns and f female interns in the state system.
20% are in Lockwood, so if p% of the males are in Lockwood,
0.2(m+f) =pm/100 + 0.22f
0.02f=0.2m –pm/100= m(0.2 –p/100)

As 0.02f > 0 and m>0, 0.2 – p/100 > 0 and p < 20 SUFF

(2) On average, the facilities have 100/n % of the female interns, and so
100/n < 22. Thus n is at least 5,and each facility has an average of at most
20% of the male interns. As Lockwood has a higher than average number of
female interns, it must have a less than average of male interns, as each
resident has the same number of interns. Thus Lockwood has less than 20% of
the male interns. SUFF

168
Each employee of Company Z is an employee of either Division X or Division Y,
but not both. If each division has some part-time employees, is the ratio of the
number of full-time employees to the number of part-time employees greater
for Division X than for Company Z?

(1) The ratio of the number of full-time employees to the number of part-
time employees is less for Division Y than for Company Z.
(2) More than half of the full-time employees of Company Z are employees
of Division X, and more than half of the part-time employees of company
Z are employees of Division Y.

(ratios, hard)

Note that the company ratio for the number of full-timers to part-timers must
be between the corresponding ratios for Divisions X and Y if these ratios are not
the same.

(1) As Division’s Y’s ratio is below the company average, Division X’s ratio
must be above the company average. SUFF
(2) Division X has more than half of the full-timers and less than half on the
part-timers, and Division Y has less than half of the full-timers and more
than half of the part-timers. Thus division X’s ratio will be greater than
division Y’s and thus Company Z’s. SUFF

169
Each employee on a task force is either a manager or a director. What percent
of the employees on the task force are directors?

(1) The average (arithmetic mean) salary of the managers on the task force
is $5,000 less than the average salary of all employees on the task force.
(2) The average (arithmetic mean) salary of the directors on the task force is
$15,000 greater than the average salary of all employees on the task
force.

(ratios, hard)

Neither (1) or (2) is sufficient alone, as (1) does not mention directors and (2)
does not mention managers. Together if there are m managers and d directors
and the average salary of all employees is x thousands of dollars, we can say:

m(x-5) +d(x+15)= (m+d)x, so 15d=5m and d:m=1:3. Thus 1/4 of all


employees are directors.

(T) SUFF

170
Each of 5 households owes at least one television set. What is the standard
deviation of the numbers of television sets owned by the 5 households ?

(1) At least 4 of the households own the same number of television sets.
(2) One of the households owns 3 television sets.

(statistics, medium)

(T) If the households that have the same number of televisions each have t
sets, the standard deviation will depend on the difference between the number
of sets the fifth household has, 3, and t. This difference may be any positive
number, so the standard deviation cannot be determined. NOT SUFF

171
Each of 10 machines works at the same constant rate doing a certain job. The
amount of time needed by the 10 machines working together to complete the
job is 16 hours. How many hours would be needed if only 8 of the machines,
working together, were used to complete the job?

18 20 22 24 26

(combined work, medium)

To complete the job, we need the 10 ten machines to work 16 hours each, a
total of 160 machine hours. 8 machines would need to work 160/8= 20 hours
each.
Alternatively, you could realize the the number of hours is inversely proportional
to the number of machines, so if we have 8/10=4/5 of the machines working,
the time needed will be 5/4 of the 16 hours needed by the 10 machines.

18 20 22 24 26

172
Each of the numbers w,x,y and z are equal to either 0 or 1. What is the value of
w+x+y+z?

(1)

(2)

(percents, medium)

(1) Getting a common denominator of 16,


23w + 22x + 2y + z = 11
Therefore, w = 1 , x= 0 , y = 1, z =1 SUFF

(2) Similarly, 33w + 32x + 3y + z = 11


Therefore, w = 1 , x= 0 , y = 1, z =1 SUFF

173
Each of the students in a certain class received a single grade of P, F, or I.
What percent of the students in the class were females?

(1) Of those who received a P, 40 percent were females.


(2) Of those who received either F or I, 80 percent were females.

(sets, percents, hard)

(1) No information is given about the students who did not receive a P.
NOT SUFF
(2) No information is given about the students who received a P.
NOT SUFF

(T) Since 40% of the students who received a P and 80% of the students
who did not receive a P were females, between 40% and 80% of the
students in the class were females. The exact percentage cannot be
determined without information about what percent of the class received
a P. NOT SUFF

Suppose x% of the class of n students received a P, then 0.4nx/100


students were females who received a P and 0.8n(100-x)/100 students
were females who did not receive a P. Thus the total number of females
was n(80 – 0.4x), (80 – 0.4x) %.

174
Each of the 25 balls in a certain box is either red, blue or white and has a
number from 1 to 10 painted on it. If one ball is to be selected from the box,
what is the probability that the ball selected will either be white or have an
even number painted on it?

(1) The probability that the ball will both be white and have an even number
painted on it is zero.
(2) The probability that the ball will be white minus the probability that the
ball will have an even number painted on it is 0.2.

(probability, sets, hard)

Each ball is equally likely to be drawn, so the probability that the ball will either
be white or have an even number painted on it is (w + e – b)/25, where w is
the number of white balls among the 25 balls, e is the number of balls marked
with an even number, and b is the number of white balls marked with an even
number.

(1) b=0. No information is given about w or e. NOT SUFF


(2) Pr (W) – Pr (E) =( w – e )/25 =0.2. No information is given about b.
NOT SUFF
(T) We have no way to find the value of w + e. Knowing that w – e is
not sufficient. NOT SUFF

175
Each of the 45 boxes on shelf J weighs less than each of the 44 boxes on shelf
K. What is the median weight of the 89 boxes of these shelves?

(1) The heaviest box on the shelf J weighs 15 pounds


(2) The lightest box on the shelf K weighs 20 pounds

(statistics, medium)

There are 89 boxes in total, so if the boxes are considered from lightest to
heaviest, we need to know the weight of the 45th box, which will be the
heaviest of the boxes on shelf J. (1) gives us this weight, but (2) does not.
All (2) tells us is that the median weight is less than 20 pounds.
(1) SUFF
(2) NOT SUFF

176
Each of the offices in a certain building has a floor area of 200, 300, or 350
square feet. How many offices are on the first floor of the building?

(1) There is a total of 9500 square feet of office space on the first floor of
the building.
(2) Ten of the offices on the first floor have areas of 350 square feet each.

(factors and multiples, hard)

Here’s a strategy to be used only when you are quite sure that neither
statement is sufficient by itself: combine the information from (1) and (2) right
away. If 10 of the offices on the first floor have an area of 350 square feet each
(from (2)), according to (1), the remaining offices have a combined area of
6000 square feet. However, 6000 is a multiple of both 200 and 300, so there
could be 30 offices of 200 square feet or 20 offices of 300 square feet each, or
even 12 of each of these two sizes. NOT SUFFICIENT

177
Each person attending a fund-raising party for a certain club was charged the
same admission fee. How many people attended the party?

(1) If the admission fee had been $.75 less and 100 more people had
attended, the club would have received the same amount in admission
fees.
(2) If the admission fee had been $1.50 more and 100 fewer people had
attended, the club would have received the same amount in admission
fees.

(algebra, hard)

Suppose that the n people attending the party were charged x dollars each. The
the club received nx dollars in admissions fees. We are asked about the value
of n.

(1) ( n + 100 ) ( x – 0.75 ) = nx + 100x – 0.75n – 75 = nx


Thus 100x – 0.75n = 75 and the value of n depends on the value of x.
NOT SUFF

(2) ( n – 100 ) ( x + 1.5 )= nx – 100x + 1.5n – 150 = nx


Thus – 100x + 1.5n = 150
NOT SUFF

(T) We have a system of two independent linear equations involving n and x.


Only one value of n satisfies both, and it can be found by adding
equations: 2.25n = 225 and n = 100. SUFF

178
Each person in a certain group supports only one of two candidates R and T. Of
the people in the group, 45 percent support Candidate R and the rest support
Candidate T. How many people in the group are in favor of a flat tax ?

(1) Of the people in the group who support Candidate R, 58 percent are in
favor of flat tax
(2) Of the people in the group who support Candidate T, 22 are in favor of a
flat tax.

(percents, sets, medium)

(T) If there are n people in the group, the number who support a flat tax is
0.58(0.45n) + 22. Since we do not know the value of n, more
information is needed. NOT SUFF

179
Each week, Harry is paid x dollars per hour for the first 30 hours and 1.5x
dollars for each additional hour worked that week. Each week, James is paid x
dollars per hour for the first 40 hours and 2x dollars for each additional hour
worked that week. Last week James worked a total of 41 hours. If Harry and
James were paid the same amount last week, how many hours did Harry work
last week?

35 36 37 38 39

(algebra, hard)

James was paid $42x last week, so if Harry worked 30 + y hours last week, he
was paid $(30x +1.5xy). Thus 12x =1.5xy and y=8, meaning that Harry worked
38 hours last week.

35 36 37 38 39

180
Each side of a certain parallelogram has length 6. if the area of the
parallelogram is 18. Which of the following is the measure of one of its angles
in degrees ?

30 45 60 90 120

(geomerty, hard)

As the area of the parallelogram is 18, the product of the length of one side
and the perpendicular distance beween that side and the opposite parallel side,
this perpendicular distance is 18/6= 3

As the leg opposite the angle with measure xº is 1/2 the length of the
hypotenuse, x = 30.

30 45 60 90 120
181
Ellen can purchase a certain computer at a local store at a price of p dollars and
pay a 6 percent sales tax. Alternatively, Ellen can purchase the same computer
from the catalog for a total of q dollars, including all taxes and shipping costs.
Will it cost more for Ellen to purchase the computer from the local store than
from the catalog?

(1) q – p < 50
(2) q = 1,150

(percents, medium)

We need to know whether 1.06p > q, i.e. p/q > 1/1.06 =100/106=50

(1) q – p < 50 . If p and q are large numbers p/q > 1/1.06, meaning that
the answer would be yes, but if p and q are small numbers (for example
p=100 and q=140), the answer could be no. NOT SUFF
(2) No information is given about p. NOT SUFF
(T) p > q – 50 = 1100 , so 1.06p > 1100 + 66 > 1150 = q SUFF

182
Equation |x/2| +|y/2|= 5 encloses a certain region in the xy- coordinate plane..
What is the area of the region?

20 50 100 200 400

(geometry, hard)

(coordinate geometry, hard)

Note that |x| =|-x| for all values of x, the graph is symmetrical about both the
horizontal and vertical axes.

Multiplying both sides by 2, we get |x| + |y| = 10


In the first quadrant, |x| = x and |y| = y, i.e. x + y = 10
The boundary of the region in the first quadrant is a line segment with
endpoints (10,0) and (0,10). The area bound by x + y = 10, and the two
coordinate axis, a triangle, has area 50 (base = height = 10).
The equation describes identical triangles in the other 3 quadrants, so the area
of the region is 200.
Alternatively, recognize that the region formed is a square with side 10 2 , the
area of which is therefore (10 2 )2 = 200

20 50 100 200 400

183
5 blue marbles, 3 red marbles and 4 purple marbles are placed in a bag. If 4
marbles are drawn without replacement, what is the probability that the result
will not be 2 blue and 2 purple marbles?

4/33 (5/36)2 1/2 (31/36)2 29/33

(probability, hard)

Rather than calculate the required probability directly, which would require
considering many cases, it is easier to calculate the probability that the the
12  11 10  9
result will be 2 blue and 2 purple marbles. There are C4  equally
4  3 2
12

probable outcomes, of which 5 C2 4 C2 correspond to the event that exactly 2


blues and 2 purples are drawn. Thus the probability that 2 blues and 2 purples
are drawn is (10 x 6)/(11 x 5 x 9) = 4/33, and the probability of the converse is
29/33.

4/33 (5/36)2 1/2 (31/36)2 29/33

184
Five pieces of wood have an average (arithmetic mean) length of 124
centimeters and a median length of 140 centimeters. What is the maximum
possible length, in centimeters, of the shortest piece of wood?

90 100 110 130 140

(statistics, hard)

The sum of the lengths is 124(5) = 620 centimeters. Suppose the lengths of
the pieces of wood are, in ascending order, w1 , w2, w3, w4 and w5. We know
that w3 = 140 and w3 ≤ w4 ≤ w5 and w1 = 620 – (w2 + w3 + w4 + w5). Thus to
maximize the length of the shortest piece of wood, we need to mimimize the
values of w2, w3, w4 and w5. Let w4 = w5 = 140 and w1 = w2. We get 2w1 =
200, so the maximum value of m1 is 100.

90 100 110 130 140

185
For a certain set of n numbers, where n>1, is the average (arithmetic mean)
equal to the median?

(1) If the n numbers in the set are listed in increasing order, then the
difference between any pair of successive numbers is 2.
(2) The range of the n numbers in the set is 2(n - 1).

(statistics, hard)

(1) If the terms are written in increasing order so that x=a1 < a2 <...< an,
An= x +2(n-1).
Thus the terms form an arithmetic sequence, and in all arithmetic
sequences, the median is equal to the arithmetic mean, which
incidentally is the mean of the first and last terms of the sequence. SUFF

(2) If the range of the n numbers in the set is 2(n-1), the set could form an
arithmetic sequence, in which case, its mean would be equal to the
median. If n>2, it could also be that all but 1 term is equal to x and 1
term is equal to x + 2(n – 1). Clearly, in this case, the median would be
x and the mean would be greater than x. NOT SUFF

186
For a convention, a hotel charges a daily room rate of $120 for 1 person and x
dollars for each additional person. What is the charge for each additional
person?

(1) The daily cost per person for 4 people sharing the cost of a room equally
is $45.
(2) The daily cost per person for 2 people sharing the cost of a room equally
is $25 more than the corresponding cost for 4 people.

(algebra, medium)

Note the total cost for a room holding n people is 120 + ( n – 1 )x. We are
asked for the value of x.

(1) (120 + 3x)/4 = 45, from which we get a linear equation involving x
SUFF
(2) (120 + x)/2 = 25 + (120 + 3x)/4, from which we get a linear equation
involving x SUFF

187
x2  2
For all numbers x such that x ≠ 1, if g(x) is defined by g ( x)  , then
x 1
1 1
 
g (2) g ( x)

6( x  1) 6( x 2  2) x2  2 x 1 x2  2
x2  2 x 1 2( x  1) 6( x 2  2) 6( x  1)

(functions, medium)

6( x 2  2) 1 1 x 1
Note that g(2)=6, so g(2)g(x)= . Thus  
x 1 g (2) g ( x) 6( x 2  2)

x 1
6( x  1) 6( x 2  2) x2  2 x2  2
x2  2 x 1 2( x  1) 6( x 2  2) 6( x  1)

188
For all positive integers m and v, the expression m ** v represents the
remainder when m is divided by v.
What is the value of [(98**33)**17] – [98**(33**17)] ?

-10 -2 8 13 17

(functions, factors and multiples, hard)

Note that 99 is a multiple of 33 and 34 is a multiple of 17.


Thus 98*33= 32 and 33*17 = 16, and the expression can be simplified as
32**17 – 98**16 = 15 – 2 = 13.

-10 -2 8 13 17

189
For all positive integers m, m = 3m when m is odd and m = 1 m when m is
2
even. Which of the following is equivalent to 9 x 6 ?

81 54 36 27 18

(functions, hard)

Since 9 is an odd number and 6 is an even number, 9 x 6 = 27 x 3 = 81 =


27 = 162

81 54 36 27 18

190
For a finite sequence of nonzero numbers, the number of variations in sign is
defined as the number of pairs of consecutive terms of the sequence for which
the product of the two consecutive terms is negative. What is the number of
variations in sign for the sequence 1, -3, 2, 5, -4, -6?

One Two Three Four Five

(sequences, medium)

In other words, when reading the numbers from left to right, how many times
does the sign change? Three times: from the first to the second element, from
the second to the third and from the fourth to the fifth.

One Two Three Four Five

191
For all integers n, the function p is defined by p(n) = an, where p is a constant.
What is the value of p(1) ?

(1) p(2) =100


(2) p(3) = -1,000

(functions, exponents, medium)

We are asked to find the value of p(1) = a1 = a.

(1) a2 = 100, so |a|= 10. NOT SUFF


(2) a3 = -1000, so a = - 10 SUFF

192
For all numbers x, the function f is defined by f(x) = 3x + 1 and the function g
is defined by g(x) = (x – 1)/3 If c is a positive number, what is the value of
g(c)?

(1) f(c) = 13
(2) f(1) = c

(functions, medium)

It is sufficient for us to find the value of c.

(1) (3c – 1)/3 = 13 SUFF


(2) c = f(1) = 3(1) + 1 SUFF

193
For a nonnegative integer n, if the remainder is 1 when 2n is divided by 3, then
which of the following must be true?

I. n is greater than zero.


II. 3n = (-3)n
III. 2 n is an integer.

I only II only I and II I and III II and III

(factors and multiples, medium)

Note that 20 = 1, which, when divided by 3, yields a remainder of 1.


Also, 21 = 2, which, when divided by 3, yields a remainder of 2.

If positive integer k is 1 greater than a multiple of 3 (i.e. of the form 3x + 1,


where x is a non-negative integer), 2k = 2(3x + 1) = 3(2x) + 2 is 2 greater
than a multiple of 3 (i.e. yields a remainder of 2 when divided by 3).

Similarly, if positive integer k is 2 greater than a multiple of 3 (i.e. of the form


3x + 2, where x is a non-negative integer), 2k = 2(3x + 2) = 3(2x) + 4 is 1
greater than a multiple of 3 (i.e. yields a remainder of 2 when divided by 3).

Thus if n is even, 2n yields a remainder of 1 when divided by 3. If n is odd, 2n


yields a remainder of 2 when divided by 3. Therefore, we know that n is even,
and II and III must be true.

One could suspect that the above is true by looking at the remainders when 20,
21, 22 , 23 and 24 are divided by 3: the remainders alternate between 1 and 2.

I only II only I and II I and III II and III

194
For any integer k > 1, the term ―length of an integer‖ refers to the number of
positive prime factors, not necessarily distinct, whose product is equal to k. For
example, if k = 24, the length of k is equal to 4, since 24 = 2 × 2 × 2 × 3. If x
and y are positive integers such that x > 1, y > 1, and x + 3y < 1000, what is
the maximum possible sum of the length of x and the length of y?

5 6 15 16 18

(factors and multiples, medium)

Note that 27 = 128, 28 =256 and 29= 512. 29 + 3 × 27 < 1000

5 6 15 16 18

195
For any integer k greater than 1, the symbol k* denotes the product of all the
fractions of the form 1/t, where t is an integer between 1 and k, inclusive. What
is the value of 5*/4*?

5 5/4 4/5 1/4 1/5

(functions and sequences)

k* = 1/1 x 1/2 x 1/3 x …x 1/(k – 1 ) x 1/k = 1/k!

Thus 5*/4*=4!/5! = 1/5

5 5/4 4/5 1/4 1/5

196
For any positive integer n, the length of n is defined as the number of prime
factors whose product is n. For example, the length of 75 is 3, since 75 = 3 x 5
x 5. How many two-digit positive integers have length 6 ?

None One Two Three Four

Perhaps the easiest way to tackle this question is to find the smallest positive
integer with length 6: 26 =64. If one of the 2’s is replaced with a 3, we get
25 x 3 = 96. Thus there are two such integers with two digits.

None One Two Three Four

197
For any positive integer x, the 2-height of x is defined to be the greatest
nonnegative integer n such that 2n is a factor of x. If k and m are positive
integers, is the 2-height of k greater than the 2-height of m?

(1) k>m
(2) m/k is an even integer.

(factors and multiples, medium)

Note that 2-height of 100= (22)(52) is 2 since 100 is divisible by 22 but not by
23.

(1) The 2-height of 28 is greater than the 2-height of 27 but greater than
that of 28 + 1. NOT SUFF
(2) If m/k is an even integer, m/k = 2x for some positive integer x. Thus the
2-height of m is x greater than the 2-height of k. SUFF

198
For any triangle in the xy-coordinate plane, the center of T is defined to be the
point whose x-coordinate is the average (arithmetic mean) of the x-coordinates
of the vertices of T and whose y-coordinate is the average of the y-coordinates
of the vertices of T. If a certain triangle has vertices at the points (0,0) and
(6,0) and center at point (3,2) what are the coordinates of the remaining
vertex?

(3, 4) (3, 6) (4, 9) (6, 4) (9, 6)

(coordinate geometry, medium)

If the center is (3,2), the sum of the x-coordinates of the 3 vertices


must be 9 and that of the y-coordinates must be 6. Thus the
remaining vertex has coordinates (3,6).

(3, 4) (3, 6) (4, 9) (6, 4) (9, 6)

199
For a recent play performance, the ticket prices were $25 per adult and $15 per
child. A total of 500 tickets were sold for the performance. How many of the
tickets sold were for adults?

(1) Revenue from ticket sales for this performance totaled $10,500
(2) The average (arithmetic mean) price per ticket sold was $21.

(algebra, medium)

Suppose a adult and c children tickets were sold. We are told that a + c = 500.
The revenue from ticket sales is 25a + 15c, and we are asked to find the value
of a.

(1) 25a + 15c =10,500 As we have two equations, we can solve for a:
25a + 15c = 10500
15a + 15c = 15(500)=7500
Thus 10a = 3000 and a= 300 SUFF
(2) 25a + 15c = 21 (a + c)
As we have two equations, we can solve for a:
4a = 6c, so c= 2a/3
So a + c =500 yields 5a/3 = 500 and a= 3(500)/5 = 300 SUFF
Also, notice that (1) and (2) give us the same information

200
For each home sold in County X, the buyer and the seller each must pay to
County X a tax of 0.5 percent of the sale price of the home. Colleen recently
sold her old home and bought a new home, both in County X. What was the
total tax that Colleen paid to County X on these home sales?

(1) Colleen’s old home had a sale price of $169,500.


(2) Colleen’s new home had a sale price 20% greater than that of her old
home.

(percents, medium)

(T) The total tax is 0.005 (169,500)(2.2) SUFF

201
For each of her sales, a saleswoman receives a commission equal to 20 percent
of the first $500 of the total amount of the sale, plus 30% of the total amount
of the sale in excess of $500. If the total amount of one of her sales was $800,
the saleswoman's commission was approximately what percent of the total
amount of the sale?

22 24 25 27 28

(percents, hard)

Her commission on a sale of $800 is 20% of 500 plus 30% of 300 = $190,
which is 23.75% of $800.

22 24 25 27 28

202
For every integer k from 1 to 10, inclusive, the kth term of a certain sequence is
k 1
given by (1) 1 2 k
If T is the sum of the first 10 terms in the sequence, then
T is

greater than 2 between 1 and 2 between 1/2 and 1


between 1/4 and 1/2 less than ¼

(sequences, hard)

T= (1/2 -1/4) + (1/8 -1/16) +…(1/29- 1/210) =. By grouping the terms of the
sequence in this way, we see that T is equal to ¼ added to positive numbers
and thus T > 1/4.
Also, T= 1/2 - (1/4 -1/8) – (1/16-1/32) - … (1/28 – 1/29) – 1/210, so T< 1/2

Alternatively, one could begin adding and realize that the sum converges on a
number between 1/4 and 1/2.

greater than 2
between 1 and 2 between 1/2 and 1
between 1/4 and 1/2 less than ¼

203
For every positive even integer n, h(n) is defined to be the product of all the
even integers from 2 to n, inclusive. If p is the smallest prime factor of
h(100)+1, then p is

between 2 and 10 between 10 and 20


between 20 and 30 between 30 and 40
greater than 40

(factors and multiples, hard)

h (100)= 2 x 4 x 6 x...x 100= 250 x 50!.

As h(100) is a multiple of every positive integer from 2 to 50, h(100) + 1 is not


a multiple of any positive integer from 2 to 50. Thus, the smallest prime factor
of h(100) +1 must be greater than 50.

between 2 and 10 between 10 and 20


between 20 and 30 between 30 and 40
greater than 40

204
2x + y = 12
|y| ≤ 12

For how many ordered pairs (x , y) that are solutions of the system above are x
and y both integers?

7 10 12 13 14

(algebra, factors and multiples)

Note that x = 6 – y/2 , so for x to be an integer, y must be an even integer.


There are 13 even integers from -12 to 12, (6 positive, 6 negative and 0).

7 10 12 13 14

205
For how many integers n is 2n = n2 ?

None One Two Three More than three

(exponents, medium)

For all integers n such that n < 0, 2n < 1 and n2 ≥ 1 , so 2n ≠ n2 .

Note that 20=1 >0 = 02, 21 = 2 > 12 , 22 = 4 = 22 , 23 < 32 , 24 = 42, 25 > 52

Suppose that f(n) = 2n . f(n+1) = 2f(n), so f(n+1)/f(n) = 2. Suppose that g(n)


= n2. g(n+1)/g(n) = (n2 + 2n + 1)/n2 = 1 + 2/n + 1/n2 Note that for all n > 5,
g(n+1)/g(n) < 2. Thus whereas the value of 2n doubles with each unit increase
in n, that of n2 increases by less than 100% with each unit increase in n.

This means that the only integers for which 2n = n2 are n=2 and n=4.

None One Two Three More than three

206
For Manufacturer M, the cost C of producing x units of its product per month is
given by C= kx + t, where C is in dollars and k and t are constants. Last month,
if Manufacturer M produced 1,000 units of its product and sold all the units for
k+60 dollars each, what was Manufacturer M’s gross profit on the 1,000 units ?

(1) Last month, Manufacturer M’s revenue from the sale of the 1,000 units
was $150,000.
(2) Manufacturer M’s cost of producing 500 units in a month is $45,000 less
than its cost of producing 1,000 units in a month.

(functions, hard)

The gross profit on the 1,000 units is the difference between the revenue on
the 1,000 units and the cost of producing 1,000 units per month.

(1) Information is given about the revenue but not about the cost. We do
know that k + 60 = 150,000/1,000 = 150, i.e. k = 90., but we are not
given any information about the value of t. NOT SUFF
(2) (1000k + t) – (500k + t) = 45,000, so k = 90. No information is given
about the revenue or about the value of t. NOT SUFF
(T) No information is given to permit us to find the value of t. NOT SUFF

207
For one toss of a certain coin, the probability that the outcome is heads is 0.6.
If this coin is tossed 5 times, which of the following is the probability that the
outcome will be heads at least 4 times ?

0.65 2(0.6)4 3 (0.6)4(0.4)


4(0.6)4 (0.4) + (0.6)5 5(0.6)4 (0.4) + (0.6)5

(probability, hard)

Either the outcome is heads 4 times or the outcome is heads 5 times. For the
five independent tosses to result in 4 heads and thus 1 tails, we could have
HHHHT, HHHTH, HHTHH, HTHHH, THHHH. As the probability each of these five
mutually exclusive outcomes is (0.6)4 (1 - 0.6) = (0.6)4 (0.4), the probability
that there will be 4 heads is 5(0.6)4 (0.4). As the probability that each of the 5
tosses results in heads is (0.6)5, the probability that the outcome will be heads
at least 4 times is 5(0.6)4 (0.4) + (0.6)5.

0.65 2(0.6)4 3 (0.6)4(0.4)


4(0.6)4 (0.4) + (0.6)5 5(0.6)4 (0.4) + (0.6)5
208
For the students in Class A, the range of their heights is r cm and the greatest
height is g cm. For the students in class B, the range of their heights is s cm.
and the greatest height is h cm. Is the least height of the students in Class A
greater than the least height of students in Class B?

(1) r<s
(2) g>h

(statistics, medium)

The least height of the students in class A is g – r, the greatest height g


subtract the range r. Similarly, the least height of the students in class B
is h – s. We are asked whether g – r > h – s , which will be the case if and
only if g – h > r – s.

(1) r – s < 0. However, nothing is said about g and h. INSUFF


(2) g – h > 0. However, nothing is said about r and s. INSUFF
(T) g–h>r–s SUFF

209
For which of the following functions f is f(x) = f (1 – x) for all x?

f(x)= 1 – x
f(x)= 1 – x2
f(x)= x2 – (1 – x)2
f(x)= x2(1 – x)2
x
f(x)=
1 x

(functions, medium)

The quickest approach in this case is to see whether f(0) = f(1) in each case.
2 2
Only f(x) = x (1 – x) passes this test. Also note that 1 – (1 – x) = x, so if
f(x)= x2(1 – x)2 f(1 – x) = (1 – x)2x2 = f(x)

210
For which of the following functions is f(a + b) = f(a) + f(b) for all positive
numbers a and b ?

f(x) = x2 f(x) = x + 1 f(x) = x f(x) = 2/x f(x) = -3x

(functions, medium)

Note that (a+b)2= a2 +b2 + 2ab ≠ a2 + b2 if a and be are positive. Nor is a + b


+ 1 equal to (a+1) +(b+1). If a and b are positive, a  b  a  b and
2 2 2 2(a  b)
   . However, -3(a+b)= -3a - 3b for all values of a and b.
ab a b ab

f(x) = x2 f(x) = x + 1 f(x) = x f(x) = 2/x f(x) = -3x

211
For which of the following values of x is 1 2 x NOT defined as a real
number?

1 2 3 4 5

(functions, roots, medium)

Note that the expression is not defined if 2  x  0 , i.e. if x  2 . Squaring


both sides of this last inequality, it is clear that this will occur for all values of x
greater than 4.

1 2 3 4 5

212
Four extra-large sandwiches of exactly the same size were ordered for m
students, where m > 4. Three of the sandwiches were evenly divided among
the students. Since 4 students did not want any of the fourth sandwich, it was
evenly divided among the remaining students. If Carol ate one piece from each
of the four sandwiches, the amount of sandwich that she ate would be what
fraction of a whole extra-large sandwich?

m4 2m  4 4m  4 4m  8 4m  12
m(m  4) m(m  4) m(m  4) m(m  4) m(m  4)

(ratios, medium)

Three of the sandwiches of divided into m pieces of 1/m th of a sandwich.


The last sandwich is divided into m – 4 pieces of 1/(m – 4 ) th of a sandwich.

Thus Carol ate 3/m + 1/(m -4 ) = (3(m-4) + m)/(m)(m – 4 ) = (4m -12)/m(m-


4) . Checking, if m= 5, she ate 3/5 + 1 = 8/5 = (4(5) -12)/4(5-1)

4m  12
m4 2m  4 4m  4 4m  8
m(m  4) m(m  4) m(m  4) m(m  4) m(m  4)

213
Four staff members at a certain company worked on a project. The amounts of
time that the four staff members worked on the project were in the ratio 2 to 3
to 5 to 6. If one of the four staff members worked on the project for 30 hours,
which of the following CANNOT be the total number of hours that the four staff
members worked on the project?

80 96 160 192 240

(ratios, hard)

The four staff members worked 2k, 3k, 5k and 6k hours repectively, a total of
16k hours. 30 is either 2k, 3k,5k or 6k, so k is either 15,10,6,or 5 and 16k is
either 240,160,96 or 80.

80 96 160 192 240

214
From a group of 3 boys and 3 girls, 4 children are to be randomly selected.
What is the probability that equal numbers of boys and girls will be selected?

1/10 4/9 1/2 3/5 2/3

(probability, hard)

There are C4  C2  15 equally likely groups of 4 children. As there are 3


6 6

subsets of 2 boys and 3 subsets of 3 girls, 9 of these 15 groups consist of 2


boys and 2 girls. Thus the required probability is 9/15 = 3/5

1/10 4/9 1/2 3/5 2/3

215
Guy’s net income equals his gross income minus his deductions. By what
percent did Guy’s net income change on January 1, 1989, when both his gross
income and his deductions increased?

(1) Guy’s gross income increased by 4 percent on January 1, 1989.


(2) Guy’s deductions increased by 15 percent on January 1, 1989.

(percents, hard)

(T) If his previous gross income was i and his deductions d, his previous net
income was i – d and his new net income was 1.04i – 1.15d. However,
without knowledge of the relationship between d and i, the ratio of the
two figures cannot be found, and thus the question cannot be answered.
NOT SUFF

216
Henry purchased 3 items during a sale. He received a 20 percent discount off
the regular price of the most expensive item and 10 percent discount off the
regular price of each of the other 2 items. Was the total amount of the 3
discounts greater than 15 percent of the sum of the regular prices of the 3
items ?

(1) The regular price of the most expensive item was $50, and the regular
price of the second most expensive item was $20.
(2) The regular price of the least expensive item was $15.

(ratios, percents, hard)

(1) Suppose that the price of the least expensive item was x dollars, where
x ≤ 20. Then the sum of the regular prices of the three items was 70 + x
and the total amount of the discounts was 0.2(50) + 0.10(20 + x)
= 12 + 0.1x.
12  0.1x
 0.15  12  0.1x  10.5  0.15 x  0.05 x  1.5  x  30
70  x

In other words, the sum of the discounts will be greater greater than 15
percent of the sum of the regular prices of the 3 items if and only if
x < 30. We know that x ≤ 20. SUFF

Alternatively, the sum of the discounts will be greater than 15% of the
sum of the regular prices as long as the total amount discounted by 20%
is greater than the total amount discounted by 10%. We know that the
former is $50 and the latter is at most $40.

(2) No information is given about the prices of the other two articles.
NOT SUFF

217
How many different numbers are prime factors of n?

(1) 2n has four different prime factors.


(2) n2 has four different prime factors.

(factors and multiples, hard)

(1) tells us that 2n has four different prime factors. We know that n has at least
three different prime factors apart from 2, but it could have a prime factor of 2
as well. Thus n has either 3 or 4 different prime factors. NOT SUFF

(2) Squaring a number does not alter the number of different prime factors- the
number of different prime factors in n2 is equal to the number of different prime
factors in n. SUFF

218
How many different positive integers d are there such that 48 divided by
positive integer d yields a remainder of d - 4?

Two Three Four Five Six

(factors and remainders, hard)

48 = qd + d – 4, so 52 =(q+1)d

d must be a factor of 52 = 13 x 22 and d – 4 must not be negative (i.e. d must


be at least 4). d could be 4, 13, 26 or 52.

Two Three Four Five Six

219
How many different 6-letter sequences consist of 1 A, 2 B’s and 3 C’s ?

6 60 120 360 720

(combinatronics, hard)

The simplest way is to choose one of the six places for the A (6 ways) and 2 of
the remaining 5 places for the B’s ( C2  10 ways). The other 3 places are for
5

the C’s. 6×10= 60

6 60 120 360 720

220
How many positive integers less than 1000 are multiples of 5 but NOT of 4 or
7?

114 121 122 129 136

(sets, factors and multiples, hard)

Since 1000/5 = 200, there are 199 multiples of 5 less than 1000.

However, some multiples of 5 are also multiples of 4 (i.e. mutliples of 20). Since
1000/20 = 50, there are 49 positive multiples of 5 that are also multiples of 4.

Similarly, as 28 < 1000/35 < 29, there are 28 positive multiples of 5 that are
also multiples of 7.

Also, there are positive multiples of 5 that are multiples of both 4 and 7 (i.e.
multiples of 5 × 4 × 7 = 140 ). As 7 < 1000/140 < 8, there are 7 such
multiples.

The number of positive integers less than 1000 are multiples of 5 but NOT of 4
or 7 is 199 – 49 – 28 + 7 = 129

114 121 122 129 136

221
How many more first-time jobless claims were filed in week P than in week T?

(1) For weeks P, Q, R and S, the average (arithmetic mean) number of first-
time jobless claims filed was 388,250.
(2) For weeks Q, R, S and T, the average (arithmetic mean) number of first-
time jobless claims filed was 383,000.

(algebra, medium)

Clearly, neither statement is sufficient by itself: (1) tells us nothing about week
T and (2) tells us nothing about week P.
Together (1) tells us that P + Q + R + S= 4(388,250) and Q + R + S +
T=4(383,000). Subtracting the second equation from the first yields P – T
=4(388,250- 383,000). SUFF

222
How many odd integers are greater than integer x and less than integer y?

(1) There are 12 even integers greater than x and less than y.
(2) There are 24 integers greater than x and less than y.

(inequalities, factors and multiples, hard)

(1) If x is even, x +2(12) is the greatest even integer less than y, so y is


either x + 25 or x + 26. If y= x + 25, the greatest odd integer less than
y is x + 23, but if y = x + 26, the greatest odd integer less than y is x +
25. NOT SUFF

(2) If there are 24 integers greater than x and less than y, exactly half of
these will be odd. 24 consecutive integers can be thought of as 12 pairs
of consecutive integers. In each pair of consecutive integers, there is
one odd integer and one even integer. SUFF

223
How many prime numbers between 1 and 100 are factors of 7,150 ?

One Two Three Four Five

(factors and multiples, medium)

One Two Three Four Five

224
How many seconds will it take a bus that is travelling at a constant of 80
kilometers per hour to travel a distance of 400 meters ? ( 1 kilometer is equal
to 1,000 meters).

5 12 18 20 30

(movement, ratios, medium)

Convert 400 meters to kilometres to hours to minutes to seconds:

400 meters x (1 km/1000 m) x (1 hour/ 80 km) x (60 minutes/1 hr) x


(60 seconds/1 min)

400(60)(60)/(80)(1000) seconds = 4(6)(60)/80 = 6(3) = 18 seconds

5 12 18 20 30

225
How much did the taxi driver charge George for the trip to the airport?

(1) George paid the taxi driver a tip equal to 15 percent of the amount the
taxi driver charged
(2) George paid the taxi driver a tip of $6.00

(percents, medium)

(T) If x was the amount charged, 0.15x = 6 SUFF

226
How much time did it take a certain car to travel 400 kilometers?

(1) The car traveled the first 200 kilometers in 2.5 hours.
(2) If the car’s average speed had been 20 kilometers greater than it was, it
would have travelled the 400 kilometers in 1 hour less time than it did.

(movement, algebra, hard)

(1) Nothing is said about the last 200 kilometers. NOT SUFF
(2) Suppose that the car’s average speed was x kilometers per hour and
thus that it took the car 400/x to travel 400 kilometers. If the car’s
average speed had been 20 kilometers greater than it was, it would have
travelled the 400 kilometers in 400/(x + 20) hours.

400 400
  1  400 x  400 x  80000  x 2  20 x  x 2  20 x  80000  0
x  20 x
( x  100)( x  80)  0  x  80

Since we can find the value of x, we can figure out how long the trip
took: 400/80 = 5 hours. SUFF

227
If Δ denotes one of the four operations addition, multiplication, division and
subtraction, what is the value of 1 Δ 2 ?

(1) n Δ 0 = n for all integers n.


(2) n Δ n = 0 for all integers n.

(algebra, medium)

Since 1+2, 1-2, ½ and 1×2 are all different, we need to know which operation

(1) could be addition or subtraction NOT SUFF


(2) must be subtraction SUFF

228
If a and b are integers, what is the value of a2 + b2 ?

(1) a3 = -8
(2) b4 = 16

(algebra, medium)

(T) From (1), we see that a= - 2 and from (2), |b| = . Thus the
2 2
values of a and b can be found and the question can be answered with
a singular numerical value. SUFF

229
If a and b are nonzero numbers on the number line, is 0 between a and b?

(1) The distance between 0 and a is greater than the distance between 0
and b.
(2) The sum of the distances between 0 and a and between 0 and b is
greater than the distance between 0 and the sum a + b.

(algebra, medium)

In other words, do a and b have different signs?

(1) a and b could have the same sign or different signs. NOT SUFF
(2) If a and b had the same sign, the sum of the distances between 0 and a
and between 0 and b would be equal to6 the distance between 0 and
a+b. Therefore a and b have different signs. SUFF

230
If a and b are positive integers such that a – b and a/b are both even integers,
which of the following must be an odd integer?

a/2 b/2 (a+b)/2 (a+2)/2 (b+2)/2

(factors and multiples, hard)

Since a/b is an even integer, a must be even and since a – b is even, b is even
as well. Thus a=kb, where k and b are even integers. Therefore, a is a multiple
of 4. a + 2 is a multiple of 2 but not of 4, so a +2 is the product of 2 and an
odd number, making (a+2)/2 an odd number.

a/2 b/2 (a+b)/2 (a+2)/2 (b+2)/2

231
-1 -1 -1 -1 -1 -1
If a and b are positive, is (a + b ) less than (a b ) ?

(1) a = 2b
(2) a+b>1

(inequalities, hard)

-1 -1 -1 -1 -1 -1
(a + b ) less than (a b )

if and only if a-1 + b-1 > a-1b-1 = 1 /ab , which (multiplying both sides by ab) is
true iff b + a > 1.

(1) NOT SUFF


(2) SUFF

232
If a and b are positive numbers, what are the coordinates of the midpoint of
line segment CD in the xy-plane?

(1) The coordinates of C are ( a, 1 – b )


(2) The coordinates of D are ( 1 – a , b)

(T) The coordinates of the midpoint of the line segment joining P(x1, y1) and
Q (x2, y2) are ((x1 + x2)/2, (y1 + y2)/2), so neither statement is sufficient
by itself. Together, the coordinates of the midpoint of are (1/2,1/2).
SUFF

233
If a, b and c are integers such that b > a, is it true that b+c > a ?

(1) c>a
(2) abc > 0

(inequalties, hard)

If c > 0, b + c > a. However, if c < 0, b + c < b and b + c may or may


not be greater than a.

(1) If a > 0, c > 0 and b+c > a.


However if c < 0 b +c could be less than a.
For example a=-5, b=c= -4 NOT SUFF
(2) If abc > 0, perhaps each of {a,b,c} is positive, in which case
b+c > a. However, perhaps b is positive and a and c are both
negative. In this case, b +c could be less than a.
For example, a= -1, b=1, c= - 10. NOT SUFF
(T) Either each of {a,b,c} or one of {a,b,c} is positive.
In the first case, b+c > a.
In the second case, either b or c is positive. If b>0, c+b > c > a
If c > 0, b+c > b > a SUFF

234
If a , b and c are positive integers, is a= 7 ?

(1) (a – 7) (b – 7) (c – 7) = 0
(2) bc = 18

(factors and multiples, algebra, medium)

(1) Either a, b or c is equal to 7. NOT SUFF


(2) a could be any real number, including 7 NOT SUFF
(T) Since 7 is not a factor of 18, neither b nor c is equal to 7, so a=7
SUFF

235
If ab ≠0, and points (-a,b) and (-b,a) are in the same quadrant of the xy-
plane, is point (-x,y) in the same quadrant?

(1) xy > 0
(2) ax > 0

(coordinate geometry, hard)

Two points are in the same quadrant if and only if their x coordinates have the
same sign and their y-coordinates have the same sign. If (-a,b) and (-b,a) are
in the same quadrant, a and b have the same sign and (-a,b) must be in either
quadrant II or quadrant IV.

(1) xy>0 tells us that x and y have the same sign, so (-x,y) is in either
quadrant II or quadrant IV. The point in question is in the same quadrant as is
(-a,b) if a and x have the same sign, but it may be that a and x have different
signs, in which case the point in question would be in the same quadrant as
(a,-b), not (-a,b). NOT SUFF

(2) a and x have the same sign, but nothing is said about the sign of y.

(1) and (2) SUFFICIENT

236
If a, b, k and m are positive integers, is ak a factor of bm?

(1) a is a factor of b.
(2) k≤m

(factors and multiples, hard)

(1) NOT SUFFICIENT. Note that 2 is a factor of 62, and 22 is a factor of 62,
but 23 is not a factor of 62.

(2) NOT SUFFICIENT. Note that 2 is a factor of 62, but not of 52.

(1) and (2) SUFFICIENT. (1) tells us that b is a positive multiple of a, so we can
say that b=xa, where x is a positive integer.

b m a m x m m m k
Thus k  k  x a , which is an integer. Thus ak is a factor of bm.
a a

237
If a > 0, b > 0 and c > 0, is a(b  c)  0 ?

(1) bc  c b
b c
(2) 
c b

(algebra, hard)

Note that a(b – c )=0 if b – c =0, i.e. b = c

(1) b – c = c – b implies that 2b=2c and thus b=c. SUFF


(2) This implies that b2 = c2, and as b and c are positive, we can deduce that
b=c. SUFF

238
If a < y < z < b, is | y – a | < | y – b | ?

(1) |z–a|<|z–b|
(2) |y–a|<|z–b|

(algebra, hard)

The question can be rephrased as follows: Is y closer to a than to b ? It


would be sufficient to acertain that y < (a+b)/2.

(1) tells us that z is closer to a than to b. In other words, z is less than


(a+b)/2. As we know that y is less than z, it follows that y < (a+b)/2.
SUFF
(2) Note that the distance between y and b is less than that between z and
b, in other words |y – b| > | z –b |. Thus the statement |y –a| < |z – b|
tells us that |y – a|< |y – b|. SUFF

239
If a certain coin is flipped, the probability that the coin will land heads is 1/2. If
the coin is flipped 5 times, what is the probability that it will land heads up on
the first 3 flips and not on the last 2 flips?

3/5 1/2 1/5 1/8 1/32

(probability, medium)

For each flip, the probability that the coin lands heads up is ½ and thus
probability that the coin wil not land heads up is ½. Thus Pr (HHHTT)=(1/2)5 =
1/32.

3/5 1/2 1/5 1/8 1/32


240
If a certain machine produces bolts at a constant rate, how many seconds will it
take the machine to produce 300 bolts?

(1) It takes the machine 56 seconds to produce 40 bolts.


(2) It takes the machine 1.4 seconds to produce 1 bolt.

(ratios, medium)

Any information that gives us the rate at which the machine produces bolts
would allow us to answer the question.

(1) SUFF
(2) SUFF

241
If a certain sample of data has a mean of 20.0 and a standard deviation of 3.0,
which of the following values is more than 2.5 standard deviations from the
mean?

12 13.5 17 23.5 26.5

(statistics, hard)

If x is more than 3 standard devations above the mean, x >20 +3(2.5)=27.5


If x is more than 3 standard deviations below the mean x < 20 – 3(2.5)=12.5

None of these values are 3 standard deviations above the mean, however 12 is
more than 3 standard deviations below the mean.

12 13.5 17 23.5 26.5

242
If a code word is defined to be a sequence of different letters chosen from the
10 letters A, B, C, D, E, F, G, H, I, and J, what is the ratio of the number of 5-
letter code words to the number of 4-letter code words?

5 to 4 3 to 2 2 to 1 5 to 1 6 to 1

(combinatronics, hard)

Note that ABCD and DCBA are different 4-letter code words. The number of 5-
letter code words is , the number of 4-
letter code words.

5 to 4 3 to 2 2 to 1 5 to 1 6 to 1

243
If a committee of 3 people is to be selected from among 5 married couples so
that the committee does not include two people who are married to each other,
how many such committees are possible?

20 40 50 80 120

(combinatronics, hard)

One way to count such committees is to count them directly: three of the five
couples must have a member on the committee, and there are 5 C3 =10 ways of
choosing these couples. For each of the 3 couples chosen, there are two ways
to decide which member will sit on the committee, so the total number of
committees is 10 x 2 x 2 x 2= 80.

Another way is to count the number of 3-member committees that could be


formed without any restrictions, 10 C3 =120, and subtract from this the number
of committees in which two of the members are married to each other, 40:
(There are 5 ways to choose the couple that will sit on the committee, and 8
ways to choose the third person.)

20 40 50 80 120

244
If an automobile averaged 22.5 miles per gallon of gasoline, approximately how
many kilometers per liter of gasoline did the automobile average? (1 mile = 1.6
kilometers and 1 gallon = 3.8 liters, both rounded to the nearest tenth.)

3.7 9.5 31.4 53.4 136.8

(ratios, medium)

22.5 miles/gallon x (1 gallon/3.8 liters) x (1.6 kilometers/1 mile)


= (22.5 x 1.6)/3.8 kilometers/liter , which is not far from 10 km per liter

3.7 9.5 31.4 53.4 136.8

245
If an integer n is to be chosen at random from the integers 1 to 96, inclusive,
what is the probability that n(n + 1)(n + 2) will be divisible by 8?

1/4 3/8 1/2 5/8 3/4

(probability, factors and multiples, hard)

The product will be a multiple of 8 if and only if n is even or n + 1 is a multiple


of 8. Half of the 96 equally probable values of n are even, and 12 are 1 less
than a multiple of 8. Thus the probability that the product is divisible by 8 is 1/2
+ 1/8 = 5/8.

1/4 3/8 1/2 5/8 ¾

246
If Ann saves x dollars each week and Beth saves y dollars each week, what is
the total amount that they save per week?

(1) Beth saves $5 more per week than Ann saves per week.
(2) It takes Ann 6 weeks to save the same amount that Beth saves in 5
weeks.

We are asked for the value of x + y .

(1) y = x + 5, so x + y = 2x + 5 NOT SUFF


(2) 6x = 5y, so x + y = x +6x/5 NOT SUFF
(T) With two linear equations involving x and y, we can find the value of
each variable and this the value of x + y. SUFF

247
If a positive decimal d has 7 nonzero digits, is d > 100 ?

(1) d has 3 nonzero digits to the right of the decimal point.


(2) d has more than 3 nonzero digits to the left of the decimal point.

(exponents, medium)

(1) d has 7 – 3 = 4 nonzero digits to the left of the decimal point, so d is


greater than 1111 SUFF
(2) d has at least 4 nonzero digits to the left of the decimal point, so d is at
greater than 1111 SUFF

248
If ax + b = 0, is x > 0 ?

(1) a+b>0
(2) a–b>0

(inequalities, hard)

Note that we can express x in terms of a and b: x= - b/a. x will be positive if


and only if a and b have different signs.

(1) If the sum of a and b is positive, it could be that a and b are both
positive, and it could be that a and b have different signs.
INSUFFICIENT
(2) tells us that a > b , which gives us no information about the signs
of a and b.
(1) and (2) a and b could both be positive, or a could be positive and b
negative. NOT SUFFICIENT

Example: 3x + 2 =0 yields x= -2/3


3x – 2 =0 yields x= 2/3

249
If a quality control check is made inspecting a sample of 2 light bulbs from a
box of 12 light bulbs, how many different samples can be chosen?

6 24 36 66 72

(combinatronics, medium)

If the order in which the light bulbs are chosen were important, there would 12
ways to choose the first light bulb and 11 ways of choosing the second light
bulb, for a total of 12 x 11 different samples. However, as the order is not
important, this figure counts the each of the different samples twice. Therefore,
there are (12 x 11)/2 = 66 samples.

6 24 36 66 72

250
If a1 = 1 and an = 1 + 1/an-1 for all n ≥ 1, what is the value of a5 ?

3/8 5/8 8/5 5/3 8/3

(sequences, medium)

Note a2=2. a3 = 1 + 1/2 =3/2, a4=5/3 and a5 = 1 + 1/(5/3) = 1 +3/5 =8/5

3/8 5/8 8/5 5/3 8/3

251
If Bob produces 36 or fewer items in a week, he is paid x dollars per item. If
Bob produces more than 36 items in a week, he is paid x dollars per item for
the first 36 items and 1 1/2 times that amount for each additional item. How
many items did Bob produce last week?

(1) Last week Bob was paid a total of $480 for the items that he produced
last week.
(2) This week Bob produced 2 items more than last week and was paid a
total of $510 for the items that he produced this week.

(algebra, hard)

Bob’s salary as a function of the p, the number of items Bob produced last
week is the greater of the two quantities xp and 36x + 1.5x (p – 36).
Note that if p > 36, the second quantity is greater, otherwise xp would be last
week’s pay.

(1) Bob was paid $480 last week. It could be that xp=480 or
36x +1.5x(p - 36)=480. In either case, p cannot be found.
INSUFF
(2) We don’t know whether p + 2 is greater than 36, so we can’t say
which of the two expressions to use.
INSUFF

(T) The difference is pay is $30. If p is no greater than 34, the difference in
pay is 2x. If p=35, the difference in pay is x + 3x/2= 5x/2. If p is at least
36, the difference in pay is 3x. Thus x could be 15, 12 or 10. If we don’t
know the value of x, we can’t find the value of p: if x=15 , p would be
32, whereas if x=10, p would be 44. NOT SUFF

252
If c and d are integers, is c even?

(1) c (d + 1) is even.
(2) (c + 2)(d + 4) is even.

(factors and multiples, hard)

(1) At least one of c and d+1 is even. If d is even, c must be even as well, but
if d is odd, c could be odd. NOT SUFFICIENT

(2) Either c +2 or d+4 is even. If d is odd, so is d+4, implying that c+2 and
hence c is even. If d is even, c could be odd. NOT SUFFICIENT

(1) and (2) According to (2), if d is odd, c is even. According to (1), if d is


even, so is c. SUFFICIENT

253
If Company M ordered a total of 50 computers and printers and Company N
ordered a total of 60 computers and printers, how many computers did
Company M order?

(1) Companies M and N ordered the same number of computers.


(2) Company N ordered 10 more printers than did Company M.

(algebra, medium)

We are told that pm + cm = 50 and pn + cn = 60 and asked to find the value of


cm.

(1) cm = cn NOT SUFF


(2) Tells us the same as (1) NOT SUFF
(T) NOT SUFF

254
If d is a positive integer and f is the product of the first 30 positive integers
what is the value of d?

(1) 10d is a factor of f.


(2) d>6

(factors and multiples, hard)

Note that 30! is a product that involves 6 terms that are multiples of 5, one of
which is 52. Thus 30! is a multiple of 57 but not of 58. As all the even terms of
this product are multiples of 2, the product is a multiple of 27. We can say, then
that 30! is a multiple of 107, but not of 108. Only together are (1) and (2)
sufficient to conclude that d=7.
(T) SUFF

255
If d is the standard deviation x, y and z, what is the standard deviation of x+5,
y+5 and z+5 ?

d 3d 15d d+5 d+15

(statistics, hard)

Remember that the standard deviation measures the average distance from
each element to the mean of the elements. As 5 is added to each term, the
average increases by 5 and the distances from the elements to the mean
remain unaltered. Thus the standard deviation will not change.

d 3d 15d d+5 d+15

256
If each term in a sum a1, a2, a3,…,an is either 7 or 77 and the sum equals 350,
which of the following could be the value of n?

38 39 40 41 42

(sequences, factors and multiples, hard)

If there are x terms that are equal to 7 and y terms equal to 77, 7x+77y=350.
Thus 7(x+11y)=7(50) and x+11y=50. Solving for x, x=50-11y. If y=1, x=39,
so n=x+y=40. In general, x + y = 50 -10y, a multiple of 10.

38 39 40 41 42

257
(100∂ +10∆ + ¥) ÷ 5 = 10Φ + ¥

If ∂, ∆, ≦, Φ are all positive integers less than 10 that satisfy the above
equation, what is the value of Φ?

(1) ∂=∆-6
(2) ∂ + ∆+ ≦ = 13

(factors and multiples, hard)

Note that 100∂ +10∆ + ¥ is a multiple of 5, so ¥, a positive integer, must be 5.


Thus 100∂ +10∆ + ≦ = 5(10Φ + 5) is a multiple of 25. It follows that ∆ is either
2 or 7, as every multiple of 25 that does not end in 0 must end in 25 or 75.

(1) As ∂ = ∆ - 6 > 0, ∆ must be 7 and ∂ must be 1.


Thus 100∂ +10∆ + ¥=175, and the value of Φ can be found. SUFF

(2) ∂ = 13 - ∆ - ¥ = 8 - ∆. If ∆ = 2, ∂ = 6. If ∆ = 7, ∂ = 1. However, as
10Φ + 5 < 100, 100∂ +10∆ + ≦ = 5 (10Φ + 5) < 500.
Thus ∂ < 5, so ∂ = 1
Thus 100∂ +10∆ + ¥=175, and the value of Φ can be found. SUFF

258
If eleven consecutive integers are listed from least to greatest, what is the
average (arithmetic mean) of the eleven integers?

(1) The average of the first nine integers is 7.


(2) The average of the last nine integers is 9.

(statistics, medium)

Suppose the first integer is this list is n. To find the average of these eleven
consecutive integers, it is sufficient to find the value of n, as the average will be
(n+(n+10))/2.

(1) The average of the first nine integers is (n+(n+8))/2. As were are told
that this is equal to 7, we can find n and then answer the question. SUFF

(2) The average of the first last integers is ((n+2)+(n+10))/2. As were are
told that this is equal to 9, we can find n and then answer the question. SUFF

259
If 500 is the multiple of 100 that is closest to x and 400 is the multiple of 100
that is closest to y, then which multiple of 100 is closest to x + y ?

(1) x < 500

(2) y < 400

(inequalities, medium)

We are told that 450 < x < 550 and that 350 < y < 450

(T) 450 < x < 500 and 350 < y < 400. Thus 800 < x + y < 900.
NOT SUFF

260
If 521 x 411= 2 x 10n, what is the value of n?

11 21 22 23 32

(exponents, medium)

Note that 411= (22)11. Thus 521 x 411=2 x 521 x 221 = 2 x 1021.

11 21 22 23 32

261
If 400(6,000) = 240 × 100x , what is the value of x?

5 4 3 2 1
(exponents, medium)

The product on the left is 24 followed by 5 zeros, whereas the product on the
right is 24 followed by 2x + 1 zeros. Thus x = 2.

5 4 3 2 1

262
If 4x < x < x3 < x2, which of the following is a possible value of x ?

–2 -3/2 –1 -1/2 0

(inequalities, medium)

Since x cannot be 0, we must choose among negative numbers. If x < x3,


and x < 0, x2 < 1 and thus -1 < x < 0. Thus choose -1/2

–2 -3/2 –1 -1/2 0

263
For all real numbers x, [x] is defined as the smallest integer that is greater than
2x. Is [2x] = 0 ?

(1) [x] = 0
(2) [3x] = 0

(functions, medium)

(1) -1 ≤ x < 0. Thus -2 ≤ 2x < 0. Thus [2x] could be -1 or 0. NOT SUFF


(2) -1 ≤ 3x < 0. Thus -1/3 ≤ x < 0 and -2/3 ≤ 2x < 0. Thus [2x] = 0. SUFF

264
If b is positive, is ab positive?

(1) a2b > 0


(2) a2 + b = 13

(algebra, medium)

If b > 0 , ab > 0 if and only if a > 0. Thus we need to know whether a > 0.

(1) That a2b > 0 tells us that a2 > 0. Thus a ≠ 0. NOT SUFF
(2) a2 = 13 – b, so a =± NOT SUFF
(T) NOT SUFF

265
x x-3 y
If 5 – 5 = (124)(5 ), what is y in terms of x?

x x-6 x-3 2x + 3 2x + 6

(exponents, hard)

, so y = x – 3.
Another way to get to the answer is to choose a value of x (3 works
wonderfully)

x x-6 x-3 2x + 3 2x + 6

266
If M is a finite set of negative integers, is the total number of integers in M an
odd number?

(1) The product of all the numbers in M is odd.


(2) The product of all the integers in M is negative.

(algebra, medium)

(1) All of the integers are odd, but we don’t know how many there are.
NOT SUFF
(2) If there were an even number of integers, the product would be positive:
we could divide the 2k integers into k pairs, and the product of each pair
would be positive, and would the product of each pair. SUFF

267
If the integer n is greater than 1, is n equal to 2?

(1) n has exactly 2 positive factors.


(2) The difference between any two distinct positive factors of n is odd.

(factors and multiples, hard)

(1) n has exactly 2 positive factors if and only if n is a prime number. Thus
n could equal to 2 or any other prime number. NOT SUFF
(2) If n is an even number greater than 2, 2 is a factor of n but n – 2 is
even. If n is an odd number greater than 2, 1 is a factor of n, but n – 1
is even. Therefore n must be 2. SUFF

268
If it takes 12 workers working independently 3 hours to make 500 boxes, how
many minutes should it take for 16 workers to make 300 boxes?

60 72 81 98 240

It is useful to think in terms of worker-hours. A total of 36 hours are needed to


make 500 boxes, so to make 300 = 500(3/5) boxes, a total of
36(3/5)=108/5=216/10=hours. As each tenth of an hour is 6 minutes, 216 x 6
minutes of work are needed. Dividing by 16, the number of workers, we get
the number of minutes per worker: 216 x 6/16= 216 x 3/8= 27 x 3= 81.

Alternatively, to produce 3/5 as many boxes with 4/3 as many workers, instead
of 180 minutes, it would take 180 ×3/5 ×3/4 = 81 minutes.

60 72 81 98 240

269
If it took Carlos ½ hour to cycle from his house to the library yesterday, was
the distance that he cycled greater than 6 miles? (1 mile = 5,280 feet)

(1) The average speed at which Carlos cycled from his house to the library
yesterday was greater than 16 feet per second.
(2) The average speed at which Carlos cycled from his house to the library
yesterday was less than 18 feet per second.

(movement, ratios, hard)

After looking at the nature of the information given in the statements, we can
rephrase the question as follows: was Carlos’s average speed greater than 12
miles per hour?

12.miles 5280. feet 1.hour 12  5280


( )( ) feet/second. Simpliying, we see
1.hour 1.mile 3600.sec onds 3600
that expressed in feet per second, 12 miles per hour corresponds to 17.6
feet/second. Thus not even the combination of (1) and (2) is sufficient.

(T) NOT SUFF

270
If J, S and V are points on the number line, what is the distance between S and
V?

(1) The distance between J and S is 20.


(2) The distance between J and V is 25.

(algebra, medium)

(T) Suppose that J=0, (1) says that |S|=20 and (2) says that |V| = 25.
Therefore the distance between S and V is 5 if they are on the same side
of J (in this case 0)and is 45 if they are on opposite sides of J.
NOT SUFF

271
If k is an integer and (0.0025)(0.025)(0.00025)×10k is an integer, what is the
least possible value of k?

-12 -6 0 6 12

(exponents, factors and multiples, hard)

Let n = (0.0025)(0.025)(0.00025)×10k . n = 253 × 10-4 × 10-3 × 10-5 ×10k


Therefore n = 253 × 1012-k = 2515-k × 212-k is an integer. Since 25 is not a
multiple of 2, the power of 2 must be at least 0. It follows that the least
possible value of k is 12.

-12 -6 0 6 12
272
If k is a positive integer, what is the remainder when 134k+2 + 8 is divided by
10?

7 4 2 1 0

(factors and multiples, hard)

We need to determine the units digit of 134k+2 + 8: this will be the remainder
when the above sum is divided by 10.

132(2k+1) + 8 = 169 × (169)2k + 8


Since the square of 169 has a units digit of 1, so does the units digit of (169)2k.
Thus the units digit of the above sum is the same as the units digit of 9 x 1 +
8, i.e. 7.

7 4 2 1 0

273
If k is a positive 3-digit number, what is the hundreds digit of k?

(1) The hundreds digit of k +150 is 4.


(2) The tens digit of k +25 is 7.

(exponents, inequalities, hard)

(1) 400 ≤ k + 150 < 500 , so 250 ≤ k < 350. Thus the hundreds digit of k
could be 2 or 3. NOT SUFF
(2) Obviously not sufficient. NOT SUFF
(T) k could be 250,251,..254,345,346,..349 NOT SUFF

274
If k is not equal to 0, 1 or -1, then is 1/k > 0 ?

(1) The reciprocal of k-1 is positive.


(2) The reciprocal of k+1 is positive.

(inequalities, medium)

1/k > 0 if and only if k > 0. Note that a number and its reciprocal are always
the same sign.

(1) k – 1 >0 so k >1. SUFFICIENT


(2) k + 1 >0, so k > -1. NOT SUFFICIENT

275
If k, m, and p are integers, is k – m – p odd?

(1) k and m are even and p is odd.


(2) k, m, and p are consecutive integers.

(factors and multiples, medium)

(1) Since k and m are even, so is k – m.


Since p is odd, k – m – p = (k – m) - p is odd. SUFF
(2) Since k m and p are consecutive integers, they can be written as x- 1, x
and x + 1 (not necessarily in that order). Thus their sum is a multiple of
3, and thus may be odd or even. NOT SUFF

276
If k, m, and t are positive integers and k/6 + m/4 = t/12 , do t and 12 have a
common factor greater than 1 ?

(1) k is a multiple of 3.
(2) m is a multiple of 3.

(factors and multiples, hard)

Note that k/6 + m/4 = (2k+3m)/12. Thus t = 2k + 3m.

(1) If k is a multiple of 3, t is the sum of multiple of 3 and is thus a multiple


of 3. Thus t and 12 have a common factor of 3. SUFF
(2) If m is a multiple of 3, we know that t is the sum of an even number and
a multiple of 9. The value of t could be 6 + 9= 15, a multiple of 3.
Alternatively, t could be 4 + 9= 13, which is a prime number.
NOT SUFFICIENT

277
If m and n are positive integers and mn=k is m + n = k +1?

(1) m=1.
(2) k is a prime number.

(factors and multiples, medium)

(1) If m=1, n=k, so m + n= 1 + k = k +1. SUFF


(2) If k is a prime number, k has only two positive factors, 1 and k. Thus
m=1 and n=k or m=k and n=1. In either case, m+n= k +1. SUFF

278
If m and n are positive integers, is m > n ?

(1) m+n=g
(2) mn = 20

(inequalities, medium)

(1) tells us absolutely nothing NOT SUFF


(2) m could be 1 and n could be 20 or vice versa NOT SUFF
(T) NOT SUFF

279
If n and p are integers, is p > 0?

(1) n+1 > 0


(2) np > 0

(inequalities, hard)

(1) Tells us that n is not negative, but reveals nothing about the value of p.
NOT SUFF
(2) Tells us that n and p are either both positive or both negative.
NOT SUFF
(T) Since (1) tells us n cannot be negative, n and thus p must be positive.
SUFF

280
If m and r are two numbers on a number line, what is the value of r?

(1) The distance between r and 0 is 3 times the distance between m and 0.
(2) 12 is halfway between m and r.

(algebra, hard)

(1) |r| = 3 |m| NOT SUFFICIENT


(2) The mean of m and r is 12. Thus m + r= 24. NOT SUFF

(T) Clearly r and m could be both positive, in which case r = 3m, so


r=18. However, m could be negative and r positive, so that r= -
3m, in which case m = -12 and r=36. NOT SUFF

281
If m is a positive odd integer between 2 and 30, then m is divisible by how
many different positive prime numbers?

(1) m is not divisible by 3


(2) m is not divisible by 5

(factors and multiples, hard)

(1) If m is not divisible by 3, then the smallest prime factor of m is 5.


Since 5 × 7 > 30, m has one only prime factor, as m is not divisible by 2.
SUFF

(2) If m is not divisible by 5, m could be 21, the product of two prime


numbers, or a prime number between 2 and 30. NOT SUFF

282
If m is a positive odd integer, what is the average (arithmetic mean) of a
certain set of m integers?

(1) The integers is the set are consecutive multiples of 3.


(2) The median of the set of integers is 33.

(statistics, medium)

(1) tells us that the set of m integers form an arithmetic sequence, so that
average is equal to the median, but we are not told about the actual
values of the integers. NOT SUFF
(2) The median of a set may be equal to, greater than, or less than the
average of that set. NOT SUFF
(T) SUFF

283
If m is a three-digit positive odd integer such that the sum of its digits is 11,
what is the value of m?

(1) The hundreds digit of m is 9.


(2) The tens digit of m is 1.

(factors and multiples, medium)

(1) The sum of the tens and units digits must be 11-9=2. Since m is odd, its
units digit must be odd. Thus the tens and units digits of m are both 1.
As we know all three digits of m, we can find the value of m. SUFF
(2) The sum of the hundreds and units digits of m is 11-1= 10. The
hundreds and units digits of m could be 9 and 1 or vice versa, among
other possibilities. NOT SUFF

284
If M= 4 3 4 4 4, then the value of M is

Less than 3 equal to 3 between 3 and 4 equal to 4 greater than 4

(roots, medium)

Note that 42 and 1 3 4  2 and 1 4 4  2 . Thus 4 < M < 6.

Less than 3 equal to 3 between 3 and 4 equal to 4 greater than 4

285
If m > 0 and n > 0, is (m + x )/(n + x) > m/n ?

(1) m>n
(2) x=1

(ratios, inequalities, hard)

Rephrasing the question, is nm + nx > mn + mx ?

i.e. Is (n-m)x > 0?

(1) tells us that n-m is negative, but no information is given about x.


NOT SUFF
(2) We know that x=1, but no information is given about n – m.
NOT SUFF
(1) and (2) SUFFICIENT

286
If mv < pv < 0, is v > 0?

(1) m<p
(2) m<0

(inequalities, hard)

We know that from the question that mv – pv = v( m – p) is negative.

(1) m<p i.e. m – p <0. Since v(m – p ) <0, p>0 SUFF


(2) As m<0 and mv<0, v>0 SUFF

287
If m and n are distinct positive integers, what is the remainder when 34n+2 + m
is divided by 10?

(1) n=2
(2) m=1

(exponents, factors and multiples, hard)

Note that the remainder when the sum in question is divided by 10 is equal to
the units digit of the sum.

(1) If n=2, we can find the units digit of 34n+2, but no information is given
about the value of m. NOT SUFF
(2) If m=1, we only need to find the units digit of 34n+2.
34n+2= 32(34)n , and as the units digit of 34=81 is 1,
the units digit of (34)n is 1,we can find the units digit of the sum. SUFF

288
If m is a positive odd integer, what is the average (arithmetic mean) of a
certain set of m integers?

(1) The integers in the set are consecutive multiples of 3.


(2) The median of the set of integers is 33.

(sequences, statistics, hard)

(1) As (1) gives no information about the values of these multiples, there is
no way to calculate the arithmetic mean. We do know however, that the
mean of the integers will be equal to the median of these integers.
NOT SUFF

(2) The average of the set of integers may be less than, greater than or
equal to the median of the set of integers.
NOT SUFF

(T) The average of the set of integers is equal to the median of the set of
integers, which is 33. SUFF

One could think that there are 2k + 1 integers, where k is an integer.


k of these integers are less than 33, 1 is equal to 33, and the remaining
k integers are greater than 33:

The integers are as follows:


33 – 3k, 33 – 3(k – 1), ... , 33 – 3 , 33 , 33 + 3, ... 33 +3(k – 1), 33 +3k

The sum of these 2k + 1 integers is 33(2k + 1) , thus the average of


these integers is 33.

289
If m, p, and t are all positive integers and m < p < t, is the product mpt an
even integer?

(1) t–p=p-m
(2) t – m = 16

(factors and multiples,hard)

(1) t – p = p - m implies that p= (t+m)/2


t, m and p could all be odd (t=5, m=1, p=3)
or p could be even (t=7, m=1, p=4) NOT SUFF

(2) t=m+ 16 NOT SUFF, as nothing is said about p, and as t and m could
both be odd, the product in question could be odd or even. NOT SUFF

(T) p=m+8
So pmt=m(m+8)(m+16)

If m is odd, so is m+8 and m+16, so pmt is odd


If m is even, pmt is even NOT SUFF

290
If m, r, x and y are positive numbers, is the ratio of m to r equal to the ratio of
x to y ?

m x
(1) 
y r

m x x
(2) 
ry y

(ratios, medium)

In other terms, is r x = m y ? or is r = m y / x ?

(1) m r = x y, so r = x y / m, which is equal to m y / x if and only if x / m =


m / x, i.e m =x. Since no information is given as to whether x = m, we
do not have enough information to answer the question. NOT SUFF
(2) (m + x ) y = x ( r + y ), so m y = x r SUFF

291
If m, w and c are numbers such that m=9/25 , w=15/32 and m + w + c = 1,
which of the following gives the values of m, w and c in increasing order?

c<m<w c<w<m m<w<c w<c<m


w<m<c

(ratios, medium)

m= 9/25 = 36/100=0.36 w= 15/32 = 1/2 – 1/32 ≈ 0.47


c = 1 – m – w ≈ 0.17

Thus c < m < w

c<m<w c<w<m m<w<c w<c<m


w<m<c

292
If n and k are positive integers, what is the remainder when 34n + 2 + k is
divided by 10?

(1) k=1
(2) n=5

(exponents, factors and multiples, hard)

34n + 2 = 32(2n+1) = 92n+1 , which has a units digit of 9, as 2n+1 is odd and 9 to
any postive odd exponent has a units digit of 9. Thus the units digit of the
above sum depends solely on the value of k.

(1) SUFF
(2) NOT SUFF

293
If n and p are intergers, is p > 0 ?

(1) n+1>0
(2) np > 0

(inequalties, medium)

(1) We can conclude that n > -1 and thus is not negative, but no
information is given about the value of p. NOT SUFF
(2) That np > 0 implies that n and p are either both positive or both
negative, so p can be either positive or negative. NOT SUFF

(T) As (1) tells us that n is not negative, and (2) tells us that n is
either positive or negative, so it is clear that n is positive. As (2)
tells us that n and p have the same sign, p is also positive.

294
If n and y are positive integers and 450y= n3, which of the following must be
an integer?
y y y
I. II. III.
3  22  5 32  2  5 3  2  52
None I only II only III only I, II and III

(factors and multiples, hard)

We are told that 450y is a perfect cube, so the power of each prime factor of
450n is a multiple of 3. As 450= 2 x 32 x 52, it must be that y is a multiple of 22,
3 and 5. Thus only I must be true.

None I only II only III only I, II and III

295
If n is an integer and xn – x-n = 0, what is the value of x ?

(1) x is an integer.
(2) n≠0

(exponents, hard)

Note that xn – x-n = 0 if and only if x2n – 1 = 0 (multiply both sides by xn )

(1) x can be either 1, - 1 or any integer at all if n=0 NOT SUFF


(2) x can be either -1 or 1. NOT SUFF
(T) x can be either -1 or 1. NOT SUFF

296
If n is an integer greater than 6, which of the following must be divisible by 3?

n (n+2) (n+3) n (n+5) (n-3) n (n+2) (n+5) n (n+1) (n-4)


n (n+1) (n-2)

(factors and multiples, hard)

Note that n – 1 will have the same remainder when divided by 3 as n – 4 , so


n(n+1)(n-4) will have the same remainder as n(n+1)(n-1). Since n – 1 , n and
n+1 are three consecutive integers, one of them will be a multiple of 3 and the
remainder when n(n+1)(n – 1) and thus n(n + 1)(n – 4) are divided by 3 is 0.
This means that n(n + 1)(n – 4) is divisible by 3.

n (n+2) (n+3) n (n+5) (n-3) n (n+2) (n+5) n (n+1) (n-4)


n (n+1) (n-2)

297
If the number 52,1n9, where n represents the tens digit, is a multiple of 3, then
the value of n could be which of the following?

6 5 3 1 0

(factors and multiples, medium)

It certainly helps to know that an integer is a multiple of 3 if and only if the sum
of its digits is a multiple of 3. In this case 17 + n must be a multiple of 3, so n
could be 1, 4 or 7. Alternatively, 51,000 is a multiple of 3, as is 999 and 120.
Thus the sum of these numbers, 52,119 is a multiple of 3.

298
If n is a positive integer, is the units digit of n equal to 0 ?

(1) 14 and 35 are factors of n.


(2) n= (25)(32)(57)(76)

(factors and multiple, medium)

(1) As 14 is a multiple of 2 and 35 is a multiple of 5, 2 and 5 are factors of


n, i.e. n is a multiple of 5 x 2= 10. Thus the units digit of n is 0. SUFF
(2) As the value of n is given, (2) is sufficient. SUFF

299
If n is a positive integer and r is the remainder when (n-1)(n+1) is divided by
24, what is the value of r?

(1) n is not divisible by 2.


(2) n is not divisible by 3.

(factors and multiples, hard)

(1) As n is odd, n – 1 and n + 1 are consecutive even numbers, one


of them will be a multiple of 4. Thus (n - 1)(n + 1) is a multiple of
8, and the remainder when this product is divided by 24 must be
either 0, 8 or 16. NOT SUFF
(2) As n is not a multiple of 3, either n – 1 or n + 1 is a mutiple of 3.
If n is odd, (n -1)(n+1) is also a multiple of 8 and thus a multiple
of 24. In that case, r=0. Conversely, if n is even (n-1)(n+1) is the
product of two odd numbers and will thus be an odd number, and
not a multiple of 24. Thus r would be a positive integer.
NOT SUFF

(T) (1) tells us that n is odd, so (2) tells us that r=0.

300
If n is a positive integer and the product of all the integers from 1 to n,
inclusive, is a multiple of 990, what is the least possible value of n?

10 11 12 13 14

(factors and multiples, hard)

As 990= 2 x 32 x 5 x 11, n! is a multiple of 990 if and only if it is a multiple of


11, a prime number. If n < 11, n! will not be a multiple of 11 and thus will not
be a multiple of 990. 11! is a multiple of 11, 2 , 9 and 5, and thus is a multiple
of 990. Thus the least possible value of n is 11.

10 11 12 13 14

301
If n is a positive integer and r is the remainder when n2 - 1 is divided by 8, what
is the value of r?

(1) n is odd.
(2) n is not divisible by 8.

(factors and multiples, hard)

Note that n2 – 1= (n + 1)(n – 1).

(1) If n is odd, n – 1 and n + 1 are two consecutive even numbers, one of


which will be a multiple of 4. Thus the product of the two factors will be
a multiple of 8 and r=0. SUFF
(2) Very little is said about n, so looking at two examples (n=1 and n=2) will
show that r will vary as does n. NOT SUFF

302
If n is a positive integer and r is the remainder when 4+7n is divided by
3, what is the value of r ?

(1) n + 1 is a multiple of 3.
(2) n > 20.

(factors and multiples, medium)

(1) If n + 1 is a multiple of 3, so is 7(n+1) = 7n + 7 and 7n + 7 – 3 = 7n +


4. Since 7n + 4 is a multiple of 3, r = 0. SUFF
(2) Consecutive possible values of 4 + 7n will differ by 7, which is not a
multiple of 3. Thus the values of r will differ. NOT SUFF

303
If n is a positive integer less than 200 and 14n is an integer, then n has how
60
many different prime factors?

Two Three Five Six Eight

(factors and multiples, hard)

14n 7n
 is an integer, and since 7 is a prime number, n must be a multiple of
60 30
30= 2 x 3 x 5. Thus n= 30k for some positive integer k. As n < 200, k <
200/30 and thus k is at most 6. Whatever the value of k, n will have exactly
three prime factors, 2 3 and 5.

Two Three Five Six Eight

304
If n is a multiple of 5 and n = p q , where p and q are prime numbers, which of
2

the following must be a multiple of 25?

p2 q2 pq p 2q 2 p 3q

(factors and multiples, hard)

If n is a multiple of 5, either p or q is a multliple of 5 and thus pq is a multiple


of 5. (pq)2 = p2q2 must be a multiple of 25. Looking at (p,q) =(2,5) and (p,q)
=(5,2) will show that none of the other choices need be a multiple of 25.

p2 q2 pq p 2q 2 p 3q

305
If n denotes a number to the left of 0 on the number line such that the square
of n is less than 1/100, then the reciprocal of n must be

less than -10 between -1 and -1/10 between -1/10 and 0


between 0 and -1/10 greater than 10

(inequalities, hard)

If n < 0 and n^2 < 1/100, 1/n^2 > 100 and 1/|n| > 10

Since n < 0, |n| = - n and - 1/ n > 10 so 1/n < -10

less than -10 between -1 and -1/10 between -1/10 and 0


between 0 and -1/10 greater than 10

306
If n is a positive integer and xn – x-n = 0 , what is the value of x?

(1) x is an integer
(2) n≠0

(exponents, medium)

We are told that xn – 1/xn = 0 , so (x2n – 1)/xn = 0.


This means that x2n = 1.

(T) As 2n is an positive even integer, x can be either 1 or – 1. NOT SUFF

307
If n is a positive integer, what is the remainder when 38n+3 + 2 is divided by 5?

0 1 2 3 4

(factors and multiples, hard)

38n+3 = 33 x 94n. Note that when 9 is raised to a positive even power, the units
digit will always be 1. As the units digit of 33 is 7, the units digit of the product
will be 7. Therefore the units digit of 38n+3 + 2 is 9, making this sum 4 greater
than a multiple of 5.

0 1 2 3 4
308
If n is a positive integer, which of the following must be an even integer?

n+1 n+2 n2 + 2 n2 + n n2 + 2n

(factors and multiples, medium)

Note that n2 + n = n ( n + 1 ), the product of two consecutive integers, one of


which must be even, making the product even.

n+1 n+2 n2 + 2 n2 + n n2 + 2n

309
If n is a three-digit positive integer, what is the sum of the digits of n?

(1) The hundreds digit of n is 3 times the units digit.


(2) The hundreds digit of n is 3 more than the tens digit.

(algebra, medium)

(T) Neither statement by itself is sufficient, as information about all three


digits is needed. If the units digit is x, (1) tells us that the hundreds digit
is 3x, and (2) tells that the tens digit is 3x-3. n could be 301, 632 or
963. NOT SUFF

310
If n is the product of the integers from 1 to 20, what is the greatest integer k
for which 2k is a factor of n?

10 12 15 18 20

(factors and multiples, hard)

n = 20!. Considering only the even numbers from 1 to 20

16 is a multiple of 24
8 is a multiple of 23
4 ,12 and 20 are multiples of 22
2,6,10,14 and 18 are multiples of 2

Thus 20! is a multiple of 218

10 12 15 18 20

311
1
If n >0, which of the following is equal to ?
n 1  n

n 1
1 2n  1 n 1  n n 1  n
n

(exponents, medium)

The simplest way to clear the denominator of radicals is to multiply numerator


and denominator by the conjugate of the denominator, making use of the fact
thet (a – b)(a + b ) = a2 – b2.

1 n 1  n
  n 1  n
n 1  n (n  1)  n

1 2n  1
n 1
n 1  n n 1  n
n

312
If n > 0, which is greater, 20 percent of n or 10 percent of the sum of n and
0.5 ?

(1) n < 0.1


(2) n > 0.01

(inequalities, medium)

0.2n is greater than 0.1(n + 0.5)= 0.1n + 0.05 i.e n > 0.5 . If n < 0.5, the
opposite is true.

(1) SUFF
(2) NOT SUFF

313
If $1000 is deposited in a certain bank account and remains in the account
along with any accumulated interest, the dollar amount of interest I earned by
the deposit in the first n years is given by the formula I= 1000[(1+r/100)n-1]
where r is the annual rate of interest paid by the bank. Is the annual interest
rate paid by the bank is greater than 8 % ?

(1) The deposit earns a total of $210 in interest in the first two years.
(2) (1+r/100)2 > 1.15

(interest, hard)

r 2
(1) (1  )  1  210 / 1000 . Thus the value of r can be calculated. SUFF
100
(2) Note that (1 + r/100)2= 1 + r/50 + r2/10000 > 1 + r/50.
1 + r/50 > 1.15 if r/50 > 0.15 i.e r > 7.5.
Thus (2) is true for all values of r > 7.5, so r may or may not be greater
than 8. NOT SUFF

314
If P is the sum of positive odd numbers which are less than 84 and Q is the
sum of the positive even integers that are less than 84, what is the value of
Q–P?

- 42 - 40 40 41 42

(algebra, medium)

We can think of Q –P as (-1+2) + (-3 +4)+…(-81 +82) – 83 = 41 – 83 = -42

- 42 - 40 40 41 42

315
If p and n are positive integers and p > n, what is the remainder when p2 - n2
is divided by 15?

(1) The remainder when p+n is divided by 5 is 1.


(2) The remainder when p-n is divided by 3 is 1.

(factors and multiples, hard)

Note that p2 - n2 =(p + n)(p – n). Obviously neither (1) nor (2) is sufficient on
its own. Combining the two statements, (1) tells us that p + n is 1 more than a
multiple of 5, i.e. 5k + 1 for some integer k. (2) tells us that p – n is 3m + 1 for
some integer m. The product p2 - n2 =(p+n)(p – n)= 15mk + 3m + 5k + 1, so
the remainder when p2 - n2 is divided by 15 depends on m and k.

For example, if m=k=1, so that p – n = 4 and p + n = 6, corresponding to p=5


and n=1, the remainder is 9, but if m=k=2, so that p – n =7 and p + n =11,
corresponding to n=2 and p= 9, the remainder is question is 2.
(T) NOT SUFF

316
If p is a positive integer, what is the value of p?

(1) p/4 is a prime number.


(2) p is divisible by 3.

(factors and multiples, hard)

(1) p= 4k where k is a prime number. NOT SUFF


(2) p= 3m, where m is a positive integer NOT SUFF
(T) 4k = 3m, so m = 4k/3. Since k is a prime number and k is an integer,
k must be equal to 3 and p=12 SUFF

317
If p is a positive odd integer, what is the remainder when p is divided by 4?

(1) When p is divided by 8, the remainder is 5.


(2) p is the sum of the squares of two positive integers.

(factors and multiples, hard)

(1) p is 5 more than a multiple of 8, and as all multiples of 8 are multiples of


4, p is 5 more than a certain multiple of 4, and thus 1 more than the
next greater multiple of 4. Thus the remainder is 1. SUFF

(2) As p is odd and the sum of the squares of two integers, one of these
integers must be even and the other odd. Even integers are multiples of
2, so the square of any even integer is a multiple of 4. Thus the
remainder when p is odd will be the same as the remainder when the
square of the odd integer is divided by 4. Since each odd integer can be
expressed as k + 1, where k is an even integer, the square of each odd
integer can be expressed as (k+1)2 =k2+2k+1, the sum of two multiples
of 4 and 1. Thus the remainder when the square of any odd integer, and
thus p, is divided by 4 is 1. SUFF

318
If p is a prime number greater than 2, what is the value of p?

(1) There are a total of 100 prime numbers between 1 and p+1.
(2) There are a total of p prime numbers between 1 and 3,912.

(factors and multiples, medium)

(1) p is the 100th smallest prime number. SUFF


(2) All we would need to do to find the value of p is count the number of
prime numbers between 1 and 3,912. SUFF

319
If p,q r and s are as shown on the number line above, which of the following
products is greatest?

pq pr qr qs rs

(algebra, medium)

As there is only one positive number, choose the two negative numbers furthest
from 0: pq

pq pr qr qs rs

320
If p, r, s, and t are nonzero integers, is p/r = s/t ?

(1) s = 3p and t = 3r.


(2) 3p = 2r and 3s = 2t.

(ratios, medium)

p/r = s/t if and only if pt = rs and p/s = r/t.

(1) SUFF
(2) SUFF

321
If p,s, and t are positive prime numbers, what is the value of p3s3 t3 ?

(1) p3st = 728


(2) t = 13

(factors and multiples, medium)

If p,s and t are determined, we can find the value of p3s3 t3.

(1) Looking at the prime factorization of 728, we can identify its 3 prime
factors, the values of p,s and t. SUFF
(2) NOT SUFF

322
If positive integer x is a multiple of 6 and positive integer y is a multiple of 14,
is xy a multiple of 105 ?

(1) x is a multiple of 9.
(2) y is a multiple of 25.

Since 105 is 3 × 5 × 7, we need to know whether x or y is a multiple of 5.

(1) NOT SUFF


(2) SUFF

323
If q is a positive integer less than 17 and r is the remainder when 17 is divided
by q, what is the value of r?

(1) q > 10
(2) q = 2k, where k is a positive integer.

(factors and multiples, hard)

(1) Note that if q=11, r=6, whereas if q=12, r=5 NOT SUFF
(2) Since 17= 24+1, 17 is 1 greater than a multiple of q.
Thus the remainder r is equal to 1. SUFF

324
If p, r, s, and t are nonzero integers, is p/r = s/t ?

(1) s = 3p and t = 3r.


(2) 3p = 2r and 3s = 2t.

(ratios, medium)

Note that p/r = s/t if and only if p/s = r/t.

(1) s/p = t/r = 1/3, thus p/s =r/t SUFF


(2) p/r = s/t = 2/3 SUFF

325
If r and s are integers and rs + r is odd, which of the following must be even?

r s r+s rs-r r2 + s

(factors and multiples, medium)

Note that rs + r = r (s+1) is odd only when both r and s+1 are odd. s+1 is odd
only if s is even.

r s r+s rs-r r2 + s

326
If r is a constant and an = r × n for all positive integers n, for how many values
of n is an < 100 ?

(1) a50 = 500


(2) a100 + a105 = 2,050

(sequences, medium)

To answer the question , it is enough to acertain the value of r.

(1) a50 = 500 - r(50) = 500 SUFF


(2) r(100) + r(105) = 2050 SUFF

327
If r=0.345, s =(0.345)2 and t = , which of the following is the correct
ordering of r,s and t?

r<s<t r<t<s s<t<r s<r<t t<r<s

(exponents, medium)

If 0 < x < 1, xn decreases toward 0 as x increases.

r<s<t r<t<s s<t<r s<r<t t<r<s

328
If r is the remainder when positive integer n is divided by 7, what is the value
of r?

(1) When n is divided by 21, the remainder is an odd number.


(2) When n is divided by 28, the remainder is 3.

We know that the value of r is one of 0 ,1 ,2, 3, 4, 5 and 6.

(1) n is a multiple of 21 plus one of {1,3,5,…,17,19}. Therefore, there are


many possible values of n. NOT SUFF
(2) n is 3 greater than a multiple of 28 and thus a multiple of 7. Therefore
r= 3. SUFF

329
If r, s, and w are positive numbers such that w = 60r + 80s and r + s = 1,
is w < 70?

(1) r > 0.5


(2) r>s

(inequalities, medium)

Note that w = 60r + 80(1-r) = 80 - 20r, so w =80 -20r < 70 if and


only if r > ½.

(1) SUFF
(2) SUFF

330
If s and t are two different numbers on the number line, is s + t = 0 ?

(1) The distance between s and 0 is equal to…


(2) 0 is between

331
If Set S consists of the numbers 1, 5, -2, 8 and n, is 0 < n < 7 ?

(1) The median of the numbers in S is less than 5.


(2) The median of the numbers in S is greater than 1.

(statistics, medium)

Consider set S with its elements in increasing order: S= {-2,1,5,8} U {n}

(1) If the median of the numbers in S is less than 5, n < 5. NOT SUFF
(2) If the median of the numbers in S is greater than 1, n>1 NOT SUFF
(T) 1< n < 5, so 0< n < 7 SUFF
If sequence S has 200 terms, what is the 192nd term of S ?

(1) The first term of S is -40.


(2) Each term of S after the first term is 3 less than the preceeding term.

332
(sequences, medium)

(T) If the first term is -40 and each term of S is 3 less than the preceeding
term, the 192nd term is – 40 – 3(191) SUFF
If 7x/3 + a = 8 and a > 8, then, in terms of a, 3x/7 =

1/a + 1/8 1/8 – 1/a 1/(8 – a) 3(8 – a)/7 9(8 – a)/49

333
(algebra, medium)

7x/3 = 8 – a
Multiplying both sides by (3/7)2 , 3x/7 = 9(8 – a)/49

1/a + 1/8 1/8 – 1/a 1/(8 – a) 3(8 – a)/7 9(8 – a)/49

334
If s is the product of the integers from 100 to 200 inclusive and t is the product
of the integers from 100 to 201 inclusive, what is 1/s + 1/t in terms of t?

(algebra, medium)

Note that t= 201s (i.e. s =t/201) and that 1/s + 1/t =201/t + 1/t = 202/t

335
If the average (arithmetic mean) of x, y, and 20 is 10 greater than the average
of x, y, 20, and 30, what is the average of x and y?

40 45 60 75 95

(algebra, medium)

Suppose that z = x + y. We are given that and asked


for the value of

z + 20 = 30 + (3z + 150)/4, so z/4 = 190/4 and z/2 = 190/2 = 95.

40 45 60 75 95

336
If the average of four different numbers is 30, how many of the numbers are
greater than 30?

(1) None of the four numbers is greater than 60.


(2) Two of the numbers are 9 and 10 respectively.

(inequalities, hard)

(1) The sum of the four numbers is 120. If one number is 60, the sum of the
other three is 60. If they are all nearly equal (i.e 19, 20, 21), there is 1
number greater than 30. On the other hand, the four numbers could be
0,1, 59, 60, in which case there are two numbers greater than 30.
NOT SUFF

(2) The sum of the two remaining numbers is 101. They could be 50 and 51,
or 20 and 81. There could be either 1 or 2 numbers greater than 30.
NOT SUFF

(T) The numbers are, in ascending order 9, 10, x , 101 – x.


Since 101 – x ≤ 60. x ≥ 41. Thus there are two numbers greater than
30. SUFF
Another way to reach this conclusion is to suppose that there is a third
number that is not greater than 30. Thus the sum of three of the
numbers is at most 9 + 10 + 30 = 39 and the fourth number is at least
120 – 39 = 81. However, (1) tells us that none of the four numbers is
greater than 60. Thus, there is no third number that is not greater than
30 (i.e the two unknown numbers are both greater than 30).

337
If the average (arithmetic mean) of six numbers is 75, how many of the
numbers are equal to 75?

(1) None of the six numbers is less than 75.


(2) None of the six numbers is greater than 75.

(inequalities, hard)

If the average of the six numbers is 75, their sum must be 6(75).

(1) If each of the numbers is at least 75, their sum must be at least 6(75). If
any of the numbers is greater than 75, their sum will be greater than
6(75). As we are told that the sum is exactly 6(75), none of the numbers
is greater than 75. Since none of the numbers is less than 75, all six of
the numbers must be equal to 75. SUFF
(2) An argument similar to that for (1) can be made to show that (2) is
sufficient as well. SUFF

338
If the integer k is a multiple of 3, which of the following is also a multiple of 3 ?

k+2 2k + 8 3k + 5 4k + 4 5k + 12

(factors and multiples, medium)

Simply by looking at a possible value for k (e.g. 3), one can see that only 5k +
12 is a multiple of 3. In fact, for integers a and b, ak + b is a multiple of 3 if
and only if b is a multiple of 3 as well. As 12 is a multiple of 3, so is 5k + 12.

339
If the integers a and n are greater than 1 and the product of the first 8 positive
integers is a multiple of an, what is the value of a?

1) an = 64
2) n = 6

(factors and multiples, hard)

Note that the product of the first 8 positive integers can be expressed as 27 x 33
x 5 x 7.

(1) Since 64 can be expressed as 26 , 43 and 82, the value of a could


be any of {2,4,8}. NOT SUFF
(2) If a6 is a factor of the product of the first 8 positive integers, a
must be 2. SUFF

340
If the operation ® is defined for all integers a and b by a®b= ab-a-b, which of
the following statements must be true for all integers a, b and c?

I. a®b=b®a
II. a®0=0®a
III. (a®b)®c= a®(b®c)

I only II only I and II only


I and III only I, II and III

(functions, hard)

I. b®a= ba – b – a =ab – a – b =a®b for all values of a and b.


II. a®0= a(0) – a – 0 = (0)a – 0 – a =0®a for all values of a.
III. (a®b)®c = (ab – a – b)®c= c(ab – a – b) – (ab – a – b) – c
=abc – ac – bc – ab + a + b – c
a®(b®c)= a®(bc – b – c)= a (bc – b – c) – a – (bc – b – c)
=abc – ab – ac – a – bc + b + c ≠ (a®b)®c

Example (1®2)®3= -1®3 = -5 but (1®(2®3))= 1®1 = -1.

I only II only I and II only


I and III only I, II and III

341
If the operation @ is one of the four arithmetic operations addition, subtraction,
multiplication and division, is (6@2)@4=6@(2@4) ?

(1) 3@2>3
(2) 3@1=3

(functions, hard)

Note that this equality will hold if @ is addition or multiplication, but not if it is
subtraction or addition.

(1) As 3@2 >3, the operation @ must be either multiplication or addition.


SUFF
(2) As 3@1= 3. the operation @ must be multiplication or division.
NOT SUFF

342
If the product of the digits of the two-digit positive integer n is 2, what is the
value of n ?

(1) n is odd.
(2) n is greater than 20.

(factors and multiples, medium)

(1) n = 21 SUFF
(2) n= 21 SUFF

343
If the range of the 6 numbers 4,3,14,7,10, and x is 12, what is the difference
between the greatest and least possible values of x?

0 2 12 13 15

Disegarding x, the range of the remaining 5 numbers is 11, so x must be either


1 less than the smallest of these 5 or 1 greater than the largest of these 5.
Thus the difference between the greatest possible value of x and the smallest
possible value of x is 13.

0 2 12 13 15

344
If the sequence x1, x2, x3, …, xn, … is such that x1 = 3 and xn+1 = 2xn – 1
for n > 1, then x20 –x19 =

219 220 221 220 – 1 221 – 1

(sequences, hard)

Look for a pattern:

x1 = 3 , x2 = 2(3) – 1= 5 , x3 = 9 , x4 = 17 , x5 = 33

Note that x2 – x1 = 2 = 21 , x3 – x2 = 4 = 22, x5 – x4 = 16 = 24


This suggests that x20 –x19 = 219

Proof that xn+1 = 2xn – 1 for all n > 1:


We know that x2 – x1 = 21
Suppose xn – xn-1 = 2n- 1 . x n+1 = 2( xn-1 + 2n-1 ) – 1 = xn + 2n

Alternatively,
x20 – x19 = x19 – 1 = 2x18 – 2 = 22x17 - 22 = ... 218x1 - 218 = 2(218) = 219

219 220 221 220 – 1 221 – 1

345
In the sequence s1, s2, ...., sn , s1=1 and s2 = 2. If for all integer values of k
s
from 3 to n, sk = (sk-1) k-2 . What is the ratio of s6 to s5 ?

28 29 210 211 212

(sequences, exponents, medium)

S3 = 21=2 s4 = 22= 4 s5 = 42 = 24 s6 = (24)4 = 216


Thus s6 / s5= 216 / 24 = 212

28 29 210 211 212

346
If the speed of x meters per second is equivalent to the speed of y kilometers
per hour, what is y in terms of x?

5x/18 6x/5 18x/5 60x 3,600,000x

(ratios, hard)

x meters is x/1000 kilometers and 1 second= 1/3600 hour, so x meters per


second is equivalent to (x/1000)/(1/3600) kilometers/hour = 18x/5 kilometers
per hour.

5x/18 6x/5 18x/5 60x 3,600,000x

347
If the symbol Δ represents either addition or multiplication, what is the value of
3Δ2 ?

(1) aΔb = bΔa for all numbers a and b


(2) aΔ(b –c) = aΔb – aΔc for all numbers a and b

(functions, medium)

(1) NOT SUFF


(2) SUFF, Δ = ×

348
If the two regions above have the same area, what is the ratio t:s ?
2:3 16:3 4: 3 2: 4 3 4: 4 3

(geometry, hard)

Regarding the equilateral triangle, the Pythagorean Theorem tells us that


2
t 3
h  t     3t 2 / 4 . This means that h 
2 2
t . As the area of the triangle is
2 2
one half the product of its base and height. Thus the area of the triangle is
3 2
t and that of the square is s 2 .
4
t2 4 t 2
Thus 2  and 4 .
s 3 s 3

4
2:3 16:3 4: 3 2: 3 4: 4 3

349
If the volume of a small container is 14,520 cubic millimeters, what is the
volume of the container in cubic centimeters?
(1 millimeter = 0.1 centimeter)

0.1452 1.452 14.52 145.2 1,452

(exponents, medium)

As 1 millimeter = 0.1 centimeter, 10 millimeters = 1 centimeter and thus 103


=1,000 millimeters =1 cubic centimeter. Thus the volume is 14.52 cubic
centimetres

0.1452 1.452 14.52 145.2 1,452

350
If 36x = 8100, what is the value of (3x-1)3 ?

90 30 10 10/3 10/9

(exponents, hard)

(3x-1)3= 33x-3= 33x / 33 = (36x)1/2/ 33 =90/27= 10/3

90 30 10 10/3 10/9

351
If 0 < x < 1, what is the median of {x , x-1, x2 x , x3} ?

x x-1 x2 x x3

Note that for 0 < x < 1 , the higher the power of x, the smaller the value.
Choose the expression whose exponent is the median of the 5 exponents: x
One could also choose a value for x between 0 and 1 e.g. 1/2

x x-1 x2 x x3

352
If water is leaking from a certain tank at a constant rate of 1,200 milliliters per
hour, how many seconds does it take for 1 milliliter of water to leak from the
tank?

1/3 1/2 3 20 21

(ratios, medium)

1 millimeter x (1 hour/1200 millileters) x (3600 seconds/1 hour) = 3 seconds

1/3 1/2 3 20 21

353
If x is a positive integer and 10x – 74 in decimal notation has digits whose sum
is 440, what is the value of x?

40 44 45 46 50

(exponents, hard)

10x – 74 is a number that has x digits. The last two digits , 2 and 6, sum
8, The other x - 2 digits sum 9(x-2). Thus 9(x – 2) = 432 and x=50

40 44 45 46 50

354
If t is a positive integer and r is the remainder when t2 + 5t + 6 is divided by 7,
what is the value of r?

(1) When t is divided by 7, the remainder is 6.


(2) When t2 is divided by 7, the remainder is 1.

(factors and multiples, hard)

Let S= t2 + 5t + 6= (t+3)(t+2)

(1) t is 6 greater than a multiple of 7, and thus 1 less than the next greatest
multiple of 7. Thus t+3 is 2 greater than a multiple of 7 and t+2 is 1
greater than that multiple of 7. We can say that t+2= 7k+1 and t+3=
7k+2 where k is an integer.

Thus, S= (t+3)(t+2)= (7k+1)(7k+2) =49k2 +7k + 2(7k) + 2


Each of the first three terms of the sum is a multiple of 7, so the sum of
the first three terms is a multiple of 7. This means that S is 2 greater
than a multiple of 7 and thus r=2. SUFF

(2) With this information, we do not know the remainder when t is divided
by 7. Looking at the first few perfect squares, we see that t could be 1 or
6. If t=1, S=12 and r=5. On the other hand, if t=6, S=72 and r=2
NOT SUFF

355
If 20 Swiss Francs is enough to buy 9 notebooks and 3 pencils, is 40 Swiss
Francs enough to buy 12 notebooks and 12 pencils?

(1) 20 Swiss Francs is enough to buy 7 notebooks and 5 pencils.


(2) 20 Swiss Francs is enough to buy 4 notebooks and 8 pencils.

(inequalities, hard)

Let n be the price of each notebook and p the price of each pencil. We need to
know whether 12 (n+p) ≤ 40 i.e n+p ≤ 10/3.
We are told that 9n + 3p ≤ 20 (a)

(1) 7n + 5p ≤ 20. This, combined with (a), means that 16n + 8p ≤ 40. If
notebooks are more expensive than pencils, 4 notebooks can be
exchanged for 4 pencils, and the price of the resulting 12 notebooks and
12 pencils will be at most 40 francs. However, if pencils cost more than
notebooks, when 4 notebooks are exchanged for 4 pencils, the total cost
will rise, perhaps over 40 francs. For example if n=1 and p=2.5, (1) is
true, but n + p > 10/3. NOT SUFF

(2) 4n + 8p ≤ 20. i.e. 2n + 4p ≤10. (a) tells us that 3n + p ≤ 20/3.


Summing these inequalities, we see that 5n + 5p ≤ 50/3 and n + p ≤
10/3. It makes sense that (2) is sufficient, as even if pencils are more
expensive than notebooks, 10 notebooks can be exchanged for 10
pencils without the price rising above 40 francs. SUFF

356
If two of the four expressions x +y, x +5y, x -y and 5x -y are chosen at
random, what is the probability that the product will be of the form x2 –(by)2,
where b is an integer?

1/2 1/3 1/4 1/5 1/6

(algebra, probability, hard)

There are 4 C2 =6 possible products, each of which is equally probable. For the
coefficient of x2 to be 1, 5x – y must not be chosen. Thus each expression must
have an x term with a coefficient of 1, and thus the sum of the coefficients of
the y terms in the 2 expressions chosen must be 0. This means that only one of
the 6 products will be of the required form, the product of x+y and x-y.

1/2 1/3 1/4 1/5 1/6

357
If 2 different representatives are to be selected at random from a group of 10
employees and if p is the probability that both representatives selected will be
women, is p > 1/2?

(1) More than 1/2 of the 10 employees are women.


(2) The probability that both representatives selected will be men is less
than 1/10.

(probability, hard)

Suppose there are n women among the 10 representatives. The probability that
n n  1 n2  n
both reps selected are women is   For this probability to be
10 9 90
greater than 1/2, n2 – n > 45.

(1) n > 5. If n ≤ 7, n2 – n < 45 , but if n > 7, n2 – n > 45 NOT SUFF


(2) If there are 7 women, the probability that both people selected are men
is 3/10 × 2/9 < 1/10. However , n2 – n < 45. If there are 9 women,
the probability that both people selected are men is 0, and n2 – n > 45
NOT SUFF
(T) n could be 7, in which case p <1/2 or n > 7, in which case n >1/2
NOT SUFF

358
If two copying machines work simultaneously at their respective constant rates,
how many copies do they produce in 5 minutes?

(1) One of the machines produces copies at the constant rate of 250 copies
per minute.
(2) One of the machines produces copies at twice the constant rate of the
other machine.

(combined work. medium)

(T) We don’t know which machine is the faster one! NOT SUFF

359
If 210x = 144, what is the value of (2x+1)5 ?

12 60 3(25) 27 3(27)

(exponents, hard)

210x = (25x)2 , so 25x = 12

Thus (2x+1)5 = 25x+5 =(25)(25x) = (25)12 = (27)3

12 60 3(25) 27 3(27)

360
If 2x - 2x-2=3(213), what is the value of x?

9 11 13 15 17

(exponents, hard)

We can write the left side of the equation as a product: 2x-2 (22 – 1)= 2x-2(3).
Thus x – 2 = 13 and x= 15.

361
6
If  1 , which of the following could be the value of x?
x( x  1)

-3.5 -2.5 2.5 3.5 4.5

(inequalities, medium)

6
If  1 , x(x+1) must be positive but less than 6. x could be positive but
x( x  1)
less than 2 or a negative number between -1 and -3

-3.5 -2.5 2.5 3.5 4.5

362
If w + x < 0, is w – y > 0?

(1) x+y<0
(2) y<x<w

(inequalities, hard)

Rephrase the question: is w > y ?

(1) As w + x < 0 and x + y < 0, w + x may be less than, equal to or


greater than x + y. If w + x < x + y, w < y. However, if w + x > x + y,
w > y. NOT SUFF
(2) w > x > y, thus w > y. SUFF

363
If wx=y , then what is the value of xy?

(1) wx2=16
(2) y=4

(algebra, medium)

(1) wx2 = wx multipled by x, so wx2 = xy. SUFF


(2) If y=4, x=4/w, so xy=16/w. NOT SUFF

364
If w, x, y and z are integers such that w/x and y/z are integers, is w/x + y/z
odd?

(1) wx + yz is odd.
(2) wz + xy is odd.

(factors and multiples, hard)

We are told that w/x is an integer, so x is a factor of w. Similarly, z is a factor


of y.

(1) If wx + yz is odd, either wx is an even integer and yz is an odd integer


or vice versa. If wx is even, w is even since w is a multiple of x. In that
case y is odd, as is z and y/z. However, w/x may be odd or even. Thus
the sum w/x + y/z could be odd or even. NOT SUFF

(2) If wz + xy is odd, either wz is even and xy is odd or vice versa.

Suppose that wz is even and xy is odd: x and y will both be odd, and
thus z , a divisor of y, is also odd. This means that w is even and w/x is
also even. As y is odd, so is y/z and therefore y/z + w/x is odd as well.

If, conversely, xy is even and wz is odd, w and z will both be odd, and
thus x, a divisor of w, is also odd. Since xy is even and x is odd, y must
be even and y/z is even. As w and z are both odd, w/x is odd and w/x +
y/z is odd as well. SUFF

Also, you can add fractions!

365
If wz < 2, is z < 1?

(1) w>2
(2) z<2

(inequalities, medium)

(1) If w > 2 > 0, wz < 2 means that z < 2/w < 1 SUFF
(2) NOT SUFF

366
If x and k are integers and (12x)(42x+1) = (2k)(32), what is the value of k ?

5 7 10 12 14

(exponents, factors and multiples, medium)

For the two expressions to be equal, the power of 3 of the left side must be
equal to the power of 3 on the right side. Thus x=2 and the power of 2 on the
left side (122 × 45 )is 14. Thus the power of 2 on the right side must be 14 as
well.

5 7 10 12 14
367
If x and y are both negative and xy < y2 , which of the following must be true?

x < y < x2 < y2 x < y < y2 < x2 y < x < x2 < y2


x2 < y2 < y < x y2 < x2 < y < x

(inequalities, medium)

If xy < y2 and y < 0, x > y . Since y < x < 0, y2 > x2 both of which are
positive. Thus y < x < 0 < x2 < y2.
One could also pick numbers: y = - 1 and x = - 1/2

x < y < x2 < y2 x < y < y2 < x2 y < x < x2 < y2


x2 < y2 < y < x y2 < x2 < y < x

368
If x and y are integers and x > 0, is y > 0?

(1) 7x - 2y > 0
(2) -y < x

(inequalities, hard)

(1) tells us that 2y < 7x and thus y < 7x/2. 7x/2 is a positive number since
x>0, so y could be positive or negative or zero. NOT SUFF
(2) tells us that x + y > 0, but y can be postitive, negative or zero.

(1) and (2) Clearly not sufficient: all we know is that –x < y <7x/2, where x is
a positive integer. If x=2, y could be any integer from -1 to 6.

369
|If x and y are integers greater than 1, is x a multiple of y ?

(1) 3y2 + 7y =x
(2) x2 - x is a multiple of y

(factors and multiples, hard)

(1) x is the sum of two multiples of y and is thus a multiple of y. SUFF


(2) x2 – x = x(x – 1) is a multiple of y.
Thus either x or x – 1 , but not both, is a multiple of y. If x – 1 is a
multiple of y, x will not be a multiple of y. For example 72 – 7= 42 is a
multiple of 6 and 7, but 7 is not a multiple of 6. NOT SUFF

370
If x and y are integers, is x + y greater than 0?

(1) x is greater than 0.


(2) y is less than 1.

(inequalities, easy)

(T) Clearly, neither statement is sufficient by itself. x is a positive integer and


y is either 0 or negative. Thus x + y could be 1, 0 or any negative
integer. NOT SUFF

371
If x and y are non-zero integers and |x| + |y| = 32, what is xy?

(1) -4x – 12y = 0

(2) |x| – |y| = 16

(algebra, hard)

(1) 4x = -12y , so x =-3y and thus |x|=3|y|.


Since |x|+|y| = 32, |y|=8 and |x|=24 and |xy|=8×24
Since x and y have different signs, xy = - 8 × 24 SUFF

(2) Adding the two equations, we see that 2|x| =48, so |x| =24, so |y| =8
Thus |xy| = 8×24, but xy could be positive or negative. NOT SUFF

372
If x and y are positive integers, and x3y4 = 2,000, which of the following is the
value of xy?

2 4 8 10 20

(factors and multiples, medium)

Since 2000 = 24 53, x must be 5 and y must be 2, and xy= 10

2 4 8 10 20

373
If x and y are positive integers and y = , what is the value of y?

(1) x<8
(2) y>1

(factors and multiples, medium)

Note that x has to be a perfect square for y to be an integer. Therefore, (x,y)


{(5,2), (8,1) }

(1) y=2 SUFF


(2) y=2 SUFF

374
If x and y are positive numbers and z = xy2, a 50 percent increase in x and 20
percent decrease in y would result in which of the following changes in z?

A decrease of 4% A decrease of 14% An increase of 4%


An increase of 20% An increase of 30%

(percents, hard)

If x is increased by 50%, the new value of x is 1.5x = 3x/2.


If y is decreased by 20%, the new value of y is 0.8y =4y/5 and that of y2 is
16y2/25.

Thus the new value of z would be be (48/50)xy = (96/100)xy, which is 4%


less than xy, the initial value of z.

A decrease of 4% A decrease of 14% An increase of 4%


An increase of 20% An increase of 30%

375
If x and y are positive integers, is the product xy even?

(1) 5x – 4y is even.
(2) 6x + 7y is even.

(factors and multiples, hard)

The product xy will be even if and only if at least one of the variables is even.

(1) If 5x – 4y is even, 5x must be even and thus x must be even. SUFF


(2) If 6x + 7y is even, 7y must be even and thus y must be even. SUFF

376
If x and y are positive integers such that x=8y+12 what is the greatest
common divisor of x and y ?

(1) x=12u, where u is an integer.


(2) y=12z, where z is an integer.

(factors and multiples, hard)

(1) As x is a multiple of 12, so is x – 12 = 8y. Thus y is a multiple of 3 and


the greatest common divisor of x and y must be at least 3. However, if y
is an even number, there will be a common divisor greater than 3. For
example if y=3, the greatest common divisor will be 3, but if y=6, the
greatest common divisor will be 6. NOT SUFF
(2) If y=12z, x= 12(8z+1). As both x and y are multiples of 12, x and y have
a common divisor of 12. Every divisor of z greater than 2 will be a divisor
of 8z and therefore will not be a divisor of 8z +1. Therefore 12 is the
greatest common divisor of x and y. SUFF

377
If x and y are positive integers, what is the remainder when x is divided by y?

(1) When x is divided by 2y, the remainder is 4.


(2) When x+y is divided by y, the remainder is 4.

(factors and multiples, hard)

(1) x is 4 more than a multiple of 2y, and 2y is greater than 4.

i.e. x=2yk+4 where k is an integer

We see that x is also 4 more than a multiple of y. It is wrong to


automatically conclude that the remainder when x is divided by y is 4,
however. If y=3, the remainder in question is 1. If y=4, the remainder is
0. Only if y>4 is the remainder is 4. NOT SUFF

(2) x+y and x will have the same remainder when divided by y. SUFF

378
If x and y are positive integers, is xy a multiple of 8?

(1) The greatest common divisor of x and y is 10.


(2) The least common multiple of x and y is 100.

(factors and multiples, hard)

(1) Both x and y are even, so xy is a multiple of 4. NOT SUFF


(2) Either x or y is a multiple of 4. NOT SUFF
(T) SUFF

379
If x and y are positive integers, what is the value of xy?

(1) The greatest common factor of x and y is 10.


(2) The least common multiple of x and y is 180.

(factors and multiples, hard)

(1) x and y are both multiples of 10, yet x can have other prime
factors that are not factors of 10. x could be 70 or 30 and y could be 10.
These possibilities yield different values for the value of the product xy.
NOT SUFF
(2) 180= 22 x 32 x 5. Perhaps x any have no common divisor greater than 1,
in which case, xy=180. Alternatively, x could be equal to y, in which case
xy =1802. NOT SUFF

(1) and (2) x and y each have at most 3 prime factors, 2 ,3 and 5. That is,
x  2  3  5 and y  2  3  5 so that xy = 2
a+d
x 3b+e x 5c+f. As the greatest
a b c d e f

common divisor of x and y is 2 x 5 and the least common multiple of x and y is


180= 22 x 32 x 5, a+d=3, b+e=2 and c+f=2. Thus xy= 8 x 9 x 25 = 1800.

It is much more efficient to realize that the the product of two positive integers
is equal to the product of their greatest common divisor and their least common
multiple. SUFF

380
If x and y are positive integers, is y an even integer?

(1) x(y + 5) is an even integer.


(2) 6y2 + 41y + 25 is an even integer.

(factors and multiples, hard)

(1) If x(y + 5) is an even integer. x and y + 5 cannot both be odd. If x is


odd, y + 5 must be even and y must be odd. If x is even, y + 5 can be odd
and thus y can be even. NOT SUFF

(2) This will be even if and only if 41y + 25 is even, as 6y2 is always even.
As 25 is odd, 41y must be odd. Thus y must be odd. SUFF

381
If x is a negative integer, which of the following expressions has the LEAST
value?

x2 - 0.50 x2 - 0.51 x3 - 0.502 x3 - 0.511 x4 - 0.512

(exponents, medium)

If x is a negative integer, x3 will be at least two less than both x2 and x4, as x3
will be a negative integer and x2 and x4 will be positive integers. Thus x3 –
0.511 will be the expression that has the least value.

382
If x is an even integer, which of the following must be an odd integer?

3x 2 3x 2
3x 3x
1 3x 2 1
2 2 2 2

(factors and multiples, hard)

3x 2 3x 2
Note that 3x 2
is a multiple of 4, so is an even integer. Thus  1 is an
2 2
odd integer.

383
If x is an integer is (x2 + 1)(x + 5) an even number?

(1) x is an odd number.


(2) Each prime factor of x2 is greater than 7.

(factors and multiples, hard)

The product will be even as long as one of its two terms is even.

(1) If x is an odd number, x2 is odd and x2 + 1 is even. SUFF


(2) 2 is not a prime factor of x2 and thus 2 is not a factor of x. This means
that x is odd and x+5 is even. SUFF

384
If x is an integer, is 3x + 7 even?

(1) (x – 5)(x + 1) = 0
(2) x is a factor of 105.

(factors and multiples, hard)

3x + 7 is even if and only if x is an odd integer.

(1) x is either 5 or -1. In either case, x in an odd integer. SUFF


(2) x is a factor of an odd number. Therefore, x is odd. SUFF

385
1
If x and y are positive, which of the following must be greater ?
x y

x y
I.
2x
x y
II.
x y
x y
III.
x y

(exponents, hard)

1 x y
Note that = . This is clear by multiplying numerator and
x y x y
denominator by the denominator. Supposing that x=y will quickly show that
neither I nor III need be true. If x and y are positive, x  y  x  y , so II
must be true.

386
x, 3, 1, 12, 8

If x is an integer, is the median of the 5 numbers shown greater than the


average (arithmetic mean) of the 5 numbers?

(1) x>6
(2) x is greater than the median of the 5 numbers.

(statistics, hard)

For {1,3,8,12} U {x}, note that the average of these integers is (24+x)/5. The
median is 3 if x is less than or equal to 3, x if x is between 3 and 8 and 8 if x is
at least 8.

(1) Clearly the average can be greater than the median. If x is a large
positive integer, the median will only be 8, but the average will be much
larger than 8. Conversely, if x=7, the median is 7, a number greater than
31/5, the arithmetic mean.
(2) This means that x is greater than 8. Clearly the average can be greater
than the mean (see (1)). However if x=9, the median is 8, a number
greater than 33/5, the arithmetic mean.
(T) NOT SUFF

387
If x is an integer, which of the following must be an odd integer?

2x + 2 4x + 3 12x – 6 13x 14x

(factors and multiples, medium)

Note that 4x, a multiple of 4, is even, so 4x + 3 is odd.

2x + 2 and 12x – 6 and 14x are even, whereas 13x is even if x is even.

2x + 2 4x + 3 12x – 6 13x 14x

388
If x is not equal to y, is (x - y)/(x + y) > 1?

(1) x>0
(2) y<0

(inequalities, hard)

If x + y > 0, we can simplify the question as follows: is x – y > x + y ?


If x + y < 0, we can simplify the question as follows: x – y < x + y ?

So if x + y >0, it is sufficient to see whether y is negative and if x + y < 0, it is


sufficient to see whether y is positive.

(2) tells us y is negative, but as x is positive from (1). we don’t know whether
x + y is positive or negative. Not even taken together are (1) and (2) sufficient

389
If x is a positive integer and y is a negative integer, what is the largest possible
1
value of ?
x y

1/4 1/2 2/3 3/4 1

(algebra, medium)

As x is a positive integer and y is a negative integer, x – y is at least 2, so the


reciprocal of x – y is at most 1/2.
1/4 1/2 2/3 3/4 1

390
If x is a positive integer, is x! + (x + 1) a prime number ?

(1) x < 10
(2) x is even

(factors and multiples, hard)

(T) Consider 8! + 8 + 1.
As both 8! and 8+1 are multiples of 3, so is 8! + 8 + 1, which is thus not
a prime number.
However 2! + 2 + 1 = 5 is a prime number.
NOT SUFF

391
If x is a positive number less than 10, is z greater than the average of x and
10?

(1) On the number line, z is closer to 10 than it is to x.


(2) z=5x

(algebra, medium)

Note that the midpoint between x and 10 is equal to the average of x and 10.

(1) This tells us that on the number line, z is to the right of the
midpoint between x and 10 and is thus greater than the average
of x and 10. SUFF
(2) As z=5x, the statement is true if and only if 5x > (x+10)/2
i.e. 9x > 10, x >10/9
The statement may or may not be true depending on the value of x
NOT SUFF

392
If x is positive, is x > 3 ?

(1) (x – 1)2 > 4


(2) (x – 2)2 > 9

(inequalities, medium)

Remember that w2  w for all real numbers w. Also, for positive numbers t

and u such that t > u, t u

(1) (x – 1)2 > 4 implies that |x – 1| > 2. Thus x – 1 > 2 or x – 1 < -2


Therefore, x > 3 or x < - 1. However, since x > 0, x > 3. SUFF
(2) (x – 2)2 > 9 implies that |x – 2| > 3. Thus x – 2 > 3 or x – 2 < -3
Therefore, x > 5 or x < - 1. However, since x > 0, x > 5 > 3. SUFF

393
If x is positive, which of the following could be the correct ordering
of 1 , 2 x and x 2 ?
x
1
I. x 2  2x 
x
1
II. x2   2x
x
1
III. 2x  x 2 
x

I only II only III only


I and II only I, II, III

(inequalities, hard)

First find out for what positive values of x these expressions could be equal:
1 2
 2x  x  1
 x2  x  1 2x  x2  x  2
x 2 x

Test x= 1/2 , 3/4, 3/2 and 3, as these values of x are between the
aforementioned values of x

x 1
x2 2x
x
1/2 2 1/4 1 I
3/4 4/3 9/16 3/2 II
3/2 2/3 9/4 3
3 1/3 9 6

I and II only

394
If x is the product of the positive integers from 1 to 8, inclusive, and if i, k, m,
and p are positive integers such that x = 2i3k5m7p, then i + k + m + p =

4 7 8 11 12

(factors and multiples, medium)

2,3,5 and 7 are prime numbers, while 4 and 6 are the product of two prime
numbers, and 8 is the product of 3 prime numbers.
Thus i + k + m + p = 4 + 4+ 3 = 11

4 7 8 11 12

395
If X is to be chosen at random from the set {1,2,3,4} and Y is to be chosen at
random from set {5,6,7}, what is the probability that XY will be even?

1/6 1/3 1/2 2/3 5/6

(probability, medium)

Rather than find the probability that XY is even, it is easier to find the
probability that XY is odd: For the product XY to be odd, both X and Y must be
odd. The probability that X is odd is 2/4=1/2 and the probability that Y is odd is
2/3. These are independent events, so the probability that both X and Y are
odd is 1/2 x 2/3= 1/3. Thus the probability that XY is not odd is 1 - 1/3= 2/3.

396
If x < 0, then x| x| 

-x -1 1 x x

(algebra, hard)

If x < 0 , |x| = - x and thus –x|x|= (-x)(-x)= x2. Therefore,  x | x | 


x 2 | x |  x.
Note that picking numbers works wonderfully for this question.

-x -1 1 x x
CORRECT

397
2
If x > 0, is x < x ?

(1) 0.1 < x < 0.4


3 2
(2) x <x

(inequalities, medium)

(E) Note that if x > 0, by dividing or dividing both sides by x, we see


that x2 < x is equivalent to x < 1 and also to x3 < x2. Each statement is
sufficient.

398
If x=1/2, is y equal to 1 ?

(1) y2 ( x + 1/2) = 1
(2) y( 2x – 1 ) = 2x – y

(algebra, medium)

(1) implies that y2 = 1, so y must be either 1 or -1. NOT SUFF


(2) implies that y = 2x = 1 SUFF

399
2 ( x y )
2

If xy =1, what is the value of ?


2 ( x y )
2

2 4 8 16 32

(exponents, medium)

2( x  y )
2

  x  y 2
 2( x  y )  2x  2 xy  y 2  ( x 2  2 xy  y 2 )
 24 xy  24  16.
2 2

2( x  y )
2

400
If x > 0.9, which of the following could be the value of x?

0.81 0.9 (0.9)2 (0.9)(0.99)


1 0.01

(exponents, medium)

0.81 =0.9 0.9 < 0.9 < 1 (0.9)2 < 0.9


(0.9)(0.99) < 0.9 1 0.01 = 1- 0.1=0.9

401
If xy = 165, where x and y are positive integers and x > y, what is the least
possible value of x – y ?

2 4 8 15 28
(factors and multiples, medium)

Look for factors that are close to the positive square root of 165.
As 165= 5 x 11 x 3, choose x = 15 and y = 11, in which case x – y would be 4.
Alternatively, realise that both numbers need to be odd and one has to be a
multiple of 5!

2 4 8 15 28
402
If x is a perfect square and x=paqbrcsd , are prime numbers p, q, r and s
distinct?

(1) 18 is a factor of ab and cd


(2) 4 is not a factor of ab and cd

(factors and multiples, hard)

If p, q, r and s are distinct prime numbers, a, b,c, d must be even numbers.

(1) a and b may or may not be both even. NOT SUFF


(2) a and b cannot be both even. Therefore p,q,r and s cannot be distinct
prime numbers. SUFF

403
If x < 0 and 0 < y < 1, which of the following has the greatest value?

x2 (xy)2 (x/y)2 x2/y x2y

(exponents, medium)

Note that if 0< y < 1, 1/y2 > 1/y > 1 > y > y2 . Thus (x/y)2 is the greatest.

x2 (xy)2 (x/y)2 x2/y x2y

404
If x < 0, y > 0, and z < 0, which of the following is negative?

xy2z x2yz x2yz2 x3yz x3yz3

(exponents, medium)

The exponent on y is irrelevant. Look for a choice for which the exponent on x
is odd and that on z is even or vice versa.

xy2z x2yz x2yz2 x3yz x3yz3

405
If x/y > 2, is 3x + 2y < 18 ?

(1) x - y is less than 2.


(2) y - x is less than 2.

(inequalities, hard)

Note that since x/y > 0 , x and y are of the same sign. If x and y are both
negative, x < 2y and thus x < y. In this first case, 3x + 2y is negative. On the
other hand, if x and y are both positive, x > 2y and thus x > y. In this second
case, 3x + 2y is positive.

(1) x – y < 2. In other terms, x < 2 + y. If x < y, x and y are both negative, as
is 3x + 2y. On the other hand if y< x < y +2, x and y are both positive and x >
2y. Thus y + 2 > 2y and y < 2. Since y < 2, x < 4 and 3x +2y < 16< 18.Thus
3x + 2y is always less than 18. SUFF

(2) y – x < 2. This means that y < x + 2. It could be that y < x, in which
case , x and y are both positive. 3x + 2y is thus greater than 3x, which could
very well be greater than 18. For example if x=6 and y=2, 3x+2y=22> 18.
Alternatively, it could be that x < y < x+2, in which case x and y are both
negative, as is 3x +2y. NOT SUFF

406
If xy < 0, which of the following must be negative?

xy2 x2y x2y2 x2y3 x3y3

(inequalities, exponents, medium)

If a number is negative, so is the result when it is raised to an odd integer


power.

xy2 x2y x2y2 x2y3 x3y3

407
If –2x > 3y, is x negative?

(1) y>0
(2) 2x + 5y – 20 = 0

(inequalities, hard)

(1) - 2x > 3y > 0 since y > 0. Thus x > 0. SUFF


(2) We know from the question that y < -2x/3 and thus 5y < -10x/3.
From (2), we can say that 20 – 2x = 5y < -10x/3.
It follows that 4x/3 < -20 and so x < 0 SUFF

408
If x > y2 > z4, which of the following statements could be true?

I. x>y>z
II. z>y>x
III. x>z>y

I only I & II I & III II & III I, II, & III

(exponents, hard)

Note that if each variable is greater than 1, x2 > x > y2, so x >y. Similarly, y4 >
y2 > z4 so y > z. Thus if each variable is greater than 1, x > y > z (I).

It may also be that each y and z are between 0 and 1, In that case y4 < y2 < y.
It could be that y4 < z4, i.e. y < z. For example if y=1/3 and z=1/2, y2 =1/9 >
1/16= z4. If x > y2, x > 1/9, so x could be less than y (II) or greater than z
(III).

I only I & II I & III II & III I, II, & III

409
If x2 + 3x + c = (x + a)(x + b) for all x, what is the value of c ?

(1) a=1
(2) b=2

(algebra, medium)

Note that ( x + a )(x + b )= x2 + ( a + b ) x + ab. Thus a + b = 3 and c = ab.


We can find the value of c if we know the values of a and b.

(1) Knowing that a = 1 allows us to find b and thus c.


(2) Knowing b = 2 allows us to find a and thus c.

410
( x  y) 2
If 0 < x < y, what is the value of
( x  y) 2

(1) x2 + y2 = 3xy
(2) xy = 3

(algebra, médium)

Note that N=(x+y)2 = x2 + y2 + 2xy and D=(x – y)2 = x2 + y2 – 2xy.

(1) If x2 + y2 = 3xy, N= 5xy and D=xy and N/D= 5. SUFF


(2) xy = 3. If x=1, y=3, N=16, D=4 and N/D=4.
If x=3/2 and y=2, N=49/4 , D=1/4 and N/D=49 NOT SUFF

411
If x and y are nonzero integers, is 18 a factor of xy2?

(1) x is a multiple of 54.


(2) y is a multiple of 6.

(factors and multiples, hard)

(1) x is a multiple of 54 and thus of every factor of 54, including 18. Thus
xy2 is a multiple of 18, i.e. 18 is a factor xy2. SUFF
2 2
(2) If y is a multiple of 6, y is a multiple of 6 = 36 and thus is a multiple of
18. Therefore xy2 is a multiple of 18. SUFF

412
If x percent of 40 is y, then 10x equals

4y 10y 25y 100y 400y

x/100 of 40 = 40x/100 =y, so 40x = 100y and 10x = 25y

4y 10y 25y 100y 400y

413
If x represents the sum of all the positive three-digit numbers that can be
constructed using each of the distinct nonzero digits a, b, and c exactly once,
what is the largest integer by which x must be divisible?
3 6 11 22 222

(factors and multiples, hard)

x= (100a +10b + c) + (100a + 10c + b) +…+(100c +10b + a) =


200(a+b+c)+20(a+b+c)+2(a + b + c)=222(a + b + c), which is a multiple of
222.

3 6 11 22 222

414
10 x 20 y
If x, y, and k are positive numbers such that   k and x < y, which
x y x y
of the following could be the value of k?

10 12 15 18 30

(inequalities, hard)

10 x 20 y 10( x  y) 10 y 10 y
    10 
x y x y x y x y x y

10 y
Since x < y , x/2 + y/2 < y < x + y. Therefore 5   10 .
x y
It follows that 15 < k < 20.

Alternatively, think that k is the average value of x+ y bills, x of which are $10
bills and y of which are $20 bills. k will be between 15 and 20, i.e. between 10
and 20 and closer to 20 than to 10.

10 12 15 18 30
415
If x, y and z are positive integers, what is the value of x + y + z ?

(1) xyz  70
x 7
(2) 
yz 10

(factors and multiples, hard)

(1) If xyz  70
70= 2 x 5 x 7. It could be that z=70 and x=y=1. In this case, x + y + z
=72. However it could be that z=7, y=5 and x=2 so that x + y + z =14.
NOT SUFF
(2) Suppose y=1. Then x:z= 7:10, so x + z is a multiple of 17. Thus x + y +
z= 1 + 17k, where k is a positive integer. NOT SUFF

(1) and (2) x =7 and yz= 10. Note that if y=1, z=10 and x + y + z =18.
However, if y=2, z=5 and x + y + z= 14. NOT SUFF

416
If xy ≠ 0, is x/y = 1?

(1) x2 = y2
(2) xy > 0

(inequalities, medium)

(1) x2 = y2 tells us that |x| = |y|, so y = x or y= - x. If y = x , x/y = 1, but if


y = - x , x/y = -1. NOT SUFF
(2) x/y > 0 NOT SUFF
(T) Since xy > 0 , y=x and x/y = 1 SUFF

417
If x, y and z are integers and xy + z is an odd integer, is x an even integer?

(1) xy + xz is an even integer.


(2) y + xz is an odd integer.

(factors and multiples, hard)

Since xy + z is an odd integer, it must be that xy is even and z is odd, or vice


versai.

(1) xy + xz is an even integer. We know that xy + z is an odd integer. Thus


(xy+xz) – (xy + z) = z (x – 1) is an odd integer. This means that x – 1 is
an odd integer and x is an even integer. SUFF
(2) (xy + z) + (y + xz) is an even integer. Factoring, (x+1)(y+z) is an even
integer. It could be that x +1 is even, so that x is odd, and y + z is it
could be that x + 1 is odd and y + z is even NOT SUFF

418
419
If x, y and z are positive integers, what is the remainder when 100x + 10y + z
is divided by 7?

(1) y=6
(2) z=3

(factors and multiples, medium)

(T) 100x + 10y + z = 100x + 63. Though 63 is a multiple of 7, 100 is not, so


the remainder will depend on the value of x. For example if x = 1, the
remainder will be positive, but if x=7, the remainder will be 0. NOT SUFF

420
If x + y = 24 and 2x -7y = 15, which of the following is equal to 13?

x–y x + 2y x – 2y 2x + y 2x – y

(algebra, medium)

Note that 24 + 15 = 39 =3(13)


Thus 3x – 6y = 3(13) and thus x – 2y = 13

x–y x + 2y x – 2y 2x + y 2x – y

421
If x ≠ 0, is ?

(1) x<1
(2) x > −1

(inequalities, medium)

if and only if , which is true if and only if x2 < 1


i.e. – 1 < x < 1

(T) SUFF

422
If xyz  0 , is x( y  z )  0 ?

(1) | y  z || y |  | z |
(2) | x  z || x |  | z |

(algebra, hard)

Since x is not equal to 0, it is enough to determine whether y + z is equal to 0

(1) tells us that y and z are the same sign, a fact that is evident when both
sides are squared:
(| y  z |) 2  (| y |  | z |) 2  y 2  2 yz  z 2  y 2  2 yz  z 2  yz  yz  0
As y and z are the same sign, their sum cannot be 0. SUFF

(2) tells us that x and z are the same sign, though nothing is said about y. If
y = - z , the statement is true. Otherwise, the statement is false.
NOT SUFF

423
If 0 < r < 1 < s < 2, which of the following must be less than 1?

I r/s
II rs
III s – r

I only II only III only I and II I and III

(inequalities, medium)

Since r and s are both positive and r < s, r/s < 1 I must less than 1
If r= 3/4 and s= 3/2 , rs = 9/8 > 1 II need not be less than 1
If r= 1/4 and s= 3/2 s–r>1 III need not be less than 1

I only II only III only I and II I and III

424
If 0 < x < 1, which of the following inequalities must be true?

I) x5 < x3
II) x4 + x5 < x3 + x2
III) x4 - x5 < x2 - x3

None I only II only I and II I, II, and III

(exponents, hard)

Note that if 0 < x < 1, xp > xq if and only if p < q.


Thus (I) is obviously true.
(II) is true if and only if x4(1 + x) < x2(1 + x ) i.e. x4 < x2 , which is always true
if 0 < x < 1.
Similarly, III is true if and only if x4(1 - x) < x2(1 - x ) i.e. x4 < x2 , which is
always true if 0 < x < 1.

None I only II only I and II I, II, and III

425
If y is an integer and y = |x| + x, is y = 0 ?

(1) x<0
(2) y<1

(algebra, hard)

Note that |x| = x if and only if x ≥ 0. Otherwise, |x| = - x


Thus if x > 0, y = x + x = 2x. If x = 0, y = 0. If x < 0 , y = x – x = 0.

(1) SUFF
(2) Suppose y ≠ 0. In this case , y = 2x > 0. Thus y would be a positive
integer. Since y < 1, y is not a positive integer. Thus y = 0 SUFF

426
If y ≥ 0, what is the value of x?

(1) |x-3| ≥ y

(2) |x-3| ≤ -y

(inequalities, medium)

(1) - y ≤ x – 3 ≤ y , so 3 – y ≤ x ≤ 3 + y. NOT SUFF


(2) Since – y ≤ 0, |x – 3| = 0 and x = 3 SUFF

427
If zn =1. What is the value of z ?

(1) n is an non-zero integer.


(2) z>0

(exponents, hard)

(1) If n is a non-zero integer, z must be either 1 or -1. For example if


n=1, z=1. However, if n=2, z =1 or z = -1. NOT SUFF
(2) If n=0, z could be any number other than 0. If n≠0, z must be 1.
NOT SUFF.
(T) (1) tells us that n≠0. Therefore, according to (2), z=1. SUFF

428
If zt < - 3, is z < 4 ?

(1) z<9
(2) t < -4

(inequalities, hard)

Combining (1) and (2),


since t < 0, z > -3/t.
Since t < -4, z > 3/4
Therefore, 3/4 < z < 9
z may or may not be less than 4.

(T) NOT SUFF

429
If zy < xy < 0, is |x - z| + |x| = |z| ?

(1) z<x
(2) y>0

(inequalities, hard)

Note that zy < xy implies that (x – z)y > 0. Thus x – z and y have the same
sign. x and y have different signs, as do z and y.

(1) z < x, so x – z > 0 and y > 0. Thus x < 0 and z < 0.


|x - z| + |x|= x – z – x = - z =|z| SUFF
(2) y > 0 implies that x – z > 0, as in (1) SUFF

430
In a box of 12 pens, a total of 3 are defective. If a customer buys 2 pens
selected at random from the box, what is the probability that neither pen will be
defective?

1/6 2/9 6/11 9/16 3/4

(probability, medium)

The probability that the first pen selected is not defective is 9/12= 3/4.
If the first pen selected is not defective, the probability that the second is not
defective is 8/11. Thus the probability that neither pen is defective is 3/4 x 8/11
= 6/11.

98
C 29 2!  6
Alternatively, the required probability is 
12
C2 12  11 11
2!
1/6 2/9 6/11 9/16 3/4

431
In a certain bathtub, both the cold water and the hot water fixtures leak. The
cold water leaks alone and would fill an empty bucket in c hours, and the hot
water leaks alone would fill the same bucket in h hours, where c < h. If both
fixtures began to leak at the same time in the empty bucket at their respective
constant rate and consequently, it took t hours to fill the bucket, which of the
following must be true?

I 0<t<h
II c<t<h
III c/2 < t <h/2

I only II only III only I and II I and III

(combined work, inequalities, hard)

Note that the cold water fixture leaks faster than does the hot water fixture. If
the hot water fixture leaked as fast as the cold water fixture, t would be c/2.
Likewise, if the cold water fixture leaked as slow as the hot water fixture, t.
would be h/2. Thus c/2 < t < h/2 (III). I is obviously true. II is obviously not
true, as t is clearly less than c.

1 1 1 hc c h
More formally,   t    . Since 0 < c < h, 0< c/h < 1
t c h h  c 1 c h 1
h c
and c/2 < t. Since h/c > 1, t < h/2.

I only II only III only I and II I and III

432
In a certain English class, 1/4 of the number of girls is equal to 1/6 of the total
number of students. What is the ratio of the number of boys to the number of
girls in the class?

1 to 4 1 to 3 1 to 2 2 to 3 2 to 1

(ratios, medium)

If 1/4 of the number of girls is equal to 1/6 of the total number of students,
multiplying by 4, we see that the number of girls is equal to 4/6 = 2/3 of the
number of students. Thus 2/3 of the students are girls and 1/3 are boys. As
there are twice as many girls as boys, the required ratio is 1 to 2.

Alternatively g/4 = 1/6( g+b)

Multiplying by 12, we get 3g =2g +2b , so g=2b and b/g= 1/2

1 to 4 1 to 3 1 to 2 2 to 3 2 to 1

433
In a certain clothing store, 60 percent of all the articles are imported and 20
percent of all the articles are priced at $100 or more. If 40 percent of the
articles priced at $100 or more are imported, what percent of the articles priced
under $100 are not imported?

35% 28% 8% 4.8% 2%

(percents, sets, hard)

Suppose that there are n articles in all. 0.6n are imported. Also, 0.2n are priced
at $100 or more, 40% of which (i.e. 0.08n) are imported and 60% of which
(i.e. 0.12n are not imported). Since the total number of articles not imported is
0.4n, 0.28n is the number of articles priced under $100 that are not imported.
Since there are 0.8n articles that are priced under $100, the required
percentage is 0.28n/0.8n x 100% = 35%.

35% 28% 8% 4.8% 2%

434
In a certain district, the ratio of the number of registered Republicans to the
number of registered Democrats was 3/5. After 600 additional Republicans and
500 additional Democrats registered, the ratio was 4/5. After these
registrations, there were how many more voters in the district registered as
Democrats than as Republicans?

100 300 400 1,000 2,500

(ratios, medium)

Before the new registrations, there were 3x Republicans and 5x Democrats.


Afterwards, there were 3x + 600 Republicans and 5x + 500 Democrats, a
difference of 2x - 100

(3x + 600)/(5x + 500) = 4/5, so 15x + 3000 = 20x + 2000


x= 200
Thus the difference is 2(200) – 100 = 300

100 300 400 1,000 2,500

435
In a certain furniture store, each week Nancy earns a salary of $240 plus 5
percent of the amount of her total sales that exceeds $800 for the week. If
Nancy earned a total of $450 one week, what were her total sales that week?

$2200 $3450 $4200 $4250 $5000

(algebra, percents, medium)

Suppose her total sales that week were s > 800


450 = 240 + (s – 800 )/20
s= 5000

$2200 $3450 $4200 $4250 $5000


436
In a certain game, the units of currency of 3 countries are the crown, shield
and the pound, respectively. If 2 crowns equal 3 shields and 3 shields equal 4
pounds, how many crowns equal 18 pounds?

9 16 18 32 36
(algebra, medium)

Suppose that the value of the crown, shield and pound are c, s and p. We are
asked for the value of 18p in terms of c.
2c = 3s and 3s = 4p. Thus 2c = 4p and c = 2p. Multiplying both sides by 9, we
get that 9c = 18p, i.e. 18 pounds are worth 9 crowns.

9 16 18 32 36

437
In a certain group of 10 members, 4 members teach only French and the rest
teach only Spanish or German. If the group is to choose a 3-member committee
that must have at least 1 member who teaches French, how many different
committees can be chosen?

40 50 64 80 100

(cominatronics, hard)

Of the C3  120 distinct 3-member committees that can be formed, C3  20


10 6

have no members that speak French. Thus the remaining 100 3-member
committees that can be formed have at least one member who teaches French.

40 50 64 80 100

438
In a certain year, the difference between Mary's and Jim's annual salaries was
twice the difference between Mary's and Kate's annual salaries. If Mary's annual
salary was the highest of the 3 people, what was the average (arithmetic
mean) annual salary of the 3 people that year?

(1) Jim's annual salary was $30,000 that year.


(2) Kate's annual salary was $40,000 that year.

(algebra, hard)

If Mary’s salary (m) is the highest, let Kate’s salary, be x less than m. Thus
Jim’s salary is m – 2x. The average of the three salaries is
m  (m  x)  (m  2 x)
 m  x , which is Kate’s salary.
3
(1) m – 2x = $30,000. NOT SUFF
(2) m – x= $40,000 SUFF

439
In a certain year the UN's total expenditures were $1.6 billion. Of this amount,
67.8 percent was paid by the 6 highest-contributing countries and the balance
was paid by the remaining 153 countries. Was Country X among the 6 highest-
contributing countries?

(1) 56 percent of the total expenditures was paid by the 4 highest-


contributing countries, each of which paid more than Country X.
(2) Country X paid 4.8 percent of the total expenditures.

(algebra, medium)

Combining (1) and (2), we know that the 4 highest-paying countries paid 56%.
Thus the fifth-highest and six-highest paying countries must have paid a total
of 11.8% of the total. Country X could certainly be the sixth-highest paying
country- another country not among the 4 highest-paying countries could have
contributed 7%. This 11.8% could, however, have been contributed in equal
sums by two countries (5.9% each), neither of which would be Country X.
(T) NOT SUFF

440
In a class of 30 students, 2 students did not borrow any books from the library.
12 student each borrowed 1 book, 10 students each borrowed 2 books and the
rest of the students each borrowed at least 3 books. If the average (mean)
number of books borrowed per students was 2. What is the maximum number
of books that any student could have borrowed?

3 5 8 13 15

(inequalities, medium)

60 books were borrowed in all, 32 of which were borrowed by the 24 students


who borrowed less than 3 books. If 5 of the 6 remaining students borrowed 3
books each, 13 books were borrowed by 1 student.

3 5 8 13 15

441
In a deck of cards, each has a positive integer on it. A multiplication game
requires drawing a card and then multiplying the integer on it with the next
largest integer. The products are each between 15 and 200. What could be the
least and greatest integers on the cards?

3 and 15 3 and 20 4 and 13 4 and 14 5 and 14

(inequalities, hard)

Let x be the least integer. We know that x(x + 1)> 15, so x is at least 4. Let y
be the greatest integer. We know that y(y + 1)< 200 so y is at most 13. Thus
the numbers on the cards must be between 4 and 13 inclusive.

3 and 15 3 and 20 4 and 13 4 and 14 5 and 14

442
In a certain senior class 72 percent of the male students and 80 percent of
female students have applied to college. What fraction of students in the class
are male?

(1) There are 840 students in the senior class.


(2) 75 percent of the students in the senior class have applied to college.

(ratios, percents, hard)

Keep in mind that if we can find the ratio of the number of male students to
that of female students, we can answer the question.

(1) Clearly not sufficient- all we know is that the number of male students is
a multiple of 18 and the number of female students is a multiple of 5.
NOT SUFF
(2) Suppose that there are m male students and f female students. We are
told that 0.72m + 0.8f = 0.75(m+f), so 0.03m = 0.05f. Thus m/f = 5/3
and m/(m+f) =5/8. SUFF

443
In a demographic study, the population and total income of a certain region
were estimated from other data, and both estimates had lower and upper
limits. At the time of the estimates, was the per capita income for the region
greater than $16,500?

(1) The lower limit for the estimate of the population was 330,000 people.
(2) The lower limit for the estimate of the total income was $5,500,000,000.

(inequalities, ratios, medium)

(T) We are asked whether I/p > 16500, so neither statement alone is
sufficient. As the lower limits, but not the upper limits of I and p are
given, not even combining (1) and (2) allows us to answer the question.
NOT SUFF

444
In a nationwide poll, N people were interviewed. If 1/4 of them answered "yes"
to question 1, and of those, 1/3 answered "yes" to question 2, which of the
following expressions represents the number of people interviewed who did
NOT answer "yes" to both questions?

N/7 6N/7 5N/12 7N/12 11N/12

(ratios, sets, hard)

It is easier to find the number of people who answered ―yes‖ to both questions
and subtract this from N, the total number of people polled. 1/3 of the N/4
people who answered ―yes‖ to question 1 also did so for question 2. Thus, N/12
of the N people polled answered ―yes‖ to both questions, and 11N/12 did not
do so.

N/7 6N/7 5N/12 7N/12 11N/12

445
In an infinite sequence of integers, the first term a1 = 40 and an < an-1 for all n
greater than 1.Are there more than 20 positive terms in this sequence ?

(1) a25 = a24/2


(2) The product of the first 25 terms of the sequence is positive.

(sequences, hard)

(1) If a25 = a24/2 and a25 < a24, a25 and a24 must both be positive. If a25=
a24/2 were negative, a24 < a25 < 0. Thus a1, a2, ... , a25 are all positive, and
there are at least 25 positive terms in the sequence. SUFF

(2) Among the first 25 terms of the sequence at hand, there is an even
number of negative terms. Thus there could be as many as 24 negative terms
among the first 25 terms. Alternatively, all of the first 25 terms could be
positive. NOT SUFF

446
In a survey of students, each student selected from a list of 12 songs the 2
songs that the student liked best. If each song was selected 4 times, how many
students were surveyed?

96 48 32 24 18

(algebra, medium)

If each song was selected 4 times, 48 songs were selected, 2 per student. Thus
24 students participated in the survey.

96 48 32 24 18

447
In a survey of 248 people, 156 are married, 70 are self-employed and 25% of
those who are married are self employed. If a person is to be randomly
selected from those surveyed, what is the probability that the preson selected
will be self employed but not married?

1/8 4/31 117/248 1/4 31/117

(sets, probability, medium)

Of the 156 who are married, 25% i.e. 39 are self employed. Thus 70 -39 = 31
people , 1/8 of the total number of people, are self-employed but not married.

1/8 4/31 117/248 1/4 31/117

448
In a stack of boards at a lumber yard, the 20th board counting from the top of
the stack is immediately below the 16th board counting from the bottom of the
stack. How many boards are in the stack?

38 36 35 34 32

(algebra, medium)

Suppose that the boards are numbered from 1 to n, starting from the bottom
one. The 20th board counting from the top is numbered n – 19, a number that
is one lower than 16, the number on the 16th board counting from the bottom.
Thus n - 19 = 16 – 1 so n=34.

Alternatively, it stands to reason that the 19th board counting from the top is
also the 16th board counting from the bottom. There are 18 boards above this
board and 15 below it. Thus there are 18 + 15 + 1 = 34 boards.

38 36 35 34 32

449
In each of the first eleven months of 2006, sales of computers at Megabits rose
between 10.0% and 12.0% compared to the same month in 2005. Did
Megabits sell more computers in 2006 than it did in 2005?

(1) December was the slowest month of 2005 in terms of computer sales.
(2) Fourth quarter sales in 2006 were more than 97% of fourth quarter sales
in 2005.

(inequalities, hard)

Suppose sn is the number of computers sold in the nth month of 2005. T, the
total sales for 2005 are s1 + s2 +…+ s12. The sales for the first 11 months of
2006 are between 1.1 (T – s12) and 1.12 (T –s12).

(1) s12 is less than each of s1, s2, …, s11 and thus less T/12. Thus the sales
for the first 11 months of 2006 are greater than 1.1 (11T/12) > T. It is
clear that the number of computers sold in the first 11 months of 2006 is
greater than the total sales of 2005. SUFF

(2) As no information is given about the sales in the 12th month of 2005, we
don’t know how sales in 2006 compared with those in 2005.
NOT SUFF

450
In Finn's preschool class, each student is assigned a unique color palette of 3
colors for a finger-painting project. Different students may have 1 or 2 colors in
common, but no 2 students have the same 3 colors. If there are 10 students in
the class, how many different colors are required?

4 5 6 7 8

(combinatronics, medium)

If there are n colours, C3n  n(n  1)(n  2) different pallets can be made. If n=4,
6
4 pallets can be made and if n=5, 10 different pallets can be made.

4 5 6 7 8

451
In May, Mrs. Lee’s earnings were 60% of the family income. In June, Mrs. Lee
earned 20% more than in May. If the rest of the family’s income was the same
in both months, then in June, Mrs. Lee’s earnings were approximately what
percent of the Lee family’s total income?

64% 68% 72% 76% 80%

(percents, hard)

Suppose that the Lee family income in May was n. Mrs. Lee’s earnings in May
was 0.6n and the combined income of the remaining family members was 0.4n.
Mrs. Lee earned 1.2(0.6n)=0.72n, and the combined income of the remaining
family members was 0.4n. Thus in June Mrs. Lee’s income accounted for
0.72n/1.12n of the total family income or 7200/112%=3600/56%=450/7%

64% 68% 72% 76% 80%

452
In May ,Xiang sold 15 used cars.For these cars the range of selling prices was
15,000 and the lowest selling price was 4,500,In june Xiang sold 10 used cars .
For these 10 cars the range of the selling price was 16,500 and the lowest
selling price was 6100 . What is the range of selling prices of the 25 used cars
sold by Xiang in May and June?

15,600 15,750 16,820 18,100 19,200

(statistics, medium)

Lowest price – 4,500


Highest price- 6,100 + 16,500

Range 16500 + 6100- 4500 = 18100

15,600 15,750 16,820 18,100 19,200

453
In 1995, Division A of Company X had 4850 customers. If there were 86 service
errors in Division A that year, what was the service-error rate, in number of
service errors per 100 customers, for Division B of Company x in 1995?

(1) In 1995 the overall service error rate for Division A and B combined was
1.5 service errors per 100 customers.
(2) In 1995 Division B had 9,350 customers, none of whom were customers
of Division A.

(algebra, ratios, hard)

Suppose that B had c customers and made n service errors in 1995. We are
asked for 100n/c. To answer this question, we must be able to find n/c

(1) We don’t know whether A and B share customers. Even if we knew that
they did not, we would get (86 + n )/(4850 + c ) =1.5/100, anequation
that does not give us the means to find the ratio of n to c. NOT SUFF

(2) We are told that c=9,350. Nothing is said about the value of n.
NOT SUFF

(T) We can subsitute the value of c into the equation in (1) and find the
value of n, as we know that A and B do not share customers. SUFF

Also, note that A’s service error rate is is 8600/4850 approx. 1.7. We know that
if A and B do not share customers and the combined service error rate was 1.5,
B’s must be lower than 1.5. How much lower depends on the weight of B in the
overall picture, information that (2) provides.

454
In the addition table shown above, what is the value of m + n ?

-19 4 5 6 22

(algebra, medium)

Note that x + 4 = 1 and x + e = 7. Thus e – 4 =6 i.e. e = 4 + 6.


Therefore any entry in row 2 is 6 greater than the entry right above it
n= 1 and m= 4, so m + n = 5.

-19 4 5 6 22

455
In the arithmetic sequence t1 , t2 ,..., tn ,... t1=23 and tn= tn-1 - 3 for each n >
1. What is n when tn = -4?

-1 7 10 14 20

(sequences, medium)

The key to doing this type of questions quickly is to try to express the nth term
in terms of n, although writing out the first few terms is faster in this case.
t1= 23, t2= 23 – 3 , t3 = 23 – 3(2) ,..., tn=23 – 3(n – 1)
Thus if tn = - 4, 3(n – 1)=27 and n=10

-1 7 10 14 20

456
In the circle above, PQ is parallel to diameter OR, and OR has length 18. What
is the length of minor arc PQ?

(geometry, hard)

Since PQ || OR, the degree measure of QPR is equal to the degree measure of
angle PRO = 35º Since O, R and P are on the circle with center C, the degree
measure of angle PCO is twice that of angle PRO. Likewise, the degree measure
of angle QCR is twice that of angle QPR. Thus the degree measure of angle
PCQ is (180 – 70 – 70)º = 40º . Therefore the length of minor arc PQ is 1/9 of
the circumference of the circle, which is 18 .

457
In the coordinate plane above, what is the area of rectanglar region MNRS ?

(1) M has coordinates (2,1)


(2) N has coordinates (2,5)

(coordinate geometry, medium)

The combination of (1) and (2) reveals that the vertical dimension of the
rectangle is 4, no information is given about the horizontal dimension.

(T) NOT SUFF

458
In the decimal representation of x, where 0 < x < 1, is the tenths digit of x
greater than 0?

(1) 16x is an integer.


(2) 8x is an integer.

(algebra, medium)

(1) This means for some integer k such that 0 < k < 16, x=k/16. If k=1, the
tenths digit of x is 0 since 1/16 < 1/10. However, if k=2, the tenths digit
is not 0 since 2/16 > 1/10. NOT SUFF
(2) This means that x= m/8 for some integer m, where 0 < m < 8. For
every possible value of m 1/10 < x < 1, so the tenths digit of x will be
greater than 0. SUFF

459
In the figure above, equilateral triangle ABC is inscribed in the circle. If the
length of the arc ABC is 24, what is the approximate diameter of the circle?

5 8 11 15 19

(geometry, medium)

The triangle breaks the circular region into 3 identical sections, so arc ABC
constitutes 2/3 of the entire circumference. The circumference, then is 36. As
the circumference is  multiplied by the diameter. Thus the diameter is 36/  ,
slightly more than 10.

5 8 11 15 19

460
In the figure above, O is the center of the circle. If the area of the shaded
region is 2 , what is the value of x?

45/2 30 45 60 90

(geometry, medium)

The area of the entire circle is . Since the area of the shaded region
is 1/8 of that of the entire circle, x = 360/8 = 45

45/2 30 45 60 90

461
In the figure above, triangle ABC is inscribed in a semi-circle. If AB = 6 and BC
= 8, what is the length of arc ABC ?

15 12 10 7 5

(geometry, hard)

Note that the measure of angle AOC (180º) is twice that of angle ABC, which is
thus 90º. AC is therefore the hypotenuse of right trangle ABC and AC2 = AB2 +
BC2. AC (the diameter of the circle through A, B and C ) is thus 10 and the
cirucumference of the entire circle is 10 . The length of arc is half of this, 5 .

15π 12π 10π 7π 5π

462
In the figure above, AB = AE = 8, BC = CD = 13, and DE = 2. What is the area
of region ABCDE ?

76 84 92 100 108

(geometry, hard)

76 84 92 100 108

463
In the figure above, if x and y are each less than 90º, and PS || QR, is the
length of line segment PQ less than the length of line segment SR?

(1) x>y
(2) x+y >90

(geometry, medium)

(1) As PS is parallel to QR, we can translate line segment PQ to the right so


that S and P coincide and a triangle SRQ’ is formed, where PQ and SQ’
have the same length. Since x is less than 90, SR is the longest side of
the triangle and thus PQ = SQ’ < SR. SUFF
(2) No information is given as to whether x < y, x > y or x=y. NOT SUFF

464
In the figure above, is quadrilateral PQRS a parallelogram?
(1) The area of ΔPQS is equal to the area of ΔQRS.
(2) QR = RS

(geometry, medium)

(T) (1) tells us that the two triangles have the same height, and (2) tells us
that ΔQRS is isosceles. P could be any point along the dotted live above.
NOT SUFF

465
In the figure above, lines l1 and l2 are parallel. What is the value of x ?

(1) y = 87
(2) z = 93

(geometry, medium)

Since the two lines are parallel, x= z. Also x + y = 180

(1) SUFF
(2) SUFF

466
In the figure above, points A, B, C, D, and E lie on a line. A is on both circles, B
is the center of the smaller circle, C is the center of the larger circle, D is on the
smaller circle, and E is on the larger circle. What is the area of the region inside
the larger circle and outside the smaller circle?

(1) AB = 3 and BC =2
(2) CD =1 and DE = 4

(geometry, medium)

We are asked to find the value of , where R and r are the radii of the
larger and smaller circle, respectively. If we can find the value of R and r, we
can answer the question.

(1) r = 3 and R – r = 2 SUFF


(2) 2r – R = 1 and 2R – 2r =4 SUFF

467
In the figure above, points P (- 3 ,1) and Q (r,s) lie on the circle with center
O. What is the value of r ?

1/2 1 2 3 2 /2

(geometry, hard)

When we consider right triangle PSO, PS:OS =1: 3 , so the angle at O is 60º
and the angle at P is 30º. Therefore, in right triangle QOT, the angle at O is 30º
and the angle at Q is 60º. Since OR and OQ are radii of the circle OP=OQ and
the two triangles are congruent. Thus the coordinates of Q are (1, 3 ).

1/2 1 2 3 2 /2

468
In the figure above, what is the value of x ?

55 60 65 70 75

(geometry, hard)

The quickest way to find the value of x is to remember the dimensions of the
right triangles whose side lengths are all integers: 3-4-5 , 5-12-13, and 7-24-25.
We see then, that the triangle in the center, has two sides of length 5 and is
thus an isosceles triangle, with two angles whose measure is 55º. Since the
sum of the three interior angles of any triangle is 180º, the measure of the
missing angle in degrees is 180 – 2(55) = 70. Thus x = 70

55 60 65 70 75

469
In the figures above, if the area of the triangle on the left is twice the area of
the area of the triangle on the right, then in terms of s, S=

2 3
s s 2s 3s 2s
2 2

(geometry, hard)

These are similar triangles, so T/t = S/s. Also, because the inner triangles are
similar, T/t= H/h. The area of the large triangle, SH/2 is twice the area of the
small triangle, sh/2. Thus S/s =2h/H=2s/S. This means that S / s  2 .
Alternatively, look at s simple case- imagine that each triangle is a right
isosceles triangle. The problem becomes much easier!

2
2
s
2
3
s 2s 3s 2s

470
In the figure shown, point O is the center of the semicircle and point points B, C
and D lie on the semicircle. If the length of line segment AB = OC, what is the
degree measure of angle BAO?

(1) angle COD = 60 degrees.


(2) angle BCO = 40 degrees.

(geometry, hard)

If we suppose that the measure of angle BAO is xº, using the properties of
isosceles triangles, we can deduce the measure of the angles above.

(1) If angle COD is 60º, angle AOC is 120º. Thus 180 -3x =120 and x=20
SUFF
(2) If angle BCO is 40º, x=20 SUFF

471
In the figure shown, the length of line segment QS = 4 3 . What is the
perimeter of equilateral triangle PQR ?

12 12 3 24 24 3 48

(geometry, hard)

Note that QS is a perpendicular bisector of PR. In other words, QS divides


equilateral triangle PQR into two congruent right triangles, triangles PSQ and
RSQ. Thus the length of PS is one half the length of PR, which has the same
length as does PQ. Therefore, if the length of line segment of PQ is x, that of
the line segment PS = x/2 and the Pythagorean Theorem tells us that

 
2
 x 2 3x 2
x2     4 3   423  x 2  43  x  8
2 4
The perimeter of PQR is 3 times the length of PQ, so the perimeter is 24.

Alternatively, recognizing that triangle PQS is a right triangle and angle QPS is
2
60º, we see that |PQ| : |PS| : |QS| = 2 : 1 : 3 and |PQ| = | QS | =8.
3
12 12 3 24 24 3 48

472
In the figure shown, what is the value of x?

(1) The length of line segment QR is equal to the length of line segment RS.
(2) The length of line segment ST is equal to the length of line segment TU.
(geometry, hard)

Note that (1) says nothing about U and (2) says nothing about Q, so neither
statement is sufficient by itself.

Combining (1) and (2) and using the properties of all triangles and isosceles
triangles in particular, we see that x=45. SUFF
(T) SUFF

473
In the figure shown, the measure of angle PRS is how many degrees greater
than the measure of angle PQR ?

(1) The measure of angle QPR is 30 degrees.


(2) The sum of the measures of angles PQR and PRQ is 150 degrees.

We are asked for the value of x – y

(1) If angle QPR is 30º, we know that y + 30 + 180 – x = 180, and we can
find the value of x – y. SUFF
(2) tells us (1) SUFF
474
In the figure, what is the area of triangle of triangular region BCD ?

8 16

(geometry, medium)

Since is an isosceles triangle, BD = . Thus , since BC = 4, the required


area is (BC x BD)/2 =

8 16

475
In the fraction x , where x and y are positive integers, what is the value of y ?
y

(1) The least common denominator of x and 1 is 6 .


y 3
(2) x=1

(factors and multiples, hard)

(1) The least common common multiple of 3 and y is 6. Therefore, y is a


multiple of 2. y may or may not be a multiple of 3, however. Therefore
there are 2 possible values of y: 2 and 6. NOT SUFF
(2) Nothing is said or implied about y. NOT SUFF
(T) Therefore there are 2 possible values of y: 2 and 6. NOT SUFF

476
In the infinite sequence a1 , a2, a3, …an , … , each term after the first is equal to
twice the previous term. If a5 – a2 = 12, what is the value of a1 ?

4 24/7 2 12/7 6/7

(sequences, exponents, hard)

If the first term is x, the nth term is 2n-1 x.


Therefore, a5 – a2 =16x – 2x =14x = 12, and x=12/14=6/7

4 24/7 2 12/7 6/7

477
In the rectangular coordinate system above, the area of triangle PQR is what
fraction of the area of triangle LMN ?

1/9 1/8 1/6 1/5 1/3

(coordinate geometry, medium)

Consider point S(8,0). MS is an altitude of triangle LMN with base LN and QS is


an altitude of triangle PQR with base PR. Remember that the area of a triangle
is bh/2 , where b is the length of the base and h is the height or altitude.
Triangle PQR has a base of 4 units, which is 1/3 of the base of triangle LMN.
Triangle PQR has a height of 4 units, 1/3 of the height of triangle LMN. Thus
the area of triangle PQR is 1/3 x 1/3= 1/9 of the area of triangle LMN.

1/9 1/8 1/6 1/5 1/3

478
In the rectangular coordinate system above, the area of triangular region PQR
is

12.5 14 10 2 16 25
(coordinate geometry, hard)

Perhaps the easiest way to proceed is to see whether two of the line segments
that make up the triangle are perpendicular. The slope of line segment PQ is
y y 3
mPQ  P Q  . Likewise, the slopes of line segments QR and PR are 1/7
xP  xQ 4
and 4/3, respectively. Since the product of the slope of line segments PQ and
PR is -1, these two line segments are perpendicular and triangle PQR is a right
triangle, whose area is one half the product of the lengths of line segments PQ
and PR. The Pythagorean theorem tells us that the length of PQ =
( yP  yQ ) 2  ( xP  xQ ) 2  9  16  5 . Likewise, the length of PR is 5. Thus the
area of the triangle is 25/2 = 12.5.

Another way to approach this problem is to notice that the area of the triangle
is simply the area of the rectangle with diagonal OR, 28, less the areas of three
right triangles, one of which is triangle OPQ:

12.5 14 10 2 16 25

479
In the rectangular coordinate system shown above, does line K intersect
quadrant II?

(1) The slope of the line is -1/6.


(2) The y-intercept of K is -6.

(coordinate geometry, medium)

The line intersects quadrant II if the slope if the line is negative, regardless of
its y-intercept. It also intersects quadrant II if the y-intercept is positive,
regardless of its slope.

(1) As the slope of the line is negative, it intersects quadrant II. SUFF
(2) The y-intercept is negative. If its slope negative, it will intersect quadrant
II. If the slope is not negative, it will not intersect quadrant II. (2) tells
us nothing about the slope of the line. NOT SUFF

480
In the rectangular solid above, the three sides shown have areas 12, 15, and
20, respectively. What is the volume of the solid?

60 120 450 1,800 3,600

(geometry, hard)

Suppose that the lengths of the edges are a b and c. Then the volume of the
solid is the product abc. ab = 12 , bc = 15 and ac = 20.
Since (ab)(bc)(ac) = (abc)2 =3 x 4 x 3 x 5 x 4 x 5 =(3 x 4 x 5)2
Thus abc = 3 x 4 x 5 = 60

60 120 450 1,800 3,600

481
In the sequence of positive numbers x1 , x2, x3, … , what is the value of x1 ?

(1) x j = xj-1/2 for all integers j > 1


(2) x5 = (x4)/(x4+1)

(sequences, hard)

Neither (1) nor (2) is sufficient by itself: (1) only gives us the ratio between any
two consecutive terms, whereas (2) does not even mention the first term.

(1) and (2) Suppose x1= k >0. (1) tells us that xn= k/2n-1. Thus x5 = k/16
and x4= k/8 and (x4)/(x4+1)= k/(k+8), which is equal to x5 = k/16, according
to (2). Thus k=x1 = 8 SUFF

482
a1, a2, a3, ... , a15

In the sequence shown, an = an-1 + k, where 2 ≤ n ≤ 15 and k is a non zero


constant. How many terms of the sequence are greater than 10 ?

(1) a1 = 24
(2) a8 = 10

(sequences, hard)

(1) If the first term is 24, the number of terms that are greater than 10 may
be as high as 15, ( for example if k > 0 ), and as low as 1 (if k = -14, for
example). NOT SUFF
(2) The eighth terms of the sequence is 10.
If k < 0, a1 > a2 > ... a7 > a8 = 10 > a9 > a14 > a15 . Thus seven terms
are greater than 10.
If k > 0, a1 < a2 < ... a7 < a8 = 10 < a9 < a14 < a15. Thus seven terms
are greater than 10. Therefore the number of terms greater than 10 is 7
regardless of the value of k, a non zero constant. SUFF

483
In the sequence 1, 2, 4, 8, 16, 32, …, each term after the first is twice the
previous term. What is the sum of the 16th, 17th, and 18th terms of the
sequence?
18 17 16 16 15
2 2 7(2 ) 3(2 ) 7(2 )

(sequences, exponents, medium)

Note that the nth term of the sequence is 2n-1 . Thus the sum of the 16th, 17th,
and 18th terms of the sequence is 215 + 216 + 217 = 215 ( 1 + 2 + 22 ) =
15
7(2 ).

18 17 16 16 15
2 2 7(2 ) 3(2 ) 7(2 )

484
In the xy-coordinate plane, line L and line K intersect at point (4,3). Is the
product of their slopes negative?

(1) The product of the x-intercepts of lines L and K is positive.


(2) The product of the y-intercepts of lines L and K is negative.

(coordinate geometry, hard)

Suppose lines L and K have x-intercepts xL and xK and y-intercepts yL and yK.
By drawing, one can see that only taken together do the statements suffice.

(1) xLxK > 0. The slope of L is 3/(4 - xL) and that of K is 3/(4 – xK). Note
that the sign of the product of the slopes is the same as the sign of
(4 - xL) (4 – xK). Note that if xL = 2 and xK =5, the product of the slopes
will be negative, but if xL=6 and xK =5, the product of the slopes will be
positive. NOT SUFF
(2) yLyK < 0. The slope of L is (4-yL)/3 and that of line K is (4-yK)/3. As in
(1), the product of the slopes can be positive e.g. (yL,yK)=(2,-2) or
negative e.g. (yL,yK)= (6,-2) NOT SUFF

(T) The slope of line K is -yK/xK and that of line L is –yL/xL. Since xLxK > 0
and yLyK < 0, the product of the slopes must be negative. SUFF

485
In the xy plane, at what 2 points does the graph of y = (x+a)(x+b) intersect at
the x axis?

(1) a+b=-1
(2) The graph intersects the y axis at (0,-6)

(coordinate geometry, algebra, medium)

The two x-intercepts are a and b.

(1) An infinite combination of numbers yield a sum of -1 . NOT SUFF


(2) ab= -6. An infinite combination of numbers yield a product of -6.
NOT SUFF
(T) The two intercepts are -3 and 2. SUFF

486
In the xy -plane, does the line with equation y=3x + 2 contain the point (r,s) ?

(1) (3r +2-s )( 4r +9-s ) = 0


(2) ( 4r -6-s )( 3r +2-s ) = 0

(coordinate geometry, algebra, medium)

In other words, is s=3r + 2 ?

(1) tells us that s= 3r + 2 or s=4r + 9. NOT SUFF


(2) tells us that s=4r – 6 or s = 3r + 2 NOT SUFF
(T) s = 3r + 2 SUFF

487
In the xy-plane, line k passes through the point (1,1) and line m passes
through the point (1,-1). Are lines k and m perpendicular to each other?

(1) Line k and m intersect at the point (1,-1).


(2) Line k intersects the x-axis at the point (1,0).

(coordinate geometry, medium)

(T) Lines k and m are perependicular to each other if and only if one of
these lines is vertical and the other horizontal or the product of their
slopes is -1. As the only information given about line m is that it passes
through the point (1, -1), we have no means of calculating its slope.
NOT SUFF

488
In the xy-plane, the line with equation ax + by + c = 0 where abc ≠ 0 has
slope 2/3. What is the value of b?

(1) a=4
(2) c=-6

(coordinate geometry, medium)

Remember that for a line y = mx + b, the slope is m.


y = (-ax – c)/b, so the slope is –a/b. If we can find the value of a, we can
answer the question.

(1) SUFF
(2) NOT SUFF

489
In the xy-plane, region R consists of all the points (x,y) such that 2x + 3y ≤ 6.
Is the point (r,s) in region R?

(1) 3r + 2s = 6
(2) r ≤ 3 and s ≤ 2

(inequalities, coordinate geometry, hard)

(r,s) is region R if and only if 2r + 3s ≤ 6.

(1) 3r + 2s = 6. If r > s, s – r < 0


Given 3r + 2s = 6 and s – r < 0, adding we get that 3r + 2s < 6
However, if r < s, 3r + 2s > 6. As we don’t know whether r is greater
than or equal to s, we cannot answer the question. NOT SUFF
(2) If r = 3 and s=3, 2r + 3s > 6, but if r=s=0, 2r + 3s < 6 NOT SUFF
(T) As (2) does not tell us whether r is greater than or equal to s, we cannot
answer the question. NOT SUFF

490
In the xy-plane, the line k passes through the origin and through the point
(a, b), where ab ≠ 0. Is b positive ?

(1) The slope of line k is negative.


(2) a<b

(coordinate geometry, medium)

The slope of this line is b/a.

(1) If b/a < 0, b and a have different signs. b is positive if a is negative and
negative if a is positive. Since we don’t know the sign of a, we cannot
deduce the sign of b. INSUFF

(2) b and a could be both negative or both positive. Also a and b could have
different signs, in which case b would be positive. INSUFF
(T) (1) says that a and b have different signs, so b must be positive,
according to (2). SUFF

491
In the xy-plane, the sides of a certain rectangle are parallel to the axes. If one
of the vertices of the rectangle is (-1,-2), what is the perimeter of the
rectangle?

(1) One of the vertices of the rectangle is (2, -2).


(2) One of the vertices of the rectangle is (2, 3).

(coordinate geometry, medium).

(1) gives us the length of only one side NOT SUFF


(2) gives us the length and width of the rectangle (5 and 3) SUFF

492
In the xy coordiante plane, the slope of line L is 3/4. Does line L pass through
the point (-2/3, 1/2 ) ?

(1) Line L passes through the point (4,4)


(2) Line L passes through the point (-4, -2)

(coordinate geometry, medium)

As the slope of the line is given, each statement, which gives a point on the
line, is sufficient. All that has to be done is see whether the slope of the line
segment from (-2/3,1/2) to the given point is 3/4, the slope of the line.

(1) SUFF
(2) SUFF

493
In triangle ABC above, what is the length of side BC?

(1) Line segment AD has length 6.


(2) x = 36

(geometry, hard)

Using the properties of all triangles and isosceles triangles in particular, AD=BC

(1) SUFF
(2) No information about length of given. NOT SUFF

494
Is m + z > 0?

(1) m - 3z > 0
(2) 4z - m > 0

(inequalities, medium)

(1) m > 3z. z and m could both be positive, in which case m + z > 0.
Alternatively, m and z could both be negative, in which case m + z < 0.
NOT SUFF
(2) m < 4z. Similarly, m and z could both be positive or negative.
NOT SUFF
(T) 3z < m < 4z, thus z > 0. (1) tells us that m is positive as well, and thus
m + z > 0. SUFF

495
Is n + n2 + n3 > 0 ?

(1) n>0
(2) 0 < n2 < 9

(exponents, inequalities, medium)

Note that for all values of n, either |n| or |n3| is greater than or equal to n2.
Thus n +n2 + n3 will be negative if n is negative and positive if n is positive.
Therefore, we need to know whether n > 0.

(1) SUFF
(2) n can be any number between -3 and 0 or between 0 and 3. NOT SUFF

496
Is the average (arithmetic mean) of 5 different positive integers at least 30?

(1) Each of the integers is a multiple of 10.


(2) The sum of the five integers is 160.

(factors and multiples, medium)

In other words, is the sum the 5 different integers at least 30(5) = 150?

(1) Suppose the integers are a1 < a2 < a3 < a4 < a5


Each of these integers are positve multiples of 10, so a1 > 10 ,
a2 > 20, a3 > 30, a4 > 40 and a5 >50. Thus the sum of the 5 different
integers is greater than 150. SUFF
(2) SUFF

497
Is the hundredths digit of the decimal d greater than 5?

(1) The tenths digit of 10d is 7.


(2) The thousandths digit of d/10 is 7.

(exponents, medium)

Suppose that decimal d = 0.xyz , where y is the hundredths digit. We are asked
whether the value of y is greater than 5.

(1) 10d = x.yz , the tenths digit of which is y. SUFF


(2) d/10 = 0.0xyz, the thousandths digit of which is y. SUFF

498
Is integer n a multiple of 15?

(1) n is a multiple of 20.


(2) n + 6 is a multiple of 3.

(factors and multiples, medium)

For n to be a multiple of 15, it must be a multiple of both 3 and 5.


(1) n is a multiple of 5 and 4. NOT SUFF
(2) n is 6 less than a multiple of 3 and thus a multiple of 3. NOT SUFF
(T) SUFF

499
Is 1/(a-b) < b – a ?

(1) a < b
(2) 1 < |a – b|

500
Is the measure of one of the interior angles of quadrilateral ABCD equal to 60
degrees?

(1) Two of the interior angles of ABCD are right angles


(2) The degree measure of ABC is twice the degree measure of angle BCD.

(geometry, hard)

Suppose that the four interior angles are w, x , y and z. w + x + y + z =360

(T) (w,x,y,z) could be (60,120,90,90) or (45,90,90,135) NOT SUFF

501
Is the positive integer n an odd integer?

(1) n+4 is a prime number.


(2) n+3 is not a prime number.

(factors and multiples, medium)

(1) As n+4 > 4 and n+4 is a prime number, n+4 must be an odd number,
and therefore n is odd as well. SUFF
(2) As n + 3 is not a prime number. If n + 3= 9, n=6, an even number.
If n + 3 =10, n=7, an odd number. NOT SUFF

502
Is positive integer n divisible by 3?

(1) n2/36 is an integer.


(2) 144/ n2 is an integer.

(factors and multiples, hard)

(1) tells us that the square of n is a multiple of 36. Since 36 is a multiple of


3, the square of n and thus n itself are multiples of 3. SUFF
(2) n must be a divisor of 12. However, some but not all divisors of 12 are
divisible by 3. For example, n could be 1, which is not divisible by 3, or 3
itself. NOT SUFF

503
Is | x - y| >|x| - |y|?

(1) y< x
(2) xy < 0

(inequalities, hard)

(1) If y < x , |x – y| > x – y

If x > 0 and y > 0 |x| - |y| = x – y NO


If x > 0 and y < 0 |x| - |y| = x –(-y) = x + y YES NOT SUFF

(2) If xy < 0 , either x < 0 < y (case i) or y < 0 < x (case ii)

Remember that |x - y | = x - y if x > y but |x - y| = y - x if x < y


Also keep in mind that |x| = x if x > 0 but |x| = - x if x < 0

In case i , |x - y| = y - x and |x| - |y| = - x - y


Since y > 0 , y > - y and thus y - x > -x - y (answer yes)

In case ii, |x - y| = x - y and |x| - |y| = x + y


Since y < 0 , - y > y and thus x - y > x + y (answer yes) SUFF

Alternatively, if x and y have opposite signs, x and - y have the same sign. x +
(- y) will thus be farther from 0 than is x

Thus |x - y| > |x| > |x| - |y| since |y| > 0 SUFF

One more alternative!

If |x| - |y| < 0 , the answer is obviously yes, as | x - y| > 0 ( x and y are not
the same number, as they have opposite signs.
If |x| - |y| >= 0 the answer will be yes if and only if the square of the left side
is greater than the square of the right
i.e |x - y|^2 = (x - y)^2 = x^2 - 2xy + y^2 > (|x| - |y|)^2 = x^2 - 2|xy| +
y^2
But since we know that xy < 0 xy < 0 < |xy| , thus -2xy > -2|xy| and the
inequality above holds.

SUFF

504
Is the integer k divisible by 4?

(1) 8k is divisible by 16.


(2) 9k is divisible by 12.

(factotr and multiples, medium)

(1) If 8k is divisible by 16. 8k is a multiple of 16.


Thus k is a multiple of 2. k may or may not be a multiple of 4.
NOT SUFF
(2) If 9k is a multiple of 12, 3k is a mutliple of 4. Since 3 is a prime number,
k must be a multiple of 4. SUFF

505
Is the integer n odd?

(1) n is divisible by 3.
(2) 2n is divisible by twice as many positive integers as n.

(factors and multiples, hard)

(1) Half of the multiples of 3 are odd and the other half are even.
NOT SUFF
(2) Clearly n is not equal to 0, as in this case n=2n. If n is positive, we can
express n = 2k x, where x is an odd integer and k is an integer greater
than or equal to 0. If k = 0 , n is odd, whereas if k > 0, n is even. If x
has d divisors, n has (k + 1)d divisors, and 2n = 2k+1 x has (k + 2)x
divisors.
Thus k + 2 = 2 (k +1) i.e k = 0. Therefore n is odd. SUFF

Alternatively, if n is even, it has at least two divisors, some of them even.


Let D be set of the divisors of n and E be the set the different possible
values of 2 times a divisor of n. D and E have the same number of
elements. All divisors of 2n are either in D or E. However, D and E have
at least one element in common, 2. Therefore 2n does not have twice as
many divisors as n, contrary to (2). Thus n cannot be even- it must be
odd

506
Is the integer x divisible by 6 ?

(1) x + 3 is divisible by 3.
(2) x + 3 is an odd number.

(factors and multiples, medium)

(1) x is 3 less than a multiple of 3 and is thus a multiple of 3. If x is even, x


is a multiple of 6. However, if x is odd, x is not a multiple of 3.
NOT SUFF
(2) x is 3 less than an odd number and is thus an even number. Some but
not all even numbers are multiples of 6. NOT SUFF
(T) Sufficient

507
Is the number of members of Club X greater than the number of members of
Club Y ?

(1) Of the members of Club X, 20 percent are also members of Club Y.


(2) Of the members of Club Y, 30 percent are also members of Club X.

(sets, medium)

(T) As neither (1) nor (2) gives us information about the number of people
who are members of only one of the two clubs, neither statement is by
itself sufficient. Suppose X and Y have x and y members respectively.
Together the two statements tell us that 0.2x = 0.3y (the number of
people who are members of both clubs), so x/y =3/2 SUFF

508
Is the number of seconds required to travel d feet at r feet per second greater
1 1
than the number of seconds required to travel d feet at r feet per second?
2 2

(1) d is 30 greater than d .


1 2
(2) r is 30 greater than r .
1 2

(movement, medium)

Remember that distance (d) = velocity (v) × time (t), so t=d/v.


Thus we are asked whether d / r > d / r
1 1 2 2
(T) We know the the first ratio is closer to 0 than the second, but no
information is given whether the second ratio (the number of seconds required
to travel d feet at r feet per second) is greater or less than 1. NOT SUFF
2 2

509
Is the standard deviation of the salaries of Company Y’s employees greater
than the standard deviation of the salaries of Company Z’s employees?

(1) The average (arithmetic mean) salary of Company Y’s employees is


greater than the average salary of Company Z’s employees.
(2) The median salary of Company Y’s employees is greater than the median
salary of Company Z’s employees.

(statistics, medium)

(T) The standard deviation has to do with how far apart the salaries are, and
no information of this sort is given. NOT SUFF

510
Is the sum of the integers x and y a prime number?

(1) x is an even prime number.


(2) y is a prime number between 10 and 20.

(factors and multiples, medium)

(1) x =2 NOT SUFF


(2) y {11,13,17,19} NOT SUFF
(T) x+y {13,15,19,21}
As 13 and 19 are prime numbers but 15 and 21 are not, we cannot say
for sure whether x + y is a prime number. NOT SUFF

511
Is the three-digit number n less than 550?

(1) The product of the digits in n is 30.


(2) The sum of the digits in n is 10.

(factors and multiples, hard)

(1) As 30= 5 x 3 x 2 = 5 x 6 x 1, n could be any of 12 numbers ranging


from 156 and 651. NOT SUFF
(2) n could be as little as 118 or as high as 811. NOT SUFF
(T) The three digits of n must be 5, 3 and 2, not necessarily in that order.
Whatever the value of n, n ≤ 532 and thus n < 550. SUFF

512
Is t  w ?
3 5
(1) w>t
(2) 5t > 3w

(inequalities, medium)

In other terms, is w < 5t/3 ?

(1) If w and t are both positive, that w > t (i.e. w/t > 1) sheds no light on
whether w < 5t /3 (i.e. w/t < 5/3). INSUFF
(2) If we divide both sides of the given inequality 3w < 5t by 3, we see
that w < 5t/3. SUFF

513
Is x an odd integer?

(1) x + 3 is an even integer.


(2) x/3 is an odd integer.

(factors and multiples, medium)

(1) x + 3 = 2k , where k is an integer. Thus, since 2k – 3 is odd, x is an odd


integer. SUFF
(2) x/3 = 2m + 1 , where m is an integer. Thus x = 6m + 3, which is an odd
integer. SUFF

514
Is x  y 2 ?

(1) x  y4
(2) x y

(exponents, hard)

Remember that if 0<y<1, y4 < y2 < y and if y≥1, y < y2 < y4

(1) If y≥1, x > y4 > y2, but if 0<y<1, y4<y2, so x could be less than y2
NOT SUFF
2
(2) If 0<y<1, x  y >y , but if y > 1, y < y2, so x could be less than
y2 . NOT SUFF
2
(T) x > y for every possible value of y (note that (2) implies that y cannot
be negative. SUFF

515
Is |x| < 1?

(1) |x + 1| = 2 |x – 1|
(2) |x – 3| ≠ 0

(inequalities, hard)

We are asked whether x is less than 1 unit from 0, i.e. whether -1 < x < 1.

(1) x is twice as far from -1 as it is from 1. This means that either x is 2/3 of
the way from -1 to 1 or that 1 is halfway between –1 and x. So x could
be between -1 and 1 or that (x – 1)/2= 1 (i.e. x=3). NOT SUFF
(2) x ≠ 3 NOT SUFF
(T) x must be between -1 and 1 SUFF

Other ways to simply (1) are to square both sides of the equation and to
consider three cases: x < - 1, -1 < x < 1 , and x > 1

516
Is -3x3 = -3 ?

(1) -3x < 3


(2) x <0

(inequalities, medium)

The question can be simplified: Is x3 = 1 or is x = 1 ?

(1) Dividing both sides by -3 (and remembering to change the inequality


sign), we see that x > -1. The value of x may or may not be 1.
NOT SUFF
(2) If x is negative, it cannot be 1. SUFF

517
Is ?

(1) x<3
(2) x>1

(inequalities, hard)

if and
if

Since , for x must be a positive


number different from 3.

(T) SUFF

518
Is xy > x/y?

(1) xy > 0
(2) y<0

(inequalities, medium)

(T) If xy > 0 , xy > x/y if and only if 1 > 1/y2 i.e. y2 > 1. As (2) tells us
that y < 0, the inequality holds if and only if y < -1. However, no
information is given about the magnitude of y. NOT SUFF

519
Is x+y < 1?

(1) x < 8/9


(2) y < 1/8

(inequalities, ratios, medium)

Neither statement is sufficient as we need to know about both variables.


Combining (1) and (2), we can deduce that x + y < 64/72 + 9/72 = 73/72, but
as 73/72 > 1, x + y may or may not be less than 1. NOT SUFF

520
Is x  52 5 x ?

(1) -x|x| < 0


(2) 5–x>0

(inequalities, hard)

|w| = w if w ≥ 0 and |w| = - w if w ≤ 0. Thus x  52  5  x  ( x  5)


if and only if x – 5 ≤ 0 , i.e x ≤ 5.

(1) Dividing both sides by |x| yields that – x < 0, in other words x > 0. x
may or may not be less than or equal to 5. NOT SUFF
(2) tells us that x < 5 SUFF

521
Is x > 0 ?

(1) x2 = 2x
(2) x3 = 3x

(algebra, medium)

(1) x2 = 2x implies that x2 – 2x x ( x – 2) = 0, so x is either 0 or 2.


Thus x may or may not be positive NOT SUFF
3 3 2
(2) x = 3x implies that x - 3x = x (x – 3) = 0, so x is either 0 or
 3 NOT SUFF
(T) (2) tells us that x cannot be 2, so it must be 0. SUFF

522
Is x > 0.05 ?

(1) x > 3/40


(2) x is greater than 5% of 3.

(percents, medium)

(1) x > 3/40 > 2/40 =0.05 SUFF


(2) x > 0.05(3) > 0.05 SUFF

523
Is xy + xz = 0 ?

(1) x= 0
(2) y+z=0

(algebra, medium)

Note that xy + xz = x ( y + z ) = 0.

(1) SUFF
(2) SUFF

524
Is x = y ?

(1) x2 + y2 = 2xy
(2) x2 + y2 = 0

(algebra, medium)

(1) x2 + y2 - 2xy = 0, so (x – y)2 = 0. Thus x=y SUFF


(2) Since x2 and y2 are both at least 0, x2 = y2 = 0.
Thus x=y=0 SUFF

525
Is x – y + 1 greater than x + y – 1 ?

(1) x>0
(2) y<0

(inequalities, medium)

Note that x – y + 1 > x + y – 1 if and only if 2 > 2y i.e. y < 1.

(1) NOT SUFF


(2) y<0<1 SUFF

526
Is x4 + y4 > z4 ?

(1) x2 + y2 > z2
(2) x+y>z

(exponents, algebra, hard)

(1) If x2 + y2 > z2, by squaring both sides, we see that x4 + y4 +2x2y2 > z4
As 2x2y2 > 0, we are unable to determine whether x4 + y4 > z4. For
example, if x2 = y2 = 10 and z2=19, x4 = y4 = 100 and z4 =192 so 200=
x4 + y4 < z4 = 192. Of course, z2 could be 0, in which case x4 + y4 > z4 .
NOT SUFF

(2) Using the same examples, if x=y = 10 and z= 19 ,


x + y > z but x4 + y4 < z4
(T) NOT SUFF

527
Is (x+1)/(x-3) < 0 ?

(1) -1< x < 1


(2) x2 – 4 < 0

(inequalities, hard)

Note that a/b is negative if and only if a and b have opposite signs. In this case,
for (x+1)/(x-3) to be negative, x + 1 must be positive and x – 3 negative. Thus
the question can be rephrased as follows: is -1 < x < 3 ?

(1) SUFF
(2) x2 < 4 i.e. |x| < 2 in other words – 2 < x < 2 NOT SUFF

528
Is 2x - 3y < x2 ?

(1) 2x - 3y = -2
(2) x > 2 and y > 0

(inequalities, hard)

(1) 2x - 3y = -2 < 0 < x2 SUFF


(2) Note that for all values of x greater than 2, x2 – 2x = x ( x – 2 ) > 0, so
x2 > 2x. Also as y > 0, -3y < 0. Thus 2x – 3y < x2. SUFF

529
Is xy > 0 ?

(1) x – y > -2
(2) x – 2y < -6

(inequalities, hard)

xy > 0 if and only if x and y are both positive or both negative. All such points
(x,y) lie in quadrants I or III of the xy- coordinate plane.

(1) x>y–2. NOT SUFF


(2) x < 2y – 6 . NOT SUFF
Note that each of the inequalities in (1) and (2) is the region on one side of a
line. As these lines are neither horizontal nor vertical, these regions will span at
least three quadrants of the xy- coordinate plane.

(T) y – 2 < x < 2y – 6 ----- 4 < y. Thus x + 2 > y > 4 and x > 2. SUFF

530
Is z equal to the median of 3 positive integers x, y and z?

(1) x<y+z
(2) y=z

(statistics, medium)

(1) Nothing can be deduced about the order the three integers. (x,y,z) could
be (1,2,3) or (1,3,2) NOT SUFF
(2) If x ≠ z, y=z is the median. If x=z, x=y=z is the median. SUFF

531
It takes printer A 4 minutes more than printer B to print 40 pages. Working
together, the two printers can print 50 pages in 6 minutes. How long in minutes
will it take Printer A to print 80 pages?

12 18 20 24 30

(combined work, hard)

It takes printer A 1 minute more than printer B to print 10 pages. Also, to print
10 pages, it takes both printers 6/5 minutes. Suppose x is the time it takes A to
print 10 pages, Thus we can write:

1 1 5
 
x x 1 6

Knowing that 5/6 = 1/2 + 1/3 makes it clear that x=3. Since it takes printer A 3
minutes to print 10 pages, 80= 8 (10) pages will take 8 times as long, 24
minutes.

12 18 20 24 30

532
Jack and Mark both received hourly wage increases of 6 percent. After the
wage increases, Jack's hourly wage was how many dollars per hour more than
Mark's?

(1) Before the wage increases, Jack's hourly wage was $5 per hour more
than Mark's.
(2) Before the wage increases, the ratio of Jack's hourly wage to Mark's was
4:3.

(percents, ratios, medium)

(1) Suppose Mark’s pre-increase wage is m. Jack’s was m + 5. Their new


wages are 1.06m and 1.06(m+5). Jack’s new wage is 1.06(5) dollars per
hour more than Mark’s. SUFF
(2) Suppose their original wages are 4k and 3k. Their new wages are
4k(1.06) and 3k(1.06), so Jack’s new hourly wage is k(1.06) higher than
Mark’s. As we don’t know the value of k, we cannot answer the question.
NOT SUFF

533
Jason’s salary and Karen’s salary were each p% greater in 1998 that in 1995.
What is the value of p?

(1) In 1995 Karen’s salary was $2000 greater than Jason’s.


(2) In 1998 Karen’s salary was $2440 greater than Jason’s.

(percents, hard)

(T) Seperately, neither (I) or (II) is sufficient. Suppose Jason’s salary in 1995
was j, so that Karen’s salary in 1995 was j + 2000. Their salaries in 1998 weree
(1 +p/100)j and (1+ p/100)(j + 2000) = (1 +p/100)j +2000 + 20p.
Thus 2000 + 20p = 2440 and we can solve for p. SUFF

534
Joanna bought only $0.15 stamps and $0.29 stamps. How many $0.15 stamps
did she buy?

(1) She bought $4.40 worth of stamps.


(2) She bought an equal number of $0.15 stamps and $0.29 stamps.

(factors and multiples, hard)

(1) Thinking in cents, 440 has a units digit of 0, the number of 29 cents
must be a multiple of 5. The amount of money spent on 15 cent stamps
must be a multiple of 15:

Number of Value of 29 cent Value of 15


29 cent stamps stamps cent stamps
0 0 440
5 145 295
10 290 150
15 435 5

Only the purchase of 10-29 cent stamps would yield a value of the 15-cent
stamps that is a multiple of 15. Thus 150/15 = 10 15-cent stamps were
bought. SUFF

(2) Obviously not sufficient NOT SUFF

535
John and Mary own stock in a certain company. Does John own more shares
than Mary?

(1) Mary owns more than 500 shares of the company's stock.
(2) The number of shares of the company's stock that John owns is 400 less
than twice the number of shares of the company's stock that Mary owns.

(algebra, medium)

(1) Nothing is said about John’s holdings. NOT SUFF


(2) j = 2m – 400, where j and m represent how many stocks John and Mary
hold. j > m if and only if m < 2m – 400 i.e if and only if m > 400. No
information is given about Mary’s holdings, however. NOT SUFF
(T) (1) tells us that m is greater than 500 > 400, so (2) tells us that John
own more shares than Mary. SUFF

536
Joshua and Jose work along with 4 more workers. For a survey, 2 out of 6
workers are chosen randomly. What is the probability that both Joshua and
Jose are chosen?

1/15 1/12 1/9 1/6 1/3

(probability, medium)

There are 6C2 = 15 equally probable groups of two workers, one of which
contains both Joshua and Jose.

1/15 1/12 1/9 1/6 1/3

537
Juan bought some paperback books that cost $8 each and some hardcover
books that cost $25 each. If Juan bought more than 10 paperback books, how
many hardcover books did he buy?

(1) The total cost of the hardcover books that Juan bought was at least
$150.
(2) The total cost of all the books that Juan bought was less than $260.

(inequalities, hard)

(1) Juan bought at least 150/25 =6 hardcover books. NOT SUFF


(2) Juan spent more than 10(8)=80 dollars on paperback books.
NOT SUFF

(T) Juan must have bought exactly 6 hardcover books, as if he had bought
more than 6, he would have spent at least $175 + $88 > $260. SUFF

538
Deli A Deli B
American $15/kg $16/kg
Provolone $17/kg $18/kg
Swiss $19/kg $20/kg

Julian bought 6 kg of cheese from each Deli A and Deli B. At which deli did he
spend more money on cheese?

(1) The ratio of American to Provolone to Swiss purchased at Deli B was


2:7:2
(2) The ratio of America to Provolone to Swiss purchased at Deli A was 6:2:1

(ratios, hard)

If he spent more in B than in A, then the average price per kg of the cheese
bought at B was higher than the average price per kg. of the cheese bought at
A.

(1) Since he bought equal amounts of American and Swiss at B, the average
amount he spent on those two types of cheese is (16 + 20)/2 = 18. We
can conclude that the average price for the 6kg of cheese bought at B
was $18, but no information is given about what he bought at A.
NOT SUFF

(2) The average price per kg of the the cheese he bought at A was (6(15) +
2(17) + 19)/9 = (6(16 -1) + 2(16 +1) + 16 + 3)/9 < 16. Since no
cheese at B costs less than $16 per kilo, the average price per kg of the
cheese bought at B was higher than the averge price per kg. of the
cheese bought at A. SUFF

539
k and m are positive integers such that if k is divided by m , the remainder is an
odd integer. Is k an even integer ?

(1) When k is divided by m, the quotient is even.


(2) m is odd.

(factors and multiples, hard)

We can say that k= qm + r where r is an odd positive number less than m,


and q is the quotient.

(1) As q is even, qm is even and k= qm + r is odd. SUFF


(2) qm will be even if q is even and odd if q is odd. As we don’t know
whether qm is odd, we don’t know whether k is even. NOT SUFF

540
Kate and David each have $10. Togather they flip a coin 5 times. Every time
the coin lands on heads, Kate gives David $1. Every time the coin lands on tails,
David gives Kate $1. After the coin is flipped 5 times, what is the probability
that Kate has more than $10 but less than $15?

5/16 15/32 1/2 21/32 11/16

(probability, hard)

Note that the number of heads t can be from 0 to 5, and the amount of money
Kate will have after the five tosses is 10 + t – (5 – t) = 5 + 2t. For Kate to have
between 10 and 15, 10 < 5 + 2t < 15, i.e. t is either 3 or 4. Note that Pr (t = 1
or 2) = Pr (t=3 or t=4), since the coin is equally likely to land on heads or tails.
Similarly Pr (t=0) = Pr (t=5) = (1/2)5 = 1/32, Pr (t=3 or th=4) =1/2(1 –
2/32)= 15/32.

5/16 15/32 1/2 21/32 11/16

541
Lana bought two lots last year and sold both of them yesterday. If the selling
price of lot B was twice that of lot A, did Lana make a profit on the sale of
these two lots?

(1) Lana sold lot A for 20% more than its purchase price.
(2) Lana sold lot B for more than 90% of its purchase price.

(percents, hard)

Suppose the selling price of A was x. Then the selling price of B was 2x and the
question can be rephrased as follows: Is the sum of the purchase prices of A
and B less than 3x?

(1) The purchase price of A is 5x/6 NOT SUFF

(2) The purchase price of B is less than 20x/9. NOT SUFF

(T) The sum of the purchase prices is less than 5x/6 + 20x/9 = 15x/18 +
40x/18 =55x/18 > 3x. Thus Lana may or may not have made a profit.
NOT SUFF

542
Larry tried to type his new 7-digit phone number on a form, but what appeared
on the form was 39269, since the '4' key on his computer no longer works. His
secretary has decided to make a list of all of the numbers that could be Larry's
new number. How many numbers will there be on the list?

21 24 25 30 36

(combinatronics, hard)

We need to see how many ways we can choose two of the seven digits of the
telephone number. These chosen digits will be 7’s: C 27  7  6  21
2
21 24 25 30 36

543
Last month 15 homes were sold in Town X. The average (arithmetic mean) sale
price of the homes was $150,000, and the median sale price was $130,000.
Which of the following statements must be true?

I. At least one of the homes was sold for more than $165,000.
II. At least one of the homes was sold for more than $130,000 and less
than $150,000.
III. At least one of the homes was sold for less than $130,000.

I only II only III only I and II I and III

(statistics, hard)

In terms of thousands of dollars, if the median sale price was 130, there were 8
houses whose selling price was less than or equal to 130. The sum of the
selling prices of the 15 houses was 15 (150). Thus the sum of the selling prices
of the 7 most expensive houses was at least 15(150) – 8 (130) = 7 (150) + 8
(20). The average selling price of these seven houses was therefore at least
150 + 8(20)/7 > 170. Thus at least one of the houses sold for more than 170,
and thus more than 165. I must be true.

It could be that 8 houses sold for exactly 130, so neither II nor III need be
true.
I only II only III only I and II I and III

544
Last Thursday, John assembled chairs at a rate of 3 chairs per hour for part of
the day and Larry assembled no chairs. Last Friday, Larry assembled chairs at a
rate of 4 chairs per hour for part of the day and John assembled no chairs. If
John and Larry assembled chairs for a total of 7 hours during these two days,
how many chairs did John assemble last Thursday?

(1) During these two days, John and Larry assembled a total of 25 chairs.
(2) During these two days, Larry assembled more chairs than John did.

(algebra, hard)

Suppose John worked for j hours on Thursday. Then Larry worked for 7 – j
hours on Friday.

(1) 3j + 4(7 – j) = 25. We can find the value of j and therefore the value of
3j, the number of chairs that John assembled on Thursday. SUFF
(2) 3j < 4(7 – j) , so j < 4 and 3j < 12, where 3j is the number of chairs that
John assembled. NOT SUFF

545
Last Sunday a certain store sold copies of Newspaper A for $1.00 each and
copies of Newspaper B for $1.25 each, and the store sold no other newspapers
that day. If r percent of the store's revenues from newspaper sales was from
Newspaper A and if p percent of the newspapers that the store sold were
copies of newspaper A, which of the following expresses r in terms of p?

100p/(125-p) 150p/(250-p) 300p/(375-p)


400p/(500-p) 500p/(625-p)

(percents, hard)

If p percent of the copies sold were of A, 100 – p percent were of B. The ratio
of the prices of A and B are in a 4:5 ratio, so the ratio of the revenue generated
by A to that generated by B is 4p: 5(100 – p) = 4p: 500 – 5p and the ratio of
the revenue generated by A to the total revenue is 4p: 500 – p. Thus A’s
revenue is 400p/(500 – p) % of the total.

100p/(125-p) 150p/(250-p) 300p/(375-p)


400p/(500-p) 500p/(625-p)

546
Last year a certain bond with a face value of $5000 yielded 8 percent of its face
value in interest. If that interest was approximately 6.5 percent of the bond’s
selling price, approximately what was the bond’s selling price?

$4063 $5325 $5351 $6000 $6154

(percents, medium)

The interest, 0.08 x 5000 = 400, is 6.5% of the selling price P.


Thus 400=0.065P, so P= 400/0.065 = 400000/65= 80000/13, which is slightly
more than 6000.

$4063 $5325 $5351 $6000 $6154

547
Last year in a group of 30 businesses, 21 reported a net profit and 15 had
investments in foreign markets. How many of the businesses did not report a
net profit nor invest in foreign markets last year?

(1) Last year 12 of the 30 businesses reported a net profit and had
investments in foreign markets.
(2) Last year 24 of the 30 businesses reported a net profit or invested in
foreign markets, or both.

(sets, medium)

Suppose that P is the set of the 30 businesses that reported a net profit and F
is the set of the 30 businesses that had invested in foreign markets. We are
asked for n( P  F ) .

Note that 30 = n( P)  n( F )  n( P  F )  n( P  F )  n( P  F )  n( P  F )
(1) 30 = 21 + 15 – 12  n( P  F ) SUFF
(2) 30 = 24  n( P  F ) SUFF

548
Last year the price per share of Stock X increased by k percent and the
earnings per share of Stock X increased by m percent, where k is greater than
m. By what percent did the ratio of price per share to earnings per share
increase, in terms of K and m?

k/m k-m 100(k-m) / (100 + k)


100(k-m) / (100 + m) 100(k-m) / (100 + k+ m)

(percents, hard)

Suppose last year’s price was P and the earnings per share is E, so that last
year’s ratio was P/E = R. This year, the share price is P(1 + k/100) and the
earnings per share is E(1 +m/100), so this year’s ratio, in terms of R, last year’s
ratio, is R(1 + k/100)/(1 + m/100)= R((100+k)/(100 + m). If the ratio
increased by x%, this year’s ratio is (1 + x/100) R.

Thus 1  x  100  k  x  k  m  x  100(k  m)


100 100  m 100 100  m 100  m

k/m k-m 100(k-m) / (100 + k)


100(k-m) / (100 + m) 100(k-m) / (100 + k+ m)

549
Leo can buy a certain computer for p1 dollars in State A, where the sales tax is
t1 percent, or he can buy the same computer for p2 dollars in State B, where
the sales tax is t2 percent. Is the total cost of the computer greater in State A
than in State B?

(1) t1 > t2
(2) p1t1 > p2t2

(inequalities, medium)

We are asked whether p1 (1+t1/100) > p2 (1 +t2/100)

(1) No information is given about the prices. NOT SUFF


(2) More tax is paid in A than in B. If the before-tax price in A were as least
that of the computer in B, the total cost of the computer would be
greater in State A than in State B, but no information is given that allows
us to compare p1 and p2. NOT SUFF
(T) It may be that p1 ≥ p2, in which case the answer is yes, or that p1 < p2
(for example p1 = 500, p2 = 1000, t1 =5 t2=2 ) NOT SUFF

550
Linda put an amount of money into each of two new investments, A and B, that
pay simple annual interest. If the annual interest rate of investment B is 1 1/2
times that of investment A, what amount did Linda put into investment A?

(1) The interest for 1 year is $50 for investment A and $150 for investment
B.
(2) The amount that Linda put into investment B is twice the amount that
she put into investment A.

(interest, hard)

Suppose Linda put a dollars into A and b dollars into B and the interest rate on
A is r

(1) ar =50 b (3r/2) = 150


3br/2 = 3ar  b = 2a.
We can deduce that Linda invested twice as much in B as in A, but as no
information is given on r, we cannot determine the value of a. SUFF

(2) tells us something that we can deduce from (1), but no more. SUFF
(T) NOT SUFF

551
Linda, Robert and Pat packed a certain number of boxes with books. What is
the ratio of number of boxes of books that Robert packed to the number of
boxes of books than Pat packed ?

(1) Linda packed 30% of the total number of boxes of books.


(2) Robert packed 10 more boxes of books than Pat did.

(ratios, medium)

(1) No information is given about the number of boxes than either Robert or
Pat packed. NOT SUFF
(2) If Pat packed p boxes, Robert packed p + 10 boxes. The required ratio,
(p + 10)/p, depends on the value of p. NOT SUFF
(T) The total number of boxes packed by Pat and Robert, 2p + 10, is 70% of
the total. We cannot find the value of p. NOT SUFF

552
Lines k and m are parallel to each other. Is the slope of line k positive?

(1) Line k passes through the point (3, 2).


(2) Line m passes through the point (-3, 2).

(coordinate geometry, medium)

(1) A line passing through (3,2) can have any slope. NOT SUFF
(2) A line passing through (-3,2) can have any slope. NOT SUFF
(T) These two lines could have any slope. NOT SUFF

553
Lines n and p lie in the xy- plane. Is the slope of line n less than the slope of
line p?

(1) Lines n and p intersect at the point (5,1).


(2) The y-intercept of line n is greater than the y-intercept of line p.

(coordinate geometry, hard)

(1) Nothing can be deduced about the slopes of the lines other than the fact
that they are not equal. NOT SUFF
(2) No information is given about the slopes of the lines. NOT SUFF

(T) Suppose that bN and bP are the y-intercepts of lines n and p. bN > bP.
Each line passes through the point (5,1). Line n passes through (0, bN),
so the slope of line n is 1  b . Likewise, the slope of line p is 1  b .
N P
5 5
As bN > bP, the slope of line n less than the slope of line p. SUFF

554
List K consists of 12 consecutive integers. If -4 is the least of the elements of K,
what is the range of the positive elements of K?

5 6 7 11 12

(statistics, medium)

Note that a set of n consecutive integers has a range of n – 1 . Thus list K has a
range of 11, and so the greatest element of K is -4 + 11 =7. The smallest
positive element is 1, so the range of the positive elements of K is 7 – 1 =6.

5 6 7 11 12

555
Lists S and T consist of the same number of positive integers. Is the median of
the integers in S greater than the average (arithmetic mean) of the integers in
T?

(1) The integers in S are consecutive even integers, and the integers in T are
consecutive odd integers.
(2) The sum of the integers in S is greater than the sum of the integers in T.

(sequences, statistics, hard)

(1) S and T are both arithmetic sequences, so the median of each set is
equal to its mean. We have no means of comparing the means of the
two sets, however. NOT SUFF
(2) The mean of S is greater than the mean of T, but we have no
information on whether the median of S is equal to, greater than or less
than the mean of S. NOT SUFF
(T) SUFF

556
List S and list T each contain 5 positive integers, and for each list the average
(arithmetic mean) of the integers in the list is 40. If the integers 30, 40, and 50
are in both lists, is the standard deviation of the integers in list S greater
deviation of the integers in list T?

(1) The integer 25 is in list S.


(2) The integer 45 is in list T.

(statistics, medium)

Note that the average of 30, 40 and 50 is 40, so the average of the two
remaining elements in each list is 40. The set whose two remaining integers
have the higher range will have the higher standard deviation.

(T) The range of the two remaining elements in S is 55 – 25 = 30, whereas


that of the two remaining elements in T is 55 – 45 = 10. SUFF
Also, the two statements taken together give us the composition of each
set

557
Machine A, operating alone at its constant rate produces 500 feet of a particular
fiber in 2 hours, Machine B operating alone at its constant rate produces 500
feet of fiber in 3 hours. Machine C operationg alone at its constant rate
produces 500 feet of the same fiber in 5 hours. How many hours will it take
Machines A,,B, and C operating together at their respective constant rates to
produce 1000 feet of fiber?

60/31 15/7 25/11 10/2 20/3

(ratios, hard)

In 30 hours, they produce 500(15) + 500(10)+ 500(6) = 500(31) feet

1000 feet x (30 hours/500(31) feet) = 60/31 hours

Alternatively, in one hour they produce 500/2 + 500/3 + 500/5 > 500 feet, so
they will need less than 2 hours

60/31 15/7 25/11 10/2 20/3

558
Machines X and Y work at their respective constant rates. How many more
hours does it take machine Y, working alone, to fill a production order of a
certain size than it takes machine X, working alone?

(1) Machine X and Y, working together, fill a production order of this size in
two-thirds the time that machine X, working alone, does.
(2) Machine Y, working alone, fills a production order of this size in twice the
time that machine X, working alone, does.

(combined work, medium)

(T) Each statement allows us to determine the ratio of the rates of the
machines, but no information is given about the size of the order or the
number of hours the machines would take, either individually or in
tandem. NOT SUFF

559
Machine A produces pencils at a constant rate of 9,000 pencils per hour, and
Machine B produces pencils at a constant rate of 7,000 pencils per hour. If the
two machines together must produce 100,000 pencils and if each machine can
operate for at most 8 hours, what is the least amount of time, in hours, that
Machine B must operate?

4 4 2/3 5 1/3 6 6¼

(inequalities, medium)

The key to this question is to see the the maximum number of pencils that can
be produced by A: since A can work for at most 8 hours and produces 9,000
pencils per hour, it can produce at most 72,000 pencils. This means that B must
produce at least 28,000 pencils, a number that can produced in 4 hours.

4 4 2/3 5 1/3 6 6¼

560
Malik's recipe for 4 servings of a certain dish requires 3/2 cups of pasta.
According to this recipe, what is the number of cups of pasta that Malik will use
the next time he prepares this dish?

(1) The next time he prepares this dish, Malik will make half as many
servings as he did the last time he prepared the dish.
(2) Malik used 6 cups of pasta the last time he prepared this dish.

(ratios, medium)

(T) He will need half as many cups as he used last time. SUFF

561
Martha bought an armchair and coffee table at an auction and sold both items
at her store. Her gross profit from the purchase and sale of the armchair was
what percent greater than her gross profit from the purchase and sale of the
coffee table?

(1) Martha paid 10 percent more for the armchair than for the coffee table.
(2) Martha sold the armchair for 20 percent more than she sold the coffee
table.

Information about both the purchase prices and the sale prices in needed, so
neither (1) nor (2) is sufficient by itself.

(T) Suppose Martha paid c for the coffee table. (1) tells us that she paid 1.1c
for the armchair. Suppose Martha sold the coffee table for s. (2) tells us
that she sold the armchair for 1.2s. Her gross profit from the purchase
and sale of the coffee table is s – c , and that from the purchase and sale
of the armchair is 1.2s – 1.1c. Thus the difference between the gross
profits is 0.2s – 0.1c. The ratio between this difference and the gross
profit from the purchase and sale of the coffee table is
0.2s  0.1c 0.1s
  0.1. This ratio clearly depends on the values of s and c,
sc sc
which are not given. NOT SUFFICIENT

562
Martha bought several pencils. If each pencil was either a 23-cent pencil or a
21-cent pencil, how many 23-cent pencils did Martha buy?

(1) Martha bought a total of 6 pencils


(2) The total value of the pencils Martha bought was 130 cents

(1) Obviously not sufficient. NOT SUFF


(2) Note that n, the number of pencils Martha bought must be between
130/23 < n < 130 / 21 inclusive. As n must be an integer , n must be 6.
If Martha bought x 23-cent pencils, she bought 6 – x 21-cent pencils, for
a total value of 23x + 21 ( 6 – x) = 126 + 2x. Since 126 + 2x = 130, the
value of x can be found. SUFF

563
Martin has worked for the last 30 years. If his average (arithmetic mean) total
annual earnings for the first 5 years is $15,000, what is the average total
annual earnings for the last 5 years?

(1) Martin’s average total annual earnings for the first 25 years is $27,000.
(2) Martin’s average total annual earnings for the last 25 years is $34,000.

(sequences, medium)

Suppose an is Martin’s salary in thousands of dollars in the nth most recent year
of his career. We are told that a26 +a27 +…+ a30 = 27(30) . We are asked for (a1
+a2 +a3 +a4 +a5)/5 .

(1) a6 + a7 + … + a30 = 27(25) NOT SUFF


(2) a1 + a2 + … + a25 = 25(34) NOT SUFF
(T) a1 +a2 +a3 +a4 +a5 = 25(34) + 27(30) – 27(25). SUFF

564
Mary has 11 more nickels ($0.05) than quarters ($0.25). How many nickels
does she have if the total value of her coins is $2.65?

17 18 19 20 21

Suppose she has q quarters. The she has q + 11 nickels, and the total value of
her coins , in cents, is 5(q+11) + 25q =30q + 55 =265. Thus q=7 and the
number of nickels she has is q + 11 = 18.

17 18 19 20 21

565
Mary persuaded n friends to donate $500 each to her election campaign, and
then each of these n friends persuaded n more people to donate $500 each to
Mary’s campaign. If no one donated more than once and if there were no other
donations, what was the value of n?

(1) The first n people donated 1/16 of the total amount donated.
(2) The total amount donated was $120,000.

(1) The first n people donated $500 each, and the second n×n =n2 people
donated $500 each. Thus the ratio of the amount donated by the first set
of people to that donated by the second set is 1/n, so the first n people
donated 1/(n+1) of the total amount. Therefore, n+1=16 and n=15
SUFF

(2) 500n +500n2 = 120,000→ 500n2 + 500n – 120,000, a quadratic equation


with a=500 and c= -120,000. As ac < 0, this equation has two distinct
solutions, one positive and the other negative. As n > 0, there is only
one possible value of n. SUFF

566
Of the cans of peaches inspected yesterday at a certain plant, 1.5 percent failed
to pass inspection. Of the cans that failed inspection, 5/6 were incorrectly
labeled and the rest were dented. If all of the cans that were incorrectly labeled
or dented failed inspection, how many of the cans of peaches inspected
yesterday at the plant were dented?

(1) 450 of the cans of peaches inspected yesterday at the plant failed to
pass inspection.
(2) 29,550 of the cans of peaches inspected yesterday at the plant passed
inspection.

(percents, medium)

Suppose x cans were inspected yesterday. Then 0.015x failed inspection, 1/6 of
which were dented. Thus any information that allows us to find the value of x
will be sufficient

(1) 0.015x = 450 SUFF


(2) x – 0.015x = 29,550 SUFF

567
Of the fruit that arrives at a cannery, 20 percent by weight is rejected before
processing. Of the fruit that is processed, 15 percent by weight is rejected
before canning. Of the fruit that arrives at the cannery, what percent by weight
is canned?

32% 35% 65% 68% 70%

(percents, medium)

Of the fruit that arrives at the cannery, 80% is processed. Of the 80% that is
processed, 85% is canned. Thus 0.85(80) = 68 % of the fruit that arrives at
the cannery is canned.

32% 35% 65% 68% 70%

568
–n
Of the following values of n, the value of (1/5) will be greatest for n =

3 2 0 –2 –3

(exponents, medium)

–n
(1/5) = (5-1)-n = 5n, the value of which increases as n increases.

3 2 0 –2 –3

569
Of the 800 employees in Company X, 70% have been with the company for at
least 10 years. If y of these "long-term" members were to retire, and no other
employee changes were to occur, what value of y would reduce the percent of
"long-term" employees in the company to 60%?

200 160 112 80 56

560 of the 800 employees have been with the company for at least 10 years. If
y of these employees were to retire, 560 – y of the 800 – y employees would
have at least 10 years of senority. If 560 – y is 60% of 800 – y,

560  y 80
 0.6  480  0.6 y  560  y  0.4 y  80  y   200
800  y 0.4

200 160 112 80 56

570
Of the 4800 voters who voted for or against Resolution K, 1800 were
Democrats and 3000 were Republicans. What was the total number of female
voters who voted for Resolution K?

(1) 3/4 of the Democrats and 2/3 Republicans voted for Resolution K.
(2) 1/3 of the Democrats who voted for Resolution K and 1/2 of the
Republicans who voted for resolution K were females.

(sets, hard)

(1) Tells us the number of Republicans and the number of Democrats that
voted for K, but no mention is made of female voters. NOT SUFF
(2) Tells us what fraction of the the Democrats and Republicans who voted
for K were women, but does not allow us to determine how many
Democrats and Republicans voted for K. NOT SUFF
(T) SUFF

571
Of the 100 companies responding to a certain survey, what perfect indicated
that they had a business recovery plan?

(1) 200 companies did not indicate that they had a business recovery plan.
(2) The number of companies that indicated that they had a business
recovery plan was four times the number that did not indicate that they
had a business recovery plan.

(percents, sets, medium)

To answer the question, it is sufficient to find x, the number of companies that


indicated that they had a plan.

(1) 1000 - x = 200 SUFF


(2) x = 4(1000 – x ) SUFF

572
Of the paintings that a certain artist created last year, the number of paintings
that were unframed was 5 times the number of paintings that were framed. If
48 percent of the unframed paintings and 30 percent of the framed paintings
were sold, what percent of the paintings that the artist created last year were
sold?

33% 36% 39% 42% 45%


(percents, medium)

Suppose x paintings were framed and thus 5x were framed. 0.3x + 0.48(5x)
2.7x paintings were sold last year of the 6x that were created, i.e 45%.

Alternatively, since 5 times as many paintings were unframed as were framed,


the required percentage must be 5 times as far from 30% as from 48%. Thus
the required percentage must be 1/6 of the 18 percentage points belolw 48
percent!

33% 36% 39% 42% 45%

573
Of the 75 houses in a certain community, 48 have a patio. How many of the
houses in the community have a swimming pool?

(1) 38 of the houses in the community have a patio but do not have a
swimming pool.
(2) The number of houses in the community that have a patio and a
swimming pool is equal to the number of houses in the community
that have neither a patio nor a swimming pool.

Note that 75 = n(P) + n(S) + n(neither) – n (both) = n(only P) + n(only S) +


n(both) + n (neither).

(1) n(only P) = 38. NOT SUFF


(2) n(both) = n(neither). Thus n(neither) – n (both)=0 and 75= n(P) + n(S).
As n(P)= 48, n(S)= 75 – 48. SUFF

574
Of the 60 animals on a certain farm, 2/3 are either pigs or cows. How many of
the animals are cows?

(1) The farm has more than twice as many cows as it has pigs.
(2) The farm has more than 12 pigs.

(inequalities, hard)

In all there are 40 animals that are either pigs or cows. If there are c cows,
there are 40 – c pigs.

(1) c > 2 (40 – c )  3c > 80  c > 80/3 , so c is at least 27. NOT SUFF
(2) 40 – c > 12  c < 28. NOT SUFF
(T) c= 27 SUFF

575
Of the 1,400 college teachers surveyed, 42 percent said that they considered
engaging in research an essential goal. How many of the college teachers
surveyed were women?

(1) In the survey, 36 percent of the men and 50 percent of the women said
that they considered engaging in research an essential goal.

(2) In the survey, 288 men said that they considered engaging in research
an essential goal.

Suppose there are w women and 1400 – w men. We are asked to find the
value of w.

(1) 0.42(1400) = 0.36(1400-w) + 0.5w As we can solve for w, SUFF


(2) 1400 – w >= 288 NOT SUFF

576
Of the students who eat in a certain cafeteria, each student either likes or
dislikes lima beans and each student either likes or dislikes brussels sprouts. Of
these students 2/3 dislike lima beans and of those who dislike lima beans, 3/5
also dislike brussels sprouts. How many of the students like brussels sprouts
but dislike lima beans?

(1) 120 students eat in the cafeteria.


(2) 40 students like lima beans.

The number of students who like brussels sprouts but dislike lima beans is
4n/15, where n is the total number of students. Any information that allows us
to deduce the value of n will allow us to answer the question posed.

(1) n = 120. SUFF


(2) n/3 = 40 SUFF

577
Of the 12 temporary employees in a certain company, 4 will be hired as
permanent employees. If 5 of the 12 temporary employees are women, how
many of the possible groups of 4 temporary employees consist of 3 women and
1 man?

22 35 56 70 105

(combinatronics, medium)

22 35 56 70 105

578
Of the 25 cars sold at a certain dealership yesterday, some had automatic
transmission and some had anti-lock brakes. How many of the cars had a
automatic transmission but not antilock brakes?

(1) All of the cars that had anti-lock brakes also had an automatic
transmission.
(2) 2 of the cars had neither automatic transmission nor anti-lock brakes.

(sets, medium)

(T) NOT SUFFICIENT

579
Of the 13 employees in a certain department, 1 has an annual salary of
$38,000, 2 have an annual salary of $45,000, 2 have an annual salary of
$42,500, 3 have an annual salary of $40,000, and 5 have an annual salary of
$48,500. What is the median annual salary for the 13 employees?

$38,000 $40,000 $42,500 $45,000 $48,500

(statistics, medium)

Find the 7th highest salary: as 5 have a salary of $48,500 and 2 have a salary
of $45,000, the 7th highest salary, and thus the median, is $45,000.

$38,000 $40,000 $42,500 $45,000 $48,500

580
m 18
1 1 1
If      , then m=
5  4 2  1035

17 18 34 35 36

Notice that the power of 5 on the left side is –m , whereas the power of 5 on
the right side is – 35, since 10= 2 x 5. The power of 2 on the left side is -36, as
is the power of 2 on the right side. Thus –m= - 35 , so m=35.

17 18 34 35 36

581
Of all the students in a certain dormitory, 1/2 are first-year students and the
rest are second-year students. If 4/5 of the first-year students have not
declared a major and if the fraction of second-year students who have declared
a major is 3 times the fraction of first-year students who have declared a major,
what fraction of all the students in the dormitory are second-year students who
have not declared a major?

1/15 1/5 4/15 1/3 2/5

If there are n students in the dormitory, n/2 are first-year students and n/2 are
two-year students. 1/5 of the first-year students have declared a major, so 3/5
of the second-year students have declared a major. Thus 2/5 of the n/2 (i.e.
n/5) second-year students have not declared a major, 1/5 of the total number
of students.

1/15 1/5 4/15 1/3 2/5

582
Of the goose eggs laid at a certain pond, 2/3 hatched, and 3/4 of the geese
that hatched from those eggs survived the first month. Of the geese that
survived the first month, 3/5 did not survive the first year. If 120 geese
survived the first year and if no more than one goose hatched from each egg,
how many goose eggs were laid at the pond?

280 400 540 600 840

(sets, medium)

If x were laid, 2x/3 hatched, of which ¾ survived. Thus x/2 survied the first
month, of which 2/5 survived the first year, i.e. x/5. Thus x=600.

280 400 540 600 840

583
Of the students in a certain school, 15 percent are enrolled in an art class and
10 percent are enrolled in a music class. What percent of the students in the
school are enrolled in neither an art class nor a music class?

(1) 2/3 of the students who are enrolled in an art class are also enrolled in a
music class.
(2) There are more than 100 students in the school.

(sets, medium)

To answer the question, it is enough to determine what percent of the students


in the school are enrolled in either an art class or a music class. We know that
10% are enrolled in music and 15% are enrolled in art, and if x% are enrolled
in both, 25 –x % are enrolled in either, and thus 75 + x % are enrolled in
neither.

(1) 2/3 of the 15% enrolled in music comprise those students who are
enrolled in both. SUFF
(2) NOT SUFF

584
Of the 200 members of a certain association, each member who speaks
German also speaks English, and 70 of the members speak only Spanish. If no
member speaks all three languages, how many of the members speak two of
the three languages?

(1) 60 of the members speak only English


(2) 20 of the members do not speak any of the three languages

(1) If 60 speak only English and 70 speak only Spanish, 70 members speak
either two of the languages or none at all, as none speak all three and
none speak only German. As we don’t know how many speak none, we
cannot determine how many speak two of the languages. NOT SUFF
(2) We know that 110 members speak only English or two of the langauges,
but as we don’t know how many speak only English, we cannot
determine how many speak two of the languages. NOT SUFF
(T) SUFF

585
On a certain day, Tim invested $1000 at 10 percent annual interest
compounded annually, and Lana invested $2000 at 5 percent annual interest
compounded annually. The total amount of interest earned by Tim’s investment
in the first 2 years was how much greater than the total amount of interest
earned by Lana’s investment in the first 2 years?

$5 $25 $50 $100 $105

Tim’s principal of $1000 generates $100 of interest in each of the two years,
and the $100 interest generated after the first year generates $10 of interest.
Thus Tim’s investment generated a total of $210 of interest. This can also be
found by subtracting $1000 from his balance after two years, 1000(1.1)2.

Similarly, Lana’s principal also generated $100 of interest in each of the two
years, but the $100 interest generated after the first year generates only $5 of
interest. Thus Lana’s investment generated a total of $205 of interest, for a
difference of $5.

Also, Tim’s interest is 1000(1.12 – 1) = 1000 (1.1+1)(1.1 - 1)=


1000(2.1)(0.1)=210

$5 $25 $50 $100 $105

586
On a certain nonstop trip, Marta averaged x miles per hour for 2 hours and y
miles per hour for the remaining 3 hours. What was her average speed, in miles
per hour, for the entire trip?

(1) 2x + 3y = 280
(2) y = x + 10

(movement, medium)

Her average speed for the entire trip is the distance she covered, 2x + 3y,
divided by the number of hours she travelled, 5.

(1) SUFF
(2) NOT SUFF

587
On a certain sightseeing tour, the ratio of the number of women to the number
of children was 5:2. What was the number of men on the sightseeing tour?

(1) On the sightseeing tour, the ratio of the number of children to the
number of men was 5:11.
(2) The number of women on the sightseeing tour was less than 30.

(ratios, hard)

(1) w : c = 5 : 2 , c : m = 5 : 11. Multiplying the two ratios, we see that


w : m = 25 : 22, so w : c : m = 25 : 10 : 22. The number of men is a
multiple of 22, but we have no means of determining the exact value of
m, the number of men. NOT SUFF

(2) Obviously not sufficient NOT SUFF


(T) 25 is the only positive multiple of 25 that is less than 25, so w= 25 and
m = 22. SUFF

588
On a certain transatlantic crossing, 20 percent of a ship’s passengers held
round-trip tickets and also took their cars abroad the ship. If 60 percent of the
passengers with round-trip tickets did not take their cars abroad the ship, what
percent of the ship’s passengers held round-trip tickets?

33 1/3% 40% 50% 60% 66 2/3%

(sets, medium)

Suppose the ship had n passengers, so 0.2n held round-trip tickets and also
took their cars abroad the ship. 40 percent of the passengers with round-trip
tickets took their cars abroad the ship, thus if x passengers held round trip
tickets, 0.4x = 0.2n and x =0.5n. Therefore 50% of the passengers held round-
trip tickets.

33 1/3% 40% 50% 60% 66 2/3%

589
On a map Town G is 10 centimeters due east of Town H and 8 centimeters due
south of Town J. Which of the following is closest to the straight-line distance,
in centimeters, between Town H and Town J on the map?

6 13 18 20 24

(geometry, medium)

HJ =

590
On an aerial photograph, the surface of a pond appears as circular region of
radius 7/16 inch. If a distance of 1 inch on the photograph corresponds to an
actual distance of 2 miles, which of the following is the closest estimate of the
actual surface area of the pond, in square miles?

1.3 2.4 3.0 3.8 5.0

(ratios, geometry, medium)

The radius of the pond is 7/16 × 2 =7/8 miles, so the area of the pond is
π(7/8)2 square miles (approximately 3(49/64) or 49/21)

1.3 2.4 3.0 3.8 5.0

591
One kilogram of a certain coffee blend consists of x kilogram of type I coffee
and y kilogram of type II coffee. The cost of the blend is C dollars per kilogram,
where C = 6.5x + 8.5y. Is x < 0.8 ?

(1) y > 0.15


(2) C ≥ 7.30

Note that x + y = 1, so C =6.5x + 8.5(1 – x) = 8.5 – 2x

(1) 1 – x > 0.15 , so x < 0.85 INSUFF


(2) 8.5 – 2x ≥ 7.30, so x ≤ 0.6 SUFF

592
On her way home form work, Janet drives through several toll booths. Is there
a pair of these toll booths that are less than 10 miles apart?

(1) The first toll booth and the last toll booth are 25 miles apart.
(2) Janet drives through 4 toll booths on her way home from work.

(1) There may be only two booths, in which case they are more than 10
miles apart. Conversely , if there are 4 booths, the largest possible
minimum distance results when the 4 booths are evenly spaced. The
length of each of the 3 intervals would be 25/3 < 10. INSUFF

(2) No information is given about the total distance between the first and
last booths. INSUFF
(T) SUFF

593
On July 1 of last year, the total number of employees at Company E was
decreased by 10%. Without any change in the salaries of the remaining
employees, the average (arithmetic mean) employee salary was 10% more
after the decrease in number of employee than before the decrease. The total
of the combined salaries of all of the employees at company E after July 1 of
last year was what percent of that before of July 1 last year?

90% 99% 100% 101% 110%

Suppose that, before July 1 of last year, Company E had n employees and that
their average salary was S. After July 1 of last year, Company E had 10% fewer
employees, 0.9n, and the remaining employees’ average salary was 10%
greater, 1.01s. Therefore, the combined salaries of the employees at Company
W after July 1 was 0.99ns, 99% of ns, the combined salaries of the
employees at Company E before July 1.

90% 99% 100% 101% 110%

594
On his trip from A to B, Julio covered the first x miles at 50 miles per hour and
the remaining miles at 60 miles per hour. How long did it take Julio to drive the
first x miles?

(1) Julio drove a total distance of 530 miles in 10 hours.


(2) It took Julio 4 more hours to drive the first 'x' miles than the remainder
of the journey.

(1) The time for the first x miles plus the time for the remaining 530 – x
miles is 10 hours.
Thus x  530  x  10  5 x  6(530  x)  3000. Since the value of x can be
50 60
found, so can the time Julio took to drive the first x miles, x . SUFF
50
x dx
(2) Suppose that the total distance is d miles.  4 . As we do not
50 60
know the value of d, x and thus the time Julio time to drive the first x
miles cannot be determined. INSUFF

595
On Jane's credit card account, the average daily balance for a 30-day billing
cycle is average (arithmetic mean) of the daily balances at the end of the 30
days. At the beginning of a certain 30-day billing cycle, Jane's credit card
account had a balance of $600. Jane made a payment of $300 on the account
during the billing cycle. If no other amounts were added to or subtracted from
the account during the billing cycle, what was average daily balance on Jane's
account for the billing cycle?

(1) Jane's payment was credited on the 21st day of the billing cycle.
(2) The average daily balance through the 25th day of the billing cycle was
$540.

(1) We know there were 20 days with a balance of $600, and 10 with a
balance of $300. Since the sum of these daily balances can be found, so
can the average daily balance over the 30-day cycle by dividing this sum
by 30. SUFF

(2) The sum of the daily balances of the first 25 days can be determined by
multiplying this average by 25. As the 25-day average is less than $600,
the payment was credited sometime before the 25th day, so the 5
remaining days had a balance of $300. As the sum of the 30 daily
balances can be found, the required average can be determined.
SUFF

596
On Saturday morning, Malachi will begin a camping vacation and he will return
home at the end of the first day on which it rains. If on the first three days of
the vacation the probability of rain on each day is 0.2, what is the probability
that Malachi will return home at the end of the day on the following Monday?

0.008 0.128 0.488 0.512 0.640

(probability, hard)

For Malachi to return home at the end of the day on the following Monday, it
must be dry on Saturday and Sunday and rain on Monday. The probability that
it rains on any given day is 0.2, so the the probability that it is dry (i.e. does not
rain) on any given day is 0.8. Thus the required probability is 0.8 × 0.8 × 0.2=
0.128

0.008 0.128 0.488 0.512 0.640

597
On the number line, if the numberk is to the left of the numbert, is the product
kt to the right of t ?

(1) t<0
(2) k<1

(inequalities, hard)

We are told that k < t and are asked whether kt > t.

(1) t < 0, and therefore so is k. Therefore kt is the product of two negative


numbers and is thus positive. Therefore kt > t. SUFF
(2) We have seen in (1) that if t < 0, kt > t.
However, if t=0, kt=t. NOT SUFF

598
On the number line shown, is zero halfway between r and s?

(1) s is to the right of zero.


(2) The distance between t and r is equal to the distance between t and –s.

In other words, is r + s = 0 ?

(1) Nothing is said about r. NOT SUFF


(2) If s > 0, - s < s. Therefore t – r = t – (-s), so r + s = 0.
However, if s < 0, r < 0 and s + r < 0. NOT SUFF
(T) We know that s > 0, so r + s = 0 SUFF

599
On the number line, the distance between x and y is greater than the distance
between x and z. Does z lie between x and y on the number line?

(1) xyz<0
(2) xy<0

z could be between x and y or z could x could be between z and y.

Clearly, neither is sufficient. Together, we see that y is negative and the


product of x and z is positive. It could be that x, y and z are all negative.
(x,y,z) could be (-3,-1,-2) or (-3,-1,-4). In other words, z may or may not be
between x and y.
(T) NOT SUFF

600
On the number line, what is the distance between the point 2x and the point
3x?

(1) On the number line, the distance between the point -x and the point x is
16.
(2) On the number line, the distance between the point x and point 3x is 16.

(algebra, medium)

We need to know the value of |2x – 3x| = |-x| = |x|

(1) |x –(-x)| =|2x| = 2|x| = 16 SUFF


(2) | x – 3x| = |-2x| = 2|x| = 16 SUFF

601
Over a certain time period, did the number of shares of stock in Ruth's portfolio
increase?

(1) Over the time period, the ratio of the number of shares to the total
number of shares of stocks and bonds in Ruth's portfolio increased.
(2) Over the time period, the total number of shares of stocks and bonds in
Ruth's portfolio increased.

Neither is sufficient by itself: a higher ratio may or may not mean a larger
number of shares of stock, and a larger total number of shares may or may not
mean a larger number of shares of stock. However, if shares of stock account
for a larger percentage of a larger total number of shares of stock and bonds,
the number of shares of stock will certainly be higher.
(T) SUFF

602
Pat bought 5 pounds of apples. How many pounds of pears could Pat have
bought for the same amount of money?

(1) One pound of pears costs $0.50 more than one pounds of apples.
(2) One pound of pears costs 1.5 times as much as one pound of apples.

If a and p are prices of apples and pears per pound. Pat spent 5a dollars, which
could have bought 5a/p pounds of pears.
What we need to know is the ratio of prices, not the difference in prices.
(1) NOT SUFF
(2) SUFF.

603
Pat, Kate and Mark charged 162 hours to a certain project. If Pat charged twice
as much time as Kate and 1/3 as much time as Mark, how many more hours
did Mark charge to the project than Kate?

18 36 72 90 108

If Kate charged k hours, Pat charged 2k hours and Mark charged three times as
many hours as Pat (i.e. 6k hours). In all, 162 hours were charged, so 9k= 162
i.e k=18. The difference between the number of hours that Mark charged (6k)
and the number of hours that Kate charged (k) is 5k=90.

18 36 72 90 108

604
Particia has several coins, each of which is worth either 5 cents or 25 cents.
How many coins does she have?

(1) The total value of Patricia’s coins is 205 cents


(2) Patricia has more than 2 coins worth 25 cents and less than 7 coins
worth 5 cents.

(inequalities, medium)

Suppose she has f five cent and t 25 cent coins. We are asked for the value of f
+ t.

(1) 5f + 25t = 205, so f + 5t = 41. The values of f and t could be (1,8),


(6,7), (11,6), (16,5), (21,4)... each ordered pair has a different sum.
NOT SUFF
(2) No information is given about an upper bound to the number of 25 cent
coins. NOT SUFF
(T) (f,t) could be (1,8) or (6,7), so there could be either 9 or 13 coins.
NOT SUFF

605
Paul is planning to save $1 this week and then save $1 more each succeeding
week than he saved the week before. His goal is to save $210. Using his
knowledge than the sum S of the first n consecutive positive integers is
S = n(n+1)/2 , where n > 1, he can determine that it will take him how many
weeks to reach his goal?

10 11 15 20 21

(algebra, medium)

We need to find the value of positive integer n such that n(n+1)/2 = 210
n(n+1)= 420. Note that 420 is a multiple of 20, so we should look at values of
n that are multiples of 20 at values of n that are 1 less than a multiple of 20.
n=20 fits the bill, as 20(21) = 420

Alternatively, n2 + n – 420 =(n +21)(n – 20) = 0

606
Professor Vásquez gave a quiz to two classes. Was the range of scores for the
first class equal to the range of scores for the second class?

(1) In each class, the number of students taking the quiz was 26, and the
lowest score in each class was 70.
(2) In each class, the average (arithmetic mean) score on the quiz was 85.

(statistics, medium)

(T) Either class could have 13 scores of 70 and 13 scores of 100, but this
need not be the case. Either class could have 1 score of 70, 11 scores of
71, 1 score of 72 and 13 scores of 99. Thus the ranges may or may not
be the same. NOT SUFF

607
Pumps A, B, and C operate at their respective constant rates. Pumps A and B,
operating simultaneously, can fill a certain tank in 6/5 hours; pumps A and C,
operating simultaneously, can fill the tank in 3/2 hours; and pumps B and C,
operating simultaneously, can fill the tank in 2 hours. How many hours does it
take pumps A, B, and C, operating simultaneously to fill the tank?

1/3 1/2 2/3 5/6 1

(combined work, hard)

Think of 3 groups of pumps working independently at their respective rates:


group 1, consisting of 1 pump A and 1 pump B, can fill the tank in 6/5 hours;
group 2, consisting of 1 pump A and 1 pump C, can fill the tank in 3/2 hours;
group 3, consisting of 1 pump B and 1 pump C, can fill the tank in 2 hours. The
fraction of the tank that the three groups can fill in one hour is 5/6 + 2/3 +
1/2= 2, so the 2 A’s 2 B’s and 2 C’s can fill one tank in 1/2 hour. Thus 1 A 1 B
and 1 C can fill the tank in 1 hour.

1/3 1/2 2/3 5/6 1

608
pa qb rc sd= x, where x is a perfect square. If p, q, r, and s are prime integers,
are they distinct?

(1) 18 is a factor of ab and cd


(2) 4 is not a factor of ab and cd

(factors and multiples, hard)

If these prime integers are distinct, a b c and d must be positive even integers.

(1) x could be (pqrs)18 and p,q r s may or may not be distinct NOT SUFF
(2) As ab is not divisible by 4, a and b are not both even. Thus p,q,r,s are
not distinct SUFF

609
Rasheed bought two kinds of candy bars, chocolate and toffee that came in
packages of 2 bars each. He handed out 2/3 of the chocolate bars and 3/5 of
the toffee bars. How many pacakges of chocolate bars did Rasheed buy?

(1) Rasheed bought 1 fewer package of chocolate bars than toffee bars.
(2) Rasheed handed out the same number of each kind of candy bar.

(algebra, medium)

If Rasheed bought c packages of chocolate bars and t packages of toffee bars,


he handed out 2c/3 chocolate bars and 3t/5 toffee bars.

(1) c = t – 1. As we do not know the value of t, we cannot determine the


value of c. NOT SUFF
(2) 2c/3 = 3t/5 -- c = 9t/10. Again, As we do not know the value of t, we
cannot determine the value of c. NOT SUFF
(T) As we have two linear equations with two unknowns, we can solve for c.
SUFF

610
Rates for having a manuscript typed at a certain typing service are $5 per page
for the first time a page is typed and $3 per page each time a page is revised.
If a certain manuscript has 100 pages, of which 40 were revised only once, 10
were revised twice, and the rest required no revisions, what was the total cost
of having the manuscript typed?

A. $430
B. $620
C. $650
D. $680
E. $770

611
R is a convex polygon. Does R have at least 8 sides?

(1) Exactly 3 of the interior angles of R are greater than 80 degrees.


(2) None of the interior angles of R are less than 60 degrees.

(geometry, hard)

If R has n sides, the sum of its interior angles in degrees is (n – 2)180.

(1) tells us that three of the angles are between 80º and 180º. Therefore
the remaining n – 3 angles are at most 80º. Thus the sum of the
measures of the n angles is less than 180(3)+ (n – 3)80.

Thus (n – 2)180 < 180(3) + (n – 3)80


100n < 300 + 360
n < 660/100
Thus R has at most 6 sides. SUFF

(2) If all of the interior angles are at least 60 degrees, the sum of the
measures of the n interior angles is at least 60n. Thus 60n ≤ 180(n – 2).
Therefore 120n ≥ 360 and n ≥ 3. Knowing that R has at least three sides
does not allow us to determine whether R has at least 8 sides. INSUFF

612
Robin split a total of $24,000 between 2 investments, X and Y. If investment Y
earns 7% simple annual interest, how much of that total did Robin put into
investment Y?

(1) Each investment earns the same dollar amount of the interest annually.
(2) Investment X earns 5 percent simple annual interest.

(interest, medium)

Suppose y dollars were invested in Y. It earns 0.07y per year, whereas X, which
earns, say, r% simple annual interest, earns (24000 – y)r/100 per year.

(1) 0.07y =(24000 – y)r/100 NOT SUFF


(2) r=5 NOT SUFF
(T) SUFF

613
Robots X,Y and Z each assemble components at their respective constant rates.
If rx is the ratio of robot X’s constant rate to robot Z’s constant rate and rY is
the ratio of robot Y’s constant rate to robot Z’s constant rate, is robot Z’s
constant rate the greatest of the three?

(1) rx < ry
(2) ry < 1

(ratios, medium)

We need to know whether rx < 1 and ry < 1.

(T) SUFF

614
Running at their respective constant rates, machine X takes 2 days longer to
produce w widgets than machine Y. At these rates, if the two machines
together produce (5/4) w widgets in 3 days, how many days would it take
machine X alone to produce 2w widgets?

4 6 8 10 12

(combined work, hard)

Suppose it takes machine X x days to produce w widgets. It then takes Y x – 2


days to do so and we can write 3/x + 3/(x – 2) = 5/4,
so 3(x – 2 ) + 3x=(5/4)x(x – 2) —>: 4(6x – 6)=5x2 – 10x
Thus 5x2 -34x + 24 = 0 —> (5x – 30)(5x – 4)=0 i.e. x=6 (x cannot be 4/5 as
x -2 would be negative)
As it takes 6 hours for X to produce w widgets, it takes 12 hours for it to
produce twice that number, 2w

4 6 8 10 12

615
Sequence s1, s2, s3 ,…, sn is such that sk=1/k – 1/(k+1) for all k such that 1 ≤ k
≤ n. Is the sum of the terms of the sequence greater than 9/10?

(1) n > 10
(2) n < 19

(sequences, medium)

Note that s1 = 1 -1/2, s2= 1/2 - 1/3, s3= 1/3 – 1/4 … Thus the sum of the first
n terms of the sequence is 1 – 1/(n+1) , which will be greater than 9/10 as
long as n is at least 10.

(1) SUFF
(2) NOT SUFF

616
Set A, B, C have some elements in common. If 16 elements are in both A and
B, 17 elements are in both A and C, and 18 elements are in both B and C, how
many elements do all three of the sets A, B, and C have in common?

(1) Of the 16 elements that are in both A and B, 9 elements are also in C.
(2) A has 25 elements, B has 30 elements, and C has 35 elements.

(sets, hard)

Note that
n A  B  C   n( A)  n( B)  n(C )  n( A  B)  n( A  C )  n( B  C )  n( A  B  C )

(1) tells us that exactly 9 elements are in all three sets. SUFF
(2) We do not know how many elements are in at least one of the sets A, B
and C. NOT SUFF

617
Set S consists of 5 consecutive integers, and set T consists of seven consecutive
integers. Is the median of the numbers in set S equal to the median of the
numbers in set T?

(1) The median of the numbers in set S is 0.


(2) The sum of the numbers in set S is equal to the sum of the numbers in
set T.

(statistics, medium)

Note that for both sets, the median is equal to the mean.

(1) The median of the numbers in set S is 0. The median of the numbers in
T if and only if the fourth element in T is 0. However, no information is
given about the value of any number in T. NOT SUFF

(2) If s is the sum of the numbers in S and also the sum of the elements in
T, the median of the numbers in S is s/5 and the median of the numbers
in T is s/7. The medians will be equal if and only if s = 0. However,
nothing is said about the actual value of s, nor are we given the means
to find it. NOT SUFF

(T) (1) tells us that the median, and thus the mean of the numbers in S is 0.
This tells us that the sum of the numbers in S is 0. Therefore, (2) tells us
that the median of the two sets of numbers is indeed equal. SUFF

618
Set X has 5 numbers, whose average is greater than their median. Set Y has 7
numbers, whose average is also greater than their median. If the 2 sets have
no number in common and are combined to a new set, is the average of the
new set greater than its median?

(1) The average of Y is greater than the average of X.


(2) The median of Y is greater than the median of X.

(statistics, hard)

(1) y > x. We know that the average of the elements in X  Y is between x


and y. We can say that at least three of the elements of X are less than
this average. If one element in Y is very large, it could be that all other
elements of Y are less than this average, so However, if all elements of
Y are nearly same, all will be above this average.
For example, suppose x=3 , y=5 and X = { 2,2,2,2,7} .
If Y= {3,3,3, ... , 3 , 17} . The average of the combined set X  Y is
greater than 4, but the median is 4.
Conversely, if Y= {4.9, 4.9,4.9,...,4,9, 5.6}, the median, 4.9 is greater
than the mean of X  Y. NOT SUFF.

(2) Since the median of X is less than the median of Y, the median of X  Y
will be less than the median of Y, but greater than the median of X.
Without information about the averages of the elements in X and those
in Y, we cannot answer the question. In each of the two examples cited
in (1), the median of X is less than the median of Y. NOT SUFF

(T) NOT SUFF

619
Seven diffferent integers are selected from the integers 1 to 100, and each
number is divided by 7. What is the sum of the remainders?

(1) The range of the seven remainders is 6.


(2) The seven numbers selected are consecutive integers.

(factors and multiples, medium)

Note that the remainder when an integer is divided by 7 is always an integer


from 0 to 6. Also note that if the remainder when integer n is divided by 7 is 6
(i.e n is one less than a multiple of 7), the remainder when n+1 is divided by 7
is 0, as n+1 is a multiple of 7

(1) One of the remainders is 0 and another is 6, but nothing can be said
about the other 5 remainders other than each is an integer from 0 to 6.
NOT SUFF
(2) The set of remainders consist of each integer from 0 to 6, for example
{1,2,3,4,5,6,0} or {3,4,5,6,0,1,2}. Therefore the sum can be found.
SUFF

620
S is a finite set of numbers. Does S contain more negative numbers than
positive numbers?

(1) The product of all the numbers in S is -1,200.


(2) There are 6 numbers in S.

(inequalities, medium)

(1) That the product of all the numbers in S is negative implies that S
contains an odd number of negative numbers. No information is given
about how many positive numbers that S contains. NOT SUFF
(2) No information is given about the actual values of the elements of S.
NOT SUFF
(T) S could contain 1 negative number and 5 positive numbers or vice versa.
Alternatively, S could contain 3 negative numbers and 3 positive
numbers. NOT SUFF

621
S is the set of all numbers that can be written as a sum involving distinct real
numbers a,b,c and d : |a|/a + |2b|/b + |3c|/c + |4d|/d + |5abcd|/abcd . What
is the range of S ?

20 25 28 29 30

(algebra, statistics, hard)

Note that if a > 0 , |a|/a = 1 and if a < 0 , |a|/a = -1. Also, |2b|=2|b|.
If a, b, c and d are all positive numbers, the above sum is 1+2+3+4+5 = 15.
acbd is negative if there is either 1 or 3 of these four variables are negative. If
a>0 and b, c and d are negative, the above sum is 1 – 2 – 3 – 4 – 5 = -13
Thus the range of S is 28.

20 25 28 29 30

622
S is a set of points in the plane. How many distinct triangles can be drawn that
have three of the points in S as vertices?

(1) The number of distinct points in S is 5.


(2) No three of the points in S are collinear.

(combinatronics, geometry, hard)

(T) Without knowing how many points there are in S and whether more than
two of these points lie on the same line, one cannot answer the
question. With this information, we can say that there are
C35  C25  5  4  2  10 ways of choosing the three vertices. SUFF

623
Six cards numbered from 1 to 6 are placed in an empty bowl. First one card is
drawn and then put back into the bowl; then a second card is drawn. If the
cards are drawn at random and if the sum of the numbers on the cards is 8,
what is the probability that one of the two cards drawn is numbered 5 ?

1/6 1/5 1/3 2/5 2/3

(probability, medium)

Note that as the first card is put back into the bowl, the outcome of the second
draw is independent of the first. The sum of the numbers on the cards drawn
is 8, an event that can happen in five equally probable outcomes: 2+6, 3+5,
4+4, 5+3, 6+2. The probabability of each outcome is 1/5, and two of these
outcomes involve drawing a card numbered 5. Therefore, the probability that
one of the two cards drawn is numbered 5 is 2/5.

624
Six countries in a certain region sent a total of 75 representatives to an
international congress, and no two countries sent the same number of
representatives. Of the six countries, if Country A sent the second greatest
number of representatives, did Country A send at least 10 representatives?

(1) One of the six countries sent 41 representatives to the congress.


(2) Country A sent fewer than 12 representatives to the congress.

(inequalities, hard)

(T) The remaining 5 countries sent a total of 75 – 41 =34 reps. Of course, A


could have sent 10 or more ( as 10 + 9 + 8 + 4 + 3 =34 ) or only 9,
as 9 + 8 + 7 + 6 + 4 =34. NOT SUFF

625
Six machines, each working at the same constant rate, together can complete a
certain job at 12 days. How many additional machines, each working at this
rate, will be needed to complete the job in 8 days?

2 3 4 6 8

(combined work, hard)

To complete this job, six machines have to work for 12 days each. Thus 72
days of machine work are needed. If the job is to be completed in 8 days,
72/8=9 machines are needed, 3 more than the original number of machines.

Alternatively, note that the number of machines needed is inversely


proportional to the number of days alloted for the completion of the job. m =
k/n, where m is the number of machines , n is the number of days and k is a
constant. As the number of days is multiplied by 8/12 =2/3, the number of
machines must be multiplied by 3/2. As there were originally 6 machines
6(3/2)=9 machines are now needed, 3 more than before.

626
Some computers at a certain company are Brand X and the rest are Brand Y. If
the ratio of the number of Brand Y computers to the number of Brand X
computers at the company is 5 to 6, how many of the computers are Brand Y?

(1) There are 80 more Brand X computers than Brand Y computers at the
company.
(2) There is a total of 880 computers at the company.

(ratios, medium)

Suppose there are x Brand X computers and y Brand Y computers at the


company. We are told that y/x = 6/5, i.e. 5y = 6x and asked for the value of y.

(1) x = y + 80 We have a system of two linear equations with two


unknowns, so the value of y can be found. SUFF
(2) x + y = 880 We have a system of two linear equations with two
unknowns, so the value of y can be found. SUFF

627
Some of the students enrolled at college T are part-time students and the rest
are full-time. By what percent did the number of full-time students enrolled at
college T increase from fall of 1999 to the fall of 2000?

(1) There were 50 more full-time students enrolled at college T in the fall of
2000 than in the fall of 1999.
(2) The total number of students enrolled at college T increased by 5
percent from the fall of 1999 to the fall of 2000.

(percents, hard)

Suppose there were f full-time students enrolled in the fall of 1999 and x full-
time students enrolled in the fall of 2000. We can answer the question if and
only if we can find the value of x/f.

(1) x = f + 50 NOT SUFF


(2) If there were p and q part time students enrolled in the fall of 1999 and
the fall of 2000 respectively, (x + q)/(f + p) = 1.05. NOT SUFF
(T) NOT SUFF. An increase of 50 full-time students may be a 5% increase
or indeed a 1% increase or 100% increase

628
Store S sold a total of 90 copies of a certain book during the seven days of last
week, and it sold different number of copies on any two of the days. If for the
seven days Store S sold the greatest number of copies on Saturday, and the
second greatest number of copies on Friday, did Store S sell more than 11
copies on Friday?

(1) Last week Store S sold 8 copies of the book on Thursday.


(2) Last week Store S sold 38 copies of the book on Saturday.

(inequalities, hard)

(1) Clearly not sufficient on it own. NOT SUFF


(2) If Store S sold n copies of the book on Friday, it sold at most n + (n-1)
+(n – 2) + (n -3) + (n -4 ) + (n -5 )= 6n – 15 copies during the six days
of last week that do not include Saturday. Thus 38 + 6n – 15 ≥ 90 , so
6n ≥ 67. Thus n > 11. SUFF

629
Some computers at a certain company are Brand X and the rest are Brand Y. If
the ratio of the number of Brand Y computers to the number of Brand X
computers is 5 to 6, how many of the computers are Brand Y?

(1) There are 80 more Brand X computers than Brand Y computersat the
company.
(2) There is a total of 880 computers at the company.

(ratios, medium)

If there are x and y of each type of computer, we are told that x/y =5/6, i.e.
6x=5y.

(1) x = y + 80 As we have two linear equations, we can solve for x SUFF


(2) x + y = 880 As we have two linear equations, we can solve for x SUFF

630
Square S is inscribed in circle T. If the perimeter of S is 24, what is the
circumference of T?

6π 12π 3√2 π 6√2 π 12√2 π

(geometry, medium)

As the perimeter of the square is 24, each side is 6. Thus the diameter of the
circle is 6√2 and so the circumference is 6√2 π.

631
Susan drove at an average speed of 30mph for the first 30 miles of a trip and
then at an average speed of 60mph for the remaining 30 miles of the trip. If
she made no stops during the trip, what was Susan’s average speed, in miles
per hour, for the entire trip?

35 40 45 50 55

(movement, medium)

Clearly Susan spent more time driving at 30 mph than at 60 mph. Thus her
average speed will be closer to 30 than to 60. In other words, her average
speed will be less than 45. She has travelled a total of 60 miles in 30/30 +
30/60 =1.5 hours, yielding an average speed of 60/1.5= 40 mph.

35 40 45 50 55

632
Tanya prepared 4 different letters to be sent in 4 different envelopes. For each
letter, she prepared an envelope with its correct address. If the 4 letters are to
be put into the 4 envelopes at random, what is the probability that only one
letter will be put into the envelope with its correct address?

1/24 1/8 1/4 1/3 3/8

(probability, hard)

There are 4!= 24 equally probable placements of the 4 letters in the 4 different
envelopes. There are 4 ways to choose which letter will be placed in its proper
envelope, and two ways to distribute the 3 remaining letters among the 3
remaining envelopes so that none of the remaining letters is assigned to its
envelope. That there are two ways to distribute the 3 remaining letters can be
seen by writing the permutations of A B C in which the first letter is not A, the
second letter is not B and the third letter is not C: {B, C, A} , {C, A, B}.
Therefore 4 x 2 = 8 of the 24 permutations of the letters result in exactly one
letter being assigned to its corresponding envelope. The required probability,
then, is 8/24=1/3.

1/24 1/8 1/4 1/3 3/8

633
10 children have ordered a total of 17 hamburgers, and each child has ordered
at least one hamburger. Has any child ordered more than 3 hamburgers?

(1) Exactly two children have ordered three hamburgers each.


(2) No child has ordered exactly two hamburgers.

(inequalities, hard)

We know that each child has ordered at least one hamburger. 7 of the 17
hamburgers have to be accounted for. Let’s call these extra hamburgers.

(1) Two children ordered exactly two extra hamburgers (in addition to the
one hamburger that each ordered), accounting for 4 of the extra
hamburgers. There may have been a starving child who has the three
remaining extra hamburgers ( 4 hamburgers in all), or 3 hungry children
who each had an extra hamburger. NOT SUFF
(2) No child had only one extra hamburger, and the children who had two
extra hamburgers cannot account for the 7 extra hamburgers. There
must have been a child who had at least three extra hamburgers, i.e at
least four hamburgers in all. SUFF

634
Ten nominees for three business awards are randomly seated at the 10 places
at a round table. Three of these nominees are then called up together to
receive the three awards. What is the probability that no two adjacent seats will
be left empty?

1/3 2/5 5/12 7/16 1/2


(probability, hard)

It is easier to find the probability of the converse i.e. the probability that no
two or three adjacent seats will be left empty. There are C310  10  9  8  120
3  2 1
equally probable ways in which three of the ten nominees can be chosen to
receive an award. There are 10 ways in which three adjacent nominees could
be called and 10 x 6 = 60 ways in which two adjacent and one other nominee
could be called. Thus in 70 of the 120 ways (i.e 7/12 of the number of ways) ,
two or three adjacent seats will be left empty. Thus the probability that no two
adjacent seats will be left empty is 1 – 7/12 = 5/12.

1/3 2/5 5/12 7/16 1/2

635
The annual rent collected by a corporation from a certain building was x
percent more in 1998 than in 1997 and y percent less in 1999 than in 1998.
Was the annual rent collected by the corporation from the building more in
1999 than in 1997?

(1) x>y
(2) 100xy < x – y

(percents, hard)

Suppose the rent in 1997 was r.


Then in 1997, the rent was r (1 + x/100).
In 1998, it was r ( 1 + x/100 ) ( 1 – y/100 ).
We need to know whether ( 1 + x/100 ) ( 1 – y/100 ) > 1.
In other terms, is (x – y)/100 – xy/1002 > 0 ?
Simply put, is x – y > xy/100 ?

(1) NOT SUFF


(2) x – y > 100xy > xy/100 ( as xy > 0 ) SUFF

636
Theater M has 25 rows with 27 seats in each row. How many of the seats were
occupied during a certain show?

(algebra, medium)

(1) During the show, there was an average (arithmetic mean) of 10


unoccupied seats per row for the front 20 rows.
(2) During the show, there was an average (arithmetic mean) of 20
unoccupied seats per row for the back 15 rows.

(T) As the 2 statements give overlapping information (rows 11-20 are


included in both statements), we cannot determine the number of
unoccupied seats among the 25 rows.
Suppose rows 11-20 have x unoccupied seats in all.
(1) tells us that rows 1-10 have 200 – x unoccupied seats
(2) tells us that rows 21-25 have 300 – x unoccupied seats.
In all, then, there are 500 – x unoccupied seats, and without a definite
number of unoccupied seats, we cannot answer the question.

NOT SUFF

637
The attendees at a certain convention purchased a total of 15,000 books. How
many attendees were female?

(1) There were a total of 4,000 attendees at the convention.


(2) Each male attendee bought 3 books and each female attendee bought 5
books.

(algebra, medium)

(T) Obviously neither statement is sufficient by itself. Together, we see that


the m men bought 3 books each and the w women bought 5 books each,
so we get two linear equations with two variables:
m + w = 4000 and 3m + 5w = 15000
We can solve this system of linear equations for w and thus answer the
question. SUFF

638
The average (arithmetic mean) score on a test taken by 10 students was
x. If the average score for 5 of the students was 8, what was the
average score, in terms of x, for the remaining 5 students who took the
test?

2x – 8 x–4 8 – 2x 16 – x 8 – x/2

(algebra, medium)

The sum of the 10 scores is 10x, so the sum of the remaining 5 scores is
10x – 5(8) = 10x – 40. Thus the average of the remaining 5 scores was
(10x- 40)/5 = 2x – 8.

Alternatively, think of the 10 students as divided into two groups of 5


students each. The difference between 8 and x is 8 – x. Thus the
difference between x and y, the required average, is also 8 – x, as there
are equal numbers of scores in the two groups. x – y = 8 – x, so y = 2x -
8

2x – 8 x–4 8 – 2x 16 – x 8 – x/2

639
The buyer of a new home must choose 2 of 4 types of flooring and 2 of 6 paint
colors. How many different combinations of type of flooring and paint color are
available to the home buyer?

4 24 48 90 96

(combinatronics, medium)

The order in which the types and colours are chosen makes no difference, so
there is 4 C2 = 4 x 3/ 2! = 6 ways of choosing the two types of flooring and 6 C2
= 6 x 5/ 2! = 15 ways of choosing the two paint colours. Therefore, there are 6
x 15 = 90 different combinations available to the buyer.

4 24 48 90 96

640
The cost of a square slab is proportional to its thickness and also proportional
to the square of its length. What is the cost of a square slab that is 3 meters
long and 0.1 meter thick?

(1) The cost of a square slab that is 2 metre long and 0.2 meter thick is
$160 more than the cost of slab that is 2 metre and 0.1 meter thick.
(2) The cost of a square slab that is 3 meter long and 0.1 meter thick is
$200 more than the cost of slab that is 2 metre and 0.1 meter thick.

(ratios, medium)

The cost of a square slab m meters long and t meters thick can be expressed as
km2t, where k is a constant. We are asked to find k(32)0.1 = 0.9k. If we can
find the value of k, we can answer the question.

(1) 0.8k = 160 + 0.4k SUFF


(2) 0.9k = 200 + 0.4k SUFF

641
The cost of delivery for an order of desk chairs was $10.00 for the first chair,
and $1.00 for each additional chair in the order. If an office manager placed an
order for n desk chairs, is n > 24 ?

(1) The delivery cost for the order totalled more than $30.00.
(2) The average (arithmetic mean) delivery cost of the n chairs was $1.36.

(inequalities, medium)

If n chairs are delivered, the delivery cost C is 10 + n – 1 = 9 + n.

(1) As C > 30, 9 + n > 30, so n > 21. n may or may not be greater than
24. NOT SUFF
(2) The average delivery cost is (9+n)/n = 1.36 Therefore 9 + n =1.36n,
and the exact value of n can be determined. SUFF

642
The cost of each adult's ticket for a certain concert was $30, and the cost of
each child's ticket was $24. If Hannah purchased tickets for this concert, what
was the average cost per ticket?

(1) Hannah purchased 2 times as many children's tickets as adult tickets.


(2) Hannah purchased 4 children's tickets.

(algebra, ratios, hard)

(1) If Hannah has purchased t adult tickets. (1) says that she bought 2t
children’s tickets. Thus she spent a total of 48t + 30t = 78t dollars on a
total of 3t tickets, so the average cost per ticket is 78t/3t = 26.
SUFF
(2) The average cost will vary from 24 to almost 30, depending on the
number of adult tickets purchase. NOT SUFF

643
The cost of renting a mower consists of a charge of $15 for up to 3 hours plus
a charge of $4 for each hour after the first 3 hours. Which of the following
gives the cost, in dollars, of renting the lawn mower for x hours, where x is an
integer greater than 3 ?

3x + 3 3x + 15 4x + 3 4x + 15 4x + 27

(algebra, medium)

The first 3 hours cost a total of 15 dollars, and the remaining (x- 3) hours cost
a total of 4(x – 3) dollars, for a grand total of 15 + 4(x – 3) = 3 + 4x.

3x + 3 3x + 15 4x + 3 4x + 15 4x + 27

644
The diameter of circle S is equal in length to the side of a certain square. The
diameter of circle T is equal in length to the diagonal of the same square. The
area of circle T is how many times the area of circle S ?

(geometry, medium)

Suppose the square has side s. Then the diameter of S is s and the diameter of
T is . Since the area of a circle is directly proportional to the square of its
radius, the ratio of the area of T to that of S is equal to the square of the ratio
: s, i.e. 2:1. Thus the area of circle T is twice that of circle S.

645
The figure above shows two entries, indicated by m and n, in an addition table.
What is the value of n + m?

(1) d + y = -3
(2) e + z = 12

(algebra, medium)

(T) m + n = e + y + d + z = (d + y) + (e + z) SUFF
646
The figure above represents a square garden that is divided into 9 rectangular
regions with the indicated dimensions in meters. The shaded regions are
planted with peas, and the unshaded regions are planted with tomatoes. If the
total area planted with peas is equal to the total area planted with tomatoes,
what is the value of x?

0.5 1 1.5 2 2.5

(algebra, geometry, hard)

Note that the area of the square garden is 9×9= 81 square meters. Thus the
combined area of the shaded regions must be 81/2. Each of the 5 shaded
regions has a vertical dimension of 3 meters. 2 of these regions have a
horizontal dimension of x, 2 have a horizontal dimension of 3 and 1 has a
horizontal dimension of 6 – x. Therefore, 6x + 18 + 3(6 – x) = 81/2
and 3x = 9/2. Thus x= 3/2 = 1.5

0.5 1 1.5 2 2.5

647
The figure above shows the dimensions of a semicircular cross section of a one-
way tunnel. The single traffic lane is 12 feet wide and is equidistant from the
sides of the tunnel. If vehicles must clear the top of the tunnel by at least ½
foot when they are inside the traffic lane, what should be the limit on the
height of vehicles that are allowed to use the tunnel?

5½ ft 7½ ft 8 ½ ft 9½ ft 10 ft

(geometry, medium)

Make sure you know the four important right triangles: 3-4-5 , 5-12-13, 1-1-
1- -2.

5½ ft 7½ ft 8 ½ ft 9½ ft 10 ft

648
The figure shown above represents a board with 4 rows of pegs, and at the
bottom of the board are 4 cells numbered 1 to 4. Whenever the ball shown
passes through the opening between adjacent pegs in the same row it will hit
directly the peg directly beneath the opening. . The ball has a probability of 1/2
of passing through the opening immediately to the left of that peg and 1/2 of
passing though the opening immediately to the right. What is the probability
that when the ball passes through the first two pegs at the top, it will end up in
cell 2?

1/4 1/3 3/8 2/5 1/2

(probability, hard)

This experiment is akin to tossing a fair coin three times, the desirable outcome
being two heads and one tail. HTH HHT THH each has a probability of (1/2)3 =
1/8, so the required probability is 3/8.

1/4 1/3 3/8 2/5 1/2

649
The figure shows seven train stations and the distances, in miles, along the
railways that connect these stations. Beginning at one of these stations, a train
takes 25 hours to travel directly to station W at an average rate of 50 miles per
hour. At which of the stations did the train begin?

T U V Y Z
(movement, medium)

It travelled 1250 = 50(25) miles from a station to W. Inspection reveals that


only U is that distance from W.

T U V Y Z

650
The figure shows the top side of a circular medallion made of a circular piece of
colored glass surrounded by a metal frame. If the radius of the medallion is r
and the width of the metal frame is s, then in terms of r and s, what is the area
of the metal frame in square centimetres?

 r  s 2  r 2  s 2  2 (r  s) r 2r  s  s2r  s 

(geometry, hard)

The area of the metal frame is simply the difference between the area of the
entire medallion , which has a radius of r, and the area of the glass, which has
a radius of r – s. Thus the area of the frame is
r 2   (r  s)2   (r 2  (r  s)2 )   (r  (r  s))(r  (r  s))  s(2r  s)

 r  s 2  r 2  s 2  2 (r  s) r 2r  s  s2r  s 

651
The function f is defined by f(x) = - 1/x for all nonzero numbers x. If f(a) = -
1/2 and f(ab) = 1/6, then b =

3 1/3 - 1/3 -3 –12

(functions, medium)

As f(ab) = 1/6, -1/ab = -1/6 and thus ab =6


Also that f(a) = -1/a =-1/2 implies that a =2. Therefore b = -3

3 1/3 - 1/3 -3 –12

652
The function f is defined for all positive integers n by the following rule: f(n) is
the number of positive integers each of which is less than n and has no positive
factor in common with n other than 1. If p is any prime number, then f(p)=

p-1 p-2 (p + 1)/2 (p - 1)/2 2

(functions, factors and multiples, hard)

Note that for any prime number p, there are p – 1 positive integers, each of
which has no factor in common with p other than 1.Thus f (p) = p – 1.

p-1 p-2 (p + 1)/2 (p - 1)/2 2

653
The function f is defined for each positive three-digit integer n by
f(n) = 2x 3y 5z, where x, y and z are the hundreds, tens, and units digits of n,
respectively. If m and v are three-digit positive integers such that f(m) = 9f(v),
then m -v = ?

8 9 18 20 80

(functions, factors and multiples, hard)

Since f(m) = 32 f (v), the only difference between m and v is that the tens digit
of m is 2 greater than the tens digit of v. Thus m= v + 20 and m – v = 20.

8 9 18 20 80

654
The height of women in Dewaria follows a normal distrubution with mean 160
cm and standard deviation of 6 cm. In a normal distribution, only 6.3 x 10-3 %
of the population is not within 4 standard deviations of the mean. If 5 women
are more than 184 cm tall, approximately how many women live in Dewaria?

16,000 40,000 80,000 100,000 160,000

(statistics, hard)

184= 160 + 4(6), so there are 5 women whose height is more than 4 standard
deviations above the mean and thus 5 women whose height is more than 4
standard deviations below the mean. Thus 10, the number of women whose
height is not within 4 standard deviations of the mean, is 6.3 x 10-3 % of the
population of women in Dewaria, P. Thus P= 10/6.3 x 10-5 = 106/6.3 ≈160,000

16,000 40,000 80,000 100,000 160,000


655
The graph of which of the following equations is a straight line that is parallel to
line L in the figure above?

3y – 2x = 0
3y + 2x = 0
3y + 2x = 6
2y – 3x = 6
2y + 3x = -6

(coordinate geometry, medium)

The slope of line L is (2 – 0)/(0 – (-3))= 2/3

Note that the slope of Ax + By = C (i.e. y= (-A/B)x +C/B is –A/B

Thus for Ax + By =C to be parallel to L , -A/B = 2/3 , A/B =-2/3 =2/ -3

3y – 2x = 0
3y + 2x = 0
3y + 2x = 6
2y – 3x = 6
2y + 3x = -6

656
The integers m and p are such that 2 < m < p and m is not a factor of p. If r is
the remainder when p is divided by m, is r > 1 ?

(1) The greatest common factor of m and p is 2.


(2) The least common multiple of m and p is 30.

(factors and multiples, hard)

(1) Both m and p are divisible by 2, so m and p are even integers. Since m is
not a factor of p, the remainder when p is divided by m cannot be 0. Nor
can it be 1, for if it were, either p or m would be an odd integer.
Therefore, the remainder in question must be greater than 1. SUFF
(2) Note that (m,p) could be (5,6) or (6,10). The remainder when p is
divided by m is 1 in the first case but 4 in the second case.
NOT SUFF

657
The interior of a rectangular carton is designed by a certain manufacturer to
have a volume of x cubic feet and a ratio of length to width to height of 3:2:2.
In terms of x, which of the following equals the height of the carton, in feet?

(geometry, ratios, hard)

Suppose the length, width and height of the carton in feet are 3k, 2k and 2k
respectively. The volume would be 12k3 cubic feet, and we need to find the

height, 2k. But since 12k3 = x , k = and 2k = = =

658
The lifetimes of all the batteries produced by a certain company in a year have
a distribution that is symmetric about the mean m. If the distribution has a
standard deviation of d, what percent of the distribution is greater than m +d?

(1) 68 percent of the distribution lies in the interval from m-d to m+d,
inclusive .
(2) 16 percent of the distribution is less than m-d.

(statistics, hard)

(1) The percent greater than m + d is equal to the percent less than m – d.
The sum of these two is 100 – 68 =32%, so the percent greater than
m + d is 16%. SUFF
(2) The percent greater than m + d is equal to the percent less than m – d.
SUFF

659
70 75 80 85 90
105 105 130 130 130

The list shown consists of the times, in seconds, that it took each of 10 school
children to run a distance of 400 meters. If the standard deviation of the 10
running times is 22.4 seconds, rounded to the nearest tenth of a second, how
many of the 10 running times are more than 1 standard deviation below the
mean of the 10 running times?

One Two Three Four Five

(statistics, hard)

The mean of the running times is 100. The easiest way to see this is to
calculate their average deviation from 100. If a running time were exactly 1
standard deviation below the mean running time, it would be 100-22.4= 77.6.
Two of the 10 times are below 77.6 i.e. more than 1 standard deviation below
the mean of the 10 running times.

One Two Three Four Five

660
The mass of 1 cubic meter of a substance is 800 kilograms under certain
conditions. What is the volume, in cubic centimeters, of 1 gram of this
substance under these conditions? (1 kilogram=1000 grams and 1 cubic meter
= 1,000,000 cubic centimeters)

0.80 1.25 8 12.5 80

(ratios, hard)

0.80 1.25 8 12.5 80


661
The main ingredient in a certain prescription drug capsule costs $500.00 per
kilogram. If each capsule contains 600 milligrams of this ingredient, what is the
cost of the ingredient in a capsule? (1 kilogram = 106 milligrams)

$0.30 $0.83 $1.20 $3.00 $3.33

(ratios, hard)

$0.30 $0.83 $1.20 $3.00 $3.33

662
The maximum temperature, in degrees Fahrenheit, recorded in a city on 5
consecutive days were x+3, 95, 87, x+7 and x. If the average (arithmetic
mean) of these temperatures was 90 degrees Fahrenheit, what is the value of
x?

85 86 87 88 89

(algebra, easy)

The sum of the 5 temperatures 3x + 192 is equal to 90 multiplied by 5


Thus 3x is equal to 90(5) – 192, so x= 30(5) – 64 = 86.

85 86 87 88 89

663
9.4 9.9 9.9 9.9 10.0 10.2 10.2 10.5

The mean and the standard deviation of the 8 numbers shown are 10 and 0.3,
respectively. What percent of the 8 numbers are within 1 standard deviation of
the mean?

90% 85% 80% 75% 70%

(statistics, hard)

2 of the 8 numbers are not between 9.7 and 10.3 , in other words, not within 1
standard deviation of the mean. Thus 6 of the 8 numbers (75%) are within 1
standard deviation of the mean.

90% 85% 80% 75% 70%

664
The members of a certain club were asked whether they speak Cantonese,
Mandarin and Japanese. 100 spoke Cantonese, 150 spoke Mandarin and 200
spoke Japanese. If 120 spoke exactly two of the three languages, how many
members does the club have?

(1) There are twice as many members who speak none of the languages as
there are who speak all three languages.
(2) Half of the members who speak Japanese and Cantonese also speak
Mandarin.

(sets, hard)

Suppose that the club has T members, x of whom speak all three of these
languages and y of whom speak none of the these languages.

T = (100 + 150 + 200) – 120 – 2x + y

(1) y = 2x SUFF

(2) There are at most 120 members speak both Japanese and Cantonese, so
at most 60 members speak all three languages. As no information is given
about y, the number of people who speak none of the languages, the value of
T cannot be determined. NOT SUFF

665
The number n of units of its product that Company X is scheduled to produce in
900
month t of its next fiscal year is given by the formula n = , where c is a
1  c 2 t
constant and t is a positive integer between 1 and 6, inclusive. What is the
number of units of its product that Company X is scheduled to produce in
month 6 of its next fiscal year?

(1) Company X is scheduled to produce 180 units of its product in month 1


of its next fiscal year.
(2) Company X is scheduled to produce 300 units of its product in month 2
of its next fiscal year.

(functions, medium)

We are asked for the value of n corresponding to t = 6. It would be sufficient to


know the value of c.

(1) 180 = 900 / (1 + 0.5c) SUFF


(2) 300 = 900 / (1 + 0.25c) SUFF

666
The numbers of books read by 7 students last year were 10, 5, p , q , r , 29
and 20. What was the range of the numbers of books read by the 7 students
last year?

(1) 5<p<q
(2) p < r < 15

(statistics, medium)

(1) p and q can be arbitrarily large. All that can be said is that the range is at
least 19. NOT SUFF
(2) p and q can be arbitrarily small. NOT SUFF
(T) The smallest of the numbers is 5 and the largest is either 29 or q.
NOT SUFF

667
The number of defects in the first five cars to come through a new production
line are 9, 7, 10, 4, and 6, respectively. If the sixth car through the production
line has either 3, 7, or 12 defects, for which of theses values does the mean
number of defects per car for the first six cars equal the median?

I. 3
II. 7
III. 12

I only II only III only I and III only I, II, and III

(statistics, medium)

Suppose the sixth car has x defects. The mean number of defects per car for
the first six cars is (36 + x)/6 = 6 + x/6

If x =3, the median is (6+7)/2= 6.5 = mean


If x=7, the mean is 7 + 1/6, which cannot be the median of a set of integers.
If x=12, the median is 8 = mean

I only II only III only I and III only I, II, and III

668
The number of stamps that Kaye and Alberto had were in the ratio of 5 : 3,
respectively. After Kaye gave Alberto 10 of her stamps, the ratio of the number
Kaye had to the number Alberto had was 7 : 5. As a result of this gift, Kaye had
how many more stamps than Alberto?

20 30 40 60 90

(ratios, hard)

Suppose Kaye and Alberto first had 5x and 3x stamps respectively. After Kaye
gave Alberto 10 of her stamps, Kaye and Alberto had 5x -10 and
3x + 10 stamps repectively, numbers that are in a 7:5 ratio.
Thus 5(5x – 10) =7(3x + 10) and 4x = 120, x=30. As a result of this gift, Kaye
had (5x – 10) - (3x + 10)= 2x - 20 = 40 stamps more than Alberto.

20 30 40 60 90

669
The number 75 can be written as the sum of the squares of 3 different
integers. What is the sum of these 3 integers?

17 16 15 14 13

(exponents, medium)

Look at the perfect squares that are less than 75: 64, 49, 36, 25, 16, 9 ,4, 1.
Since 75= 49 + 25 + 1 = 72 + 52 + 12, the sum of the three integers is 13.

17 16 15 14 13

670
The numbers x and y are NOT integers. The value of x is closest to which
integer?

(1) 4 is the integer that is closest to x+y


(2) 1 is the integer that is closest to x-y

(inequalities, hard)

Neither statement is sufficient by itself.


(1) says that 3.5 < x + y < 4.5.
(2) says that 0.5 < x - y < 1.5.
(T) Adding the two inequalities, 4 < 2x < 6, i.e 2 < x < 3.
Thus x may be closest to 2 to or 3. NOT SUFF

671
The numbers x and y are three-digit positive integers, and x + y is a four-digit
integer. The tens digit of x equals 7 and the tens digit of y equals 5. If x < y,
which of the following must be true?

I. The units digit of x + y is greater than the units digit of either x or y.


II. The tens digit of x + y equals 2.
III. The hundreds digit of y is at least 5.

II only III only I and II I and III II and III

(algebra, medium)

Note that x + y ≥ 1000

I. Need not be true: if the sum of the units digits of x and y is greater than
9, I will not be true, nor will be it be true if the units digits of the x and y
are not both positive.
II Need not be true: if the sum of the units digits of x and y is greater than
9, the units digit of x + y will be 3, not 2.
III. True: If the units digit of y were less than 5, y would be less than 500
and x + y would be a 3-digit number, not a 4-digit number.

II only III only I and II I and III II and III

672
The operation ⊗ is defined for all nonzero numbers a and b by a ⊗ b = a/b –
b/a. If x and y are nonzero numbers, which of the following statements must
be true?

I. x ⊗ xy = x(1 ⊗ y)

II. x ⊗ y = -(y ⊗ x)

III. 1/x ⊗ 1/y = y ⊗ x

I only II only III only I and II II and III

(functions and sequences, hard)

Note that a ⊗ b = (a2 – b2) / ab

Thus x ⊗ xy = ( x2 – x2y2 ) / x2y = ( 1 – y2 )/ y = 1 ⊗ y I not always true


x2 – y2 = -(y2 – x2) II always true
1/x ⊗ 1/y = (1/x)2 – (1/y)2 / (1/xy) =
( y2 – x2 ) / xy = y ⊗ x III always true

I only II only III only I and II II and III

673
The operation Ɵ represents either addition, subtraction, or multiplication of
integers. What is the value of 1 Ɵ 0 ?

(1) 0Ɵ2=2
(2) 2Ɵ0=2

(functions and sequences, medium)

(1) Since 0 + 2 = 2 , 0 – 2 = -2 and 0 x 2 = 0, Ɵ must be addition, so 1 Ɵ 0


=1+0 SUFF
(2) Since 2 – 0 = 2 + 0 = 2 and 2 x 0 = 0, Ɵ must be either addition or
subtraction. However 1 – 0 = 1 + 0 SUFF

674
The participants in a race consisted of 3 teams with 3 runners on each team. A
team was awarded 6 – n points if one of its runners finished in n th place,
where 1 ≤ n ≤ 5. If all of the runners finished the race and if there were no
ties, was each team awarded at least one point?

(1) No team was awarded more than a total of 6 points.


(2) No pair of team-mates finished in consecutive places among the top five
places.

(algebra, hard)

(1) If no team was awarded more than 6 points, the top 2 teams in terms of
points were awarded at most 12 points. However, 5 + 4 + 3 + 2 + 1=
15 points were awarded in all, so the last team in terms in points
received at least 3 points. SUFF
(2) Members of the top 2 teams could have taken alternative places: one
team placing runners in 1st , 3rd and 5th places and another placing
runners in 2nd and 4th places. NOT SUFF

675
The perimeter of a certain isosceles right triangle is 16 +16 2 . What is the
length of its hypotenuse?

8 16 4 2 8 2 16 2
(geometry, hard)

Suppose the length of its hypotenuse is h. Thus 2x2 = h2, where x is the length
of each of the two legs of this triangle. Since 2x = h 2 , the perimeter of the
triangle is 2x + h = h + h 2 . Thus h=16

8 16 4 2 8 2 16 2

676
The perimeter of square region S and rectangular region R are equal. If the
sides of R are in the ratio 2:3, what is the ratio of the area of region R to the
area of region S?

25:16 24:25 5:6 4:5 4:9

(geometry, hard)

Suppose that the width and length of the rectangle are 2x and 3x respectively.
The perimeter of the rectangle is 10x, and thus each side of the square is 5x/2.
The area of R is 6x2 and that of the square is 25x2/4. The ratio of these areas is
24:25.

25:16 24:25 5:6 4:5 4:9

677
The points A, B, C and D are on a number line, not necessarily in that order. If
the distance between A and B is 18 and the distance between C and D is 8,
what is the distance between B and D?

(1) The distance between C and A is equal to the distance between C and B.
(2) A is to the left of D on the number line.

(algebra, hard)

(T) If one statement is clearly insufficient, such as (2) in this question, one
can begin by combining the two statements. Suppose that A=0 and B=18. (1)
says C is halfway between B and C, so C = 9. As D is 8 units from C, D could be
17 or 1, both of which are greater than 0, the value of A, satisfying (2). Thus
the distance from B to D could be 1 or 17. NOT SUFF

678
The points R, T, and U lie on a circle that has radius 4. If the length of arc RTU
is 4  /3, what is the length of line segment RU?

4/3 8/3 3 4 6

(geometry, hard)

The length of arc RTU is 2r   , where r is the radius of the circle and Ɵ is
360
the angle of intersection of line segments OR and OU, where O is the center of
the circle. Thus Ɵ = 60º, and triangle ORU is an equilateral triangle. Thus the
length of line segment RU is 4.

4/3 8/3 3 4 6

679
The positive integer k has exactly two positive prime factors, 3 and 7. If k has a
total of 6 positive factors, including 1 and k, what is the value of k?

(1) 32 is a factor of k.
(2) 72 is NOT a factor of k.

(factors and multiples, medium)

If 3 and 7 are prime factors of k, then k is a multiple of 21 and every factor of


21: 1, 3, 7 and 3 x 7. As k has 6 positive factors, k must be a multiple of either
32 and thus 7 × 32 (in which case k =7 × 32 ) or 72 and thus 3 × 72 (in which
case k = 3 × 72 ).

(1) SUFF
(2) SUFF

680
The prime sum of a integer n greater than 1 is the sum of all the prime factors
of n, including repititions. For example, the prime sum of 12 is 7, since 12 = 2 x
2 x 3 and 2 + 2 + 3 = 7. For which of the following integers is the prime sum
greater than 35?

440 512 620 720 750

(factors and multiples, medium)

440 = 44 x 10 = 2 x 2 x 2 x 5 x 11 Sum 22
512 = 29. Sum 18
620 = 10 x 62 = 2 x 2 x 5 x 31 Sum 40

440 512 620 720 750

681
The product of the units digit, the tens digit, and the hundreds digit of the
positive integer m is 96. What is the units digit of m?

(1) m is odd.
(2) The hundreds digit of m is 8.

(factors and multiples, hard)

Note that 96=25 × 3.

(1) m is odd if and only the units digit of m is odd. The only odd divisors of
96 are 1 and 3. The units digit of m cannot be 1, as no two positive
digits have a product of 96. Thus the units digit of m is 3. SUFF
(2) The product of the tens and the units digit is 96/8=12, so the units digit
can be any positive factor of 12 other than 1 and 12 itself. NOT SUFF

682
The positive integers x, y and z are such that x is a factor of y and y is a factor
of z. Is z even?

(1) xz is even.
(2) y is even.

(factors and multiples, hard)

(1) As x is a factor of y and y is a factor of z, x is a factor of z. If z were odd,


x would also be odd, as would be xz. Since xz is even, not odd, z must
be odd as well. SUFF
(2) Since y is a factor of z, z is a multiple of y. Since y is even, z is also even.
SUFF

683
The ―prime sum‖ of an integer n greater than 1 is the sum of all the prime
factors of n, including repetions. For example, the prime sum of 12 is 7, since
12 = 2×2×3 and 2 + 2 + 3 = 7. For which of the following integers is the
prime sum greater than 35 ?

440 512 620 700 750

(factors and multiples, medium)

Look for a number with a large prime factor: 620 = 62 × 10 , which is a


multiple of 31. The prime sum of 620 will be greater than 35.

440 512 620 700 750

684
The range of set A is r. If a number with a value equal to r is added to set A,
will the range of set A increase?

(1) All numbers in the set are positive


(2) The mean of the new set is smaller than R

(statistics, hard)

(1) If the original numbers in A have a range that is less than the smallest
element in A, the range will increase. For example A={3,4}. Otherwise,
the range will not increease. For example A ={3,6}. NOT SUFF

(2) r is greater than the mean of set A. If the elements of A are all negative
r will be greater than the greatest element in A and the new range will
be higher. If the numbers are all positive, r , greater than the mean of A,
must be greater than the smallest element in A, Also since r is the
difference between the greatest and smallest element in A, r must be
less than the greatest element in A. Thus adding r to set A will not affect
the range of the set. NOT SUFF

(T) SUFF

685
The ratio of the number of boys to the number of girls in a certain school club
is 3 : 4. If there are 5 more girls than boys in the club, how many girls are in
the club?

7 8 12 15 20

(ratios, medium)

If the number of boys is 3x, the number of girls is 4x, and there are x more
girls than boys. Since x=5, the club has 4x=4(5)= 20 girls.

7 8 12 15 20

686
The rate of a certain chemical reaction is directly proportional to the square of
the concentration of chemical A present and inversely proportional to the
concentration of the chemical B present. If the concentration of the chemical B
is increased by 100 percent, which of the following is closest to the percent
change in the concentration of chemical A required to keep the reaction rate
unchanged?

100% decrease 50% decrease 40% decrease


40% increase 50% increase

(ratios, hard)

We are told that the rate of reaction, r , is equal to ka2/b, where k is a


constant, a is the concentration of A and b is the concentration of B
Suppose the concentration of B is increased from b to 2b, an increase of 100%
and the concentration of A is changed from a to x. If the rate of reaction is to
ka2 kx2
remain constant,   x  2a  1.4a . Thus the concentration of A is
b 2b
increased by approximately 40%.

100% decrease 50% decrease 40% decrease


40% increase 50% increase

687
There are 15 slate rocks, 20 pumice rocks, and 10 granite rocks randomly
distributed in a certain field. If 2 rocks are to be chosen at random and without
replacement, what is the probability that both rocks will be slate rocks?

1/15 7/66 1/9 1/10 2/10

(probability, medium)

The probability that the first rock chosen is a slate rock is 15/45 = 1/3.
If the first rock is slate, the probability that the second slate rock chosen is a
slate rock is 14/44 = 7/22.
Thus the probability that both rocks are slate is 1/3 x 7/22 = 7/66.

1/15 7/66 1/9 1/10 2/10

688
In a class of 30 students, 2 did not borrow any book, 12 students borrowed 1
book each and 10 students borrowed 2 books each and the remaining students
borrowed at least 3 books. If the average (arithmetic mean) number of books
borrowed per student was 2, what was the maximum number of books that any
single student could have borrowed ?

4 6 8 13 15

(inequalities, hard)

60 books were borrowed: the 6 students who borrowed at least 3 books


borrowed 60 – 12 – 20 =28 books. If five of these borrowed exactly 3 books
each, the sixth borrowed 13 books.

4 6 8 13 15

689
There are two bars of gold-silver alloy; The first is 40% gold and 60% silver,
the second is 30% gold and 70% silver. If both bars are melted into an 8 kg.
bar that is 5 parts gold and 11 parts silver, what was the weight in kilograms of
the first bar?

1 3 5 6 7

(ratios,hard)

Suppose weight of first bar is x: 2x/5 is gold. The weight of second bar is 8- x :
(3/10)(8-x) is gold.
Total gold = 24/10 + x/10 =(5/16)8=5/2
x/10 = 1/10
x=1

1 3 5 6 7

690
There were 2 apples and 5 bananas in a basket. After additional apples and
bananas were placed in the basket, the ratio of the number of apples to the
number of bananas was 1:2. How many apples were added?

(1) The number of apples added was 2/3 of the number of bananas added.
(2) A total of 5 apples and bananas were added.

(ratios, medium)

Suppose that x apples and y bananas were added to the basket, so that the
basket then contained 2 + x apples and 5 + y bananas. We are told that 5 + y
= 2(2 + x). Another other linear equation involving x and y will suffice, as long
as it does not represent a parallel line

(1) x = 2y/3 SUFF


(2) x+y=5 SUFF

691
The relation between Celsius (C) and Fahrenheit (F) temperature readings is
given by the following formula C  5 ( F  32) .
9
At what temperature are the Celsius and Fahrenheit readings identical?

-60 -40 0 24 32

(algebra, medium)

If C = F, F  5 ( F  32)  9 F  F  32  F   32  5  40
9 5 4

-60 -40 0 24 32

692
The selling price of an article is equal to the cost of the article plus the markup.
The markup on a certain television set is what percent of the selling price?

(1) The markup on the television set is 25 percent of the cost.


(2) The selling price of the television is $250.

(percents, medium)

If the selling price of the television is s and the its cost is c, the markup m is
equal to s – c. We are asked to find 100m/(m+c)

(1) m=0.25c, so 100m/(m+c) = 25c/1.25c SUFF


(2) s = 250 NOT SUFF

693
2
 9  80  9  80  
 

1 94 5 18  4 5 18 20

(exponents, algebra, hard)

Remember that (a + b )2 = a2 + 2ab + b2


2 2
 9  80  9  80   ( 9  80 )2  2 9  80 9  80   9  80  
   
9  80  2 92  80  9  80  18  2(1)  20

1 94 5 18  4 5 18 20

694
The sale price of certain jacket was 15 percent less than its original price, and
the sale price of a certain shirt was 10 percent less than its original price. How
much greater was the original price of the jacket than the original price of the
shirt?

(1) The sale price of the jacket was $83 greater than the sale price of the
shirt.
(2) The original price of the jacket was $140.

(percents, algebra, medium)

We are asked to find j – s, where j and s are the regular prices of the jacket
and shirt respectively.

(1) 0.85j – 0.9s = 83 NOT SUFF


(2) j = 140
(T) SUFF

695
The total of Company C's assets in 1994 was 300 percent greater than the total
in 1993, which in turn was 400 percent greater thatn the total in 1992. If the
total of Company C's assets in 1992 was N dollars, which of the following
represents the total of Company C's assets, in dollars, in 1994?

7N 8N 9N 12N 20N

(percents, medium)

Remember that x(1 + p/100) is p% greater than x.

If the assets in 1992 totalled N, in 1993, they totalled N(1 + 300/100)=4N


In 1994, they totalled 4N (1 + 400/100) = 20N

7N 8N 9N 12N 20N

696
The sequence a1 , a2 ,a3 ,..., an of n integers is such that ak = k if k is odd and
ak =– ak-1 if k is even. If the sum of the terms in the sequence positive?

(1) n is odd.
(2) an is positive.

(sequences, hard)

(1) If n is odd, the sum of the terms is


1 – 1 + 3 – 3 + . . . + (n – 2) – (n – 2) + n = n > 0. SUFF
(2) If an is positive, n must be odd, and the sum is as in (1). SUFF

697
The sequence a1, a2, … , an, … is such that an = 2an-1 - x for all positive
integers n ≥ 2 and for a certain number x. If a5 = 99 and a3 = 27, what is the
value of x?

3 9 18 36 45

(sequences, medium)

We know that a4 = 2a3 – x and a5 = 2a4 – x = 2(2a3 – x) – x = 4a3 – 3x


Thus 99 =4(27) – 3x and x=3

3 9 18 36 45

698
The sum of all factors of 100000 which are divisible by 10 is

200000 305000 275000 231250 242110

(factors and multiples, hard)

The sum of all of the factors that are multiples of 2 but not of 22 is
10 + 50 + 250 + 1250 + 6250 = 7810 = S

The sum of all factors of 100,000 which are divisible by 10 is


S + 2S + 4S + 8S + 16S = 31S = 242110

699
The sum of positive integers x and y is 77. What is value of xy?

(1) x = y+1
(2) x and y have the same tens digit.

(algebra, medium)

(1) Since x + y =77, (y+1) + y =77. Thus y and x can be determined, as


can the value of the product xy. SUFF
(2) As the sum of x and y is 77, their average is 77/2= 38.5.
Thus x=38.5 + k and y = 38.5 – k for some number k. For x and y to
have the same tens digit, -1.5 < k < 1.5. For x and y to be integers, k
could be 0.5 so that x=39 and y=38 or k could be -0.5, so that x=38 and
y=39. In either case, the product xy is 38 multiplied 39. SUFF

700
The sum of the first 50 positive even integers is 2,550. What is the sum of even
integers from 102 to 200 inclusive ?

5,100 7,550 10,100 15,500 20,100

(algebra, hard)

It is given that 2 + 4 + ... + 98 + 100 = 2,550.


102 + 104 + ... + 198 + 200 = (100 + 2) + (100 + 4) + ... + (100 + 98) +
(100 + 100) = (2 + 4 + ... + 98 + 100) + 50 (100) = 2,550 + 5,000 = 7, 550

5,100 7,550 10,100 15,500 20,100

701
The sum of the reciprocals of two numbers is 46/21 and the difference of their
reciprocals is 10/21. What are the 2 numbers?

3, 3/29 3/4 , 7/6 4/3, 6/7 8/21, 5/21 1/21, 1/21

(algebra, medium)

1 1 46 1 1 10
Suppose the two numbers are x and y and x < y.   and  
x y 21 x y 21
Adding the two equations, 2  56  8  x  3 / 4 .
x 21 3

3, 3/29 3/4 , 7/6 4/3, 6/7 8/21, 5/21 1/21, 1/21

702
The sum of three integers is 40. The largest integer is 3 times the middle
integer, and the smallest integer is 23 less than the largest integer. What is the
product of the three integers?

1,104 972 672 294 192

(algebra, medium)

Suppose the middle integer is x. The largest is 3x and the smallest is 3x -23.
Thus 7x – 23 = 40 and x = 9, so the integers are 9,27 and 4.
The product is 972

1,104 972 672 294 192


703
The surface of a certain planet reflects 80 percent of the light that strikes it.
The clouds around the planet then absorb 40 percent of the reflected light.
What percent of the light that strikes the planet is reflected from the surface
and passes through the clouds without being absorbed?

32% 40% 48% 60% 88%

(percents, medium)

Suppose that x units of light strike the planet. 0.8x is reflected, of which 60% is
not absorbed by the clouds around the planet. Thus 0.6(0.8x) = 0.48x is the
amount of light that strikes the planet is reflected from the surface and passes
through the clouds without being absorbed.

32% 40% 48% 60% 88%

704
The total charge to rent a car for one day from Company A consists of a fixed
charge of $15.00 plus a charge of $0.20 per mile driven. The total charge to
rent a car for one day from Company B consists of a fixed charge of $20.00
plus a charge of $0.10 per mile driven. Is the total charge to rent a car from
Company A for one day and drive it x miles less than $25.00 ?

(1) The total charge to rent a car from Company B for one day and drive it x
miles is less than $25.00
(2) x < 50

(inequalities, medium)

Note that for small values of x, B costs more than A. Only when the the extra
$0.10 a mile charged by A exceeds the $5.00 difference (i.e. when x> 50) in
fixed charges is A more expensive than B. If x=50, each company charges $25.

(1) SUFF
(2) SUFF

705
The total cost of an office dinner was shared equally by k of the n employees
who attended the dinner. What was the total cost of the dinner?

(1) Each of the k employees who shared the cost of the dinner paid $19.
(2) If the total cost of the dinner had been shared equally by k + 1 of the n
employees who attended the dinner, each of the k + 1 employees would
have paid $18.

(algebra, medium)

(1) The total cost of the dinner, 19k, cannot be determined without
knowledge of the value of k. NOT SUFF
(2) The total cost of the dinner, 18(k+1), cannot be determined without
knowledge of the value of k. NOT SUFF
(T) 19k = 18(k+1), so the value of k can be found, as can the total cost of
the dinner.

706
The total cost for Company X to produce a batch of tools is $10,000 plus $3 per
tool. Each tool sells for $8. The gross profit earned from producing and
selling these tools is the total income from sales minus the total production
cost. If a batch of 20,000 tools is produced and sold, then Company X’s gross
profit per tool is

$3.00 $3.75 $4.50 $5.00 $5.50

(algebra, medium)

The gross profit is 5x – 10,000, where x is the number of tools produced and
sold. If x= 20,000, the gross profit is 90,000, yielding a profit per tool of
90,000/20,000 = 4.5

$3.00 $3.75 $4.50 $5.00 $5.50

707
Score Number
Interval of Scores

50-59 2
60-69 10
70-79 16
80-89 27
90-99 18

The table above shows the distribution of scores for a group of management
trainees. Which interval contains the median of the 73 scores?

60-69 70-79 80-89


90-99 It cannot be determined from the information given

(statistics, medium)

The median of the 73 scores is the 37th highest score , which is in the 80-89
interval.

60-69 70-79 80-89


90-99 It cannot be determined from the information given

708
Favorable Unfavorable Not Sure
Candidate M 40 20 40
Candidate N 30 35 35

The table above shows the results of a survey of 100 voters . Each responded
―favorable‖ or ―unfavorable‖ or ―not sure‖ when asked about his or her
impressions of candidate M and of candidate N. What was the number of voters
who responded ―favorable‖ for both candidates?

(1) The number of voters who did not respond ―favorable‖ for either
candidate was 40.
(2) The number of voters who responded ―unfavorable‖ for both candidates
was 10.

(sets, hard)

We can say that 100= 70 – x + y, where x is the number of responded


―favorable‖ to both and y is the number who so responded to neither.

(1) y= 40. The value of x can be found. SUFF


(2) y ≥10. Nothing is said about the number who are unsure. NOT SUFF

709
The table shows the number of calls received by each of five operators during
each of 4 one-hour periods. For which operator was the standard deviation of
the numbers of calls received during these 4 periods the least?

A B C D E

(statistics, hard)

Operators B and D both have the samllest range, but two of Operator B’s call
data are exactly equal to the mean, 5.

A B C D E

710
Stock Number of Shares

v 68
w 112
x 56
y 94
z 45

The table shows the number of shares of each of the 5 stocks owned by Mr.
Sami. If Mr Sami were to sell 20 shares of Stock x and buy 24 shares of stock y,
what would be the increase in range of the number of shares of the 5 stocks
owned by Mr Sami?

4 6 9 15 20

(statistics, medium)

Currently, the least and greatest of the above numbers are 45 and 112.
The number of shares of x and y that Mr. Sami would hold would be 36 (9 less
than 45) and 118 (6 greater than 112).
Thus the range would increase by 9 + 6 = 15

4 6 9 15 20

711
The sum of the square roots of two integers is 9  6 2 . What is the sum of
the squares of these two integers?

40 43 45 48 52

(exponents, hard)

Suppose that these integers are x and y. Thus x y  96 2

Thus x  y  2 xy  9  6 2  9  2 18.
x + y = 9 and xy = 18, so x and y are 3 and 6 or vice versa.
x2+y2= 9 + 36 = 45

40 43 45 48 52

712
The table shows the quantities of eggs, flour, and sugar used by a certain baker
to make one cake of each of the three different types. If the baker had 24
eggs, 30 cups of flour, and 10 cups of sugar available and had to make 2 angel
food cakes and 1 devil’s food cake, what is the maximum number of pound
cakes that the baker could make with the remaining ingredients?

3 4 5 6 8

(inequalties, medium)

The mandatory cakes require a total of 15 eggs, 10 cups of flour, and 4 cups of
sugar, so 9 eggs, 20 cups of flour and 6 cups of sugar remain. There is enough
flour for 5 pound cakes and enough sugar for six pound cakes, but only enough
eggs for 4.5 pound cakes. Thus at most 4 pound cakes can be made.

3 4 5 6 8

713
Ann $450,000
Bob $360,000
Cal $190,000
Dot $210,000
Ed $680,000

The table above shows the total sales recorded in July for the 5 salespeople at
Acme Truck Sales. It was discovered that one of Cal’s sales was incorrectly
recorded as one of Ann’s sales. After this error was corrected, Ann’s total sales
were still higher than Cal’s total sales, and the median of the 5 sales totals was
$330,000. What was the value of the incorrectly recorded sale?

$100,000 $110,000 $120,000 $140,000 $150,000

(statistics, hard)

In thousands of dollars, we know that Ed’s sales (E) were 680, Bob’s (B) were
360 and Dot’s (D) were 210. If the value of the incorrectly recorded sale is x
thousands of dollars, Ann’s sales (A) were 450 – x and Cal’s (C) were 190 + x,
where A > C. The median of the sales figures, 360, corresponds to the person
who recorded the third largest sales. We therefore know that E > B > A > C >
D. Thus A is the median. 450 – x = 330 and x = 120.

$100,000 $110,000 $120,000 $140,000 $150,000

714
The temperature of a certain cup of coffee 10 minutes after it was poured was
120 degrees Fahrenheit. If the temperature F of the coffee t minutes after it
at
was poured can be determined by the formula F  120  (2 )  60 , where
F is in degrees Fahrenheit and a is a constant, then the temperature of the
coffee 30 minutes after it was poured was how many degrees Fahrenheit ?

65 75 80 85 90

(exponents, hard)

As the temperature of a certain cup of coffee 10 minutes after it was poured


was 120 degrees Fahrenheit, 120 = 120(2-10a) + 60. Thus 2-10a = 1/2 = 2-1.
We see that a = 1/10. so the the temperature of the coffee 30 minutes after it
was poured is 120(2-3) + 60 = 120  60  75
8
65 75 80 85 90

715
The temperatures in degrees Celsius recorded at 6 o’clock in the morning in
various parts of a certain country were 10 , 5 , -2 , -1 , -5 and 15.
What is the median of these temperatures?

-2 -1 2 3 5

(statistics, easy)

Because there are 5 temperatures, find the average of the 3rd and 4th largest:
(i.,e. 5 and – 1) Therefore, the median is 2 .

-2 -1 2 3 5

716
The total price of a basic computer and printer was $2500. If the same printer
had been purchased with an enhanced computer whose price was $500 more
than the price of the basic computer, then the price of printer would have been
1/5 of the that total. What was the price of the basic computer?

1500 1600 1750 1900 2000

(algebra, medium)

Suppose the price of the basic computer is c. The price of the printer
is 2500 – c. 2500 – c is 1/5 of 3000, the total price of the printer and the
enhanced computer.. Thus 2500 – c = 600 and c= 1900.

1500 1600 1750 1900 2000

717
The value of (108-102)/(107-103) is closest to which of the following?

1 10 102 103 104


(exponents, hard)

Note that the numerator is practically 108 and the denominator is practically
107, so the quotient will be every close to 10.

1 10 102 103 104

718
The values of x and y vary with the value of z so that each additive increase of
2 in the value of z corresponds to the value of x increasing by a factor of 2 and
to the value of y increasing by a factor of 3, If x and y are positive for each z >
0, what is the value of x/(x+y) when z =12 ?

(1) When z=6, x=5y


(2) When z=0, x= y + 1

(ratios, hard)

To find the value of x/(x+y), it is sufficient to know the value of x/y. We know
that every increase of 2 in the value of z results in x/y being multiplied by 2/3.
If we are given the ratio of x/y for any even value of z, we can find the value of
x/y that corresponds to z=12.

(1) x/y = 5 when z=6. SUFF


(2) We do not know the ratio of x to y when z=0. NOT SUFF

719
The value of –3 + 9  k is positive for which of the following values of k ?

I. -1
II. 0
III. 1

None I only II only III only I and III

(exponents, medium)

For –3 + 9  k > 0. 9  k > 3 and 9 – k > 9, k < 0

None I only II only III only I and III

720
7 8
The value of (9×10 )(9×10 ) is closest to which of the following?
16 17 56 57 58
10 10 10 10 10
(exponents, medium)

Note that this is equal to 81 × 1015, which is slightly less than 102 × 1015 =
1017.

16 17 56 57 58
10 10 10 10 10

721
This year Henry will save a certain fraction of his income and will spend the
rest. Next year Henry will have no income, but for each dollar that he saves this
year, he will have 1+r dollars available to spend. In terms of r, what fraction of
his income should Henry save this year so that next year the amount he is
available to spend will be equal to half the amount that he spends this year?

1/(r+2) 1/(2r+2) 1/(3r+2) 1/(r+3) 1/(2r+3)

(ratios, hard)

Suppose that he saves fraction f of this year’s income and therefore spends
fraction 1 – f , where 0 < f < 1. Next year, he will be able to spend f(1+ r) of
this year’s income, so f(1 + r) = (1 – f)/2  2f + 2fr = 1 – f  f(3 + 2r) =1
Thus f = 1/(2r + 3)

1/(r+2) 1/(2r+2) 1/(3r+2) 1/(r+3) 1/(2r+3)

722
Three boxes of supplies have an average (arithmetic mean) weight of 7
kilograms and a median weight of 9 kilograms. What is the maximum possible
weight, in kilograms, of the lightest box?

1 2 3 4 5

(inequalities, hard)

As the three boxes of supplies have an average (arithmetic mean) weight of 7


kilograms, the sum of their weights is 21 kilograms. One of the boxes weighs 9
kilograms, so if the lightest and heaviest boxes weigh x and y kilograms, x + 9
+ y = 21. Thus x = 12 – y. Since y is the weight of the heaviest box, y ≥ 9.
Thus x ≤ 3. If y = 9, x = 3 .

1 2 3 4 5

723
3 grades of milk are 1%, 2% and 3% fat by volume. If x gallons of 1% grade, y
gallons of 2% grade, and z gallons of 3% grade are mixed to give x+y+z
gallons of 1.5% grade, what is x in terms of y and z?

y+3z (y+z)/4 2y+3z 3y+z 3y+4.5z

(algebra, percents, hard)

The amount of fat is 1.5% of the total volumen, so


0.01x + 0.02y + 0.03z = .015 (x + y + z)
Multiplying both sides by 1000,
10x + 20y + 30z = 15x +15y+ 15z
5x= 5y + 15z
x= y + 3z

y+3z (y+z)/4 2y+3z 3y+z 3y+4.5z

724
Three printing presses R, S and T, working together at their respective constant
rates, can do a certain printing job in 4 hours. S and T, working together at
their respective constant rates, can do the same job in 5 hours. How many
hours would it take R, working alone at its constant rate, to do the same job?

8 10 12 15 20

(combined work, medium)

In one hour, the three machines can do ¼ of the printing job, and S and T can
do 1/5 of the job. Thus R can do ¼ - 1/5 = 1/20. This means that R would take
20 hours to so the job working alone.

8 10 12 15 20

725
Tickets to a concert were priced at $3.00 for each student and $4.50 for each
non-student. If the total receipts were $9,000, and an equal number of student
and non-student tickets were sold, how many tickets were sold in total?

900 1,200 1,800 2,000 2,400

(algebra, medium)

If x of each type of ticket were sold, 3x + 4.5x = 9000


Thus 7.5x = 9000 and x = 9000/7.5 =3000/2.5= 3000(4)/10= 1200
The total number of tickets is 2x = 2400.

Alternatively, if the same number of each type of ticket were sold, their average
price is $3.75 = $30/8. The number of tickets, then, is 9000/(30/8)=8(300)=
2400.

900 1,200 1,800 2,000 2,400

726
To celebrate a colleague's retirement, the t coworkers in an office agreed to
share equally the cost of a catered lunch. If the lunch costs a total of x dollars
and s of the coworkers fail to pay their share, which of the following represents
the additional amount in dollars, that each of the remaining co workers would
have to contribute, so the cost of the lunch paid?

x x sx sx x(t  s )
t ts ts t (t  s) t

(algebra, hard)

Each colleague was supposed to pay x , and s failed to pay, so there is a


t
sx
shortfall of , which will be paid by in equal parts by the t – s colleagues who
t
did pay. Each of these colleagues will then pay an extra sx .
t (t  s)
Alternatively, find the difference between x , the amount each of the paying
ts

colleagues contributed and x , the amount each of the t colleagues would have
t
contributed if they had all actually done so.
sx
x(t  s )
t (t  s )
x x sx
t ts ts t

727
To determine her state income tax last year, Elena computed 5% of her gross
income, rounded the resulting figure down to the nearest $100, and subtracted
$50 for each dependent she claimed. If Elena’s gross income was $34,500 and
she claimed 2 dependents, how much did she determine she state income tax
to be?

$1,600 $1,625 $1,650 $1,675 $1,700

(percents, easy)

5% of $34,500 is 50% of $3,450 i.e. $1,725, which, when rounded down to the
nearest $100, is $1,700. $100 was then subtracted, $50 for each of Elena’s two
dependents, so she determined her state income tax to be $1,600.

$1,600 $1,625 $1,650 $1,675 $1,700

728
To furnish a room in a model home, an interior decorator is to select 2 chairs
and 2 tables from a collection of chairs and tables in a warehouse that are all
different from each other. If there are 5 chairs in the warehouse and if 150
different combinations are possible, how many tables are in the warehouse?

6 8 10 15 30

(combinatronics, hard)

There are C2  10 ways of choosing the chairs. Therefore, there must be


5

150/10= 15 = C2 ways of choosing the tables. Thus there are 6 tables to choose
6

from.

6 8 10 15 30

729
To install cable television in a home, a certain company charges a basic fee of
$30 plus a fee of $20 for each cable outlet installed in the home. How much did
the cable company charge the Horace family for installing cable television in
their home?

(1) The cable company installed three cable outlets in the Horace family
home.
(2) The amount that the cable company charged the Horace family for
installing cable television in their home was equivalent to an
average (arithmetic mean) charge of $30 per cable outlet installed.

(algebra, medium)

If x outlets were installed, the total charge is 30 + 20x. To answer the


question, it is sufficient to determine the value of x.

(1) x=3 SUFF


(2) (30 + 20x)/x = 30 SUFF

730
To receive a driver’s license, sixteen year-olds at Culliver High School have to
pass both a written and a practical driving test. Everyone has to take the tests,
and no one failed both tests. If 30% of the 16 year-olds who passed the written
test did not pass the practical, how many sixteen year-olds at Culliver High
School received their driver’s license?

(1) There are 188 sixteen year-olds at Culliver High School.


(2) 20% of the sixteen year-olds who passed the practical test failed the
written test.

(sets, medium)

Neither (1) nor (2) is sufficient by itself. Combining (1) and (2) and supposing
that there are n students. If 100x% passed the written test and 100y% passed
the practical test, 0.7xn=0.8yn. In other words, the students who passed both
tests constitute 70% of those who passed the written test and 80% of those
who passed the practical test. As nobody failed both tests, n = xn + 7xn/8 –
7xn/10 so n = xn ( 47/40).
Thus x =40/47, and the number that got their license was 0.7(40/47)(188).
(T) SUFF

731
Two members of a club are to be selected to represent the club at a national
meeting. If there are 190 different possible selections of the 2 members, how
many members does the club have?

20 27 40 57 95

(combinatronics, hard)

If there are n members in the club, there are C2n  n(n  1) different possible
2
selections of the 2 members. Thus n(n – 1 ) = 380. n= 20.

20 27 40 57 95

732
2  2  22  23  24  25  26  27  28 
29 210 216 235 237

(exponents, hard)

Note that 2 + 2 = 4 = 22 and for all integers n, 2n + 2n = 2n+1 . Thus the above
sum is equal to 29 .
Alternatively, it is good to know that 1 + 2 + 22 + 23 + … + 2n-1 = 2n – 1 for all
positive integers n.

29 210 216 235 237

733
What fraction of this year's graduating students at a certain college are male?

(1) Of this year's graduating students, 33% of the males and 20% of the
females transferred from another college.
(2) Of this year's graduating students, 25% transferred from another
college
(ratios, hard)

(1) No information is given about fraction who transferred from another


college. If 33 males and 20 females transferred, there are 100 males and
100 females in all. However if 66 males and 1 female transferred, there
are 200 males and 5 females in all. NOT SUFF
(2) Gender is not mentioned. NOT SUFF
(T) If there are m males and f females, 0.33m + 0.2f =0.25(m + f), the
number of graduating students who transferred. Thus 0.08m=0.05f, and
the ratio of male students to females students, and thus the fraction of
the students that are male can be found: m: f = 5:8 and m: m+f =5:13
SUFF

734
What is the average height of the n people in a certain group?

(1) The average height of the n/3 tallest people in the group is 6 feet 2 1/2
inches, and the average height of the rest of the people in group is 5
feet 10 inches.
(2) The sum of the heights of the n people is 178 feet and 9 inches.

(algebra, ratios, hard)

(1) As we know the average height of the two groups and the ratio of the
number of people in each group, the average height of the n people can
be calculated: As 1 foot is equal to 12 inches, the sum of the heights is
n/3(74.5)+(2n/3)(70), so the average height is 1/3(74.5)+2/3(70).
SUFF

(2) Without information about n or information about the heights, we cannot


determine the average height. NOT SUFF

735
What is the average (arithmetic mean) of 11 integers?

(1) The average of the first 9 integers is 7.


(2) The average of the last 9 integers is 9.

(algebra, medium)

(T) Neither is sufficient on its own. The first implies that the sum of the first
9 integers is 63, while (2) implies that the sum of the last 9 integers is
81. Thus the sum of the last two integers is 18 higher than the sum of
the first two integers. Suppose the sum of the first two integers is x.
Then the sum of the middle 7 integers is 63 – x and the sum of the last
two integers is x + 18. Therefore the sum of the 11 integers is x + (63 –
x) + (x + 18) = x + 81. As the sum of the 11 integers cannot be found,
nor can their average be found. NOT SUFF

736
What is the greatest common divisor of positive integers m and n?

(1) m is a prime number.


(2) 2n=7m

(factors and multiples, hard)

(1) As m is a prime number, it has only two divisors, 1 and m. If n is a


multiple of m, the greatest common divisor is m. However, if n is not a
multiple of m, the greatest common divisor is 1. NOT SUFF
(2) n:m = 7:2, so n=7x and m=2x for some positive integer x. Since 7 and 2
are prime numbers, the greatest common divisor is x, the value of which
cannot be determined. NOT SUFF
(T) As m is a prime number, x=1 SUFF

737
What is the greatest possible area of a triangular region with one vertex at the
center of a circle of radius 1 and the other two vertices on the circle?

3/4 1/ 2  /4 1 2

(geometry, hard)

Consider a triangle OAB with vertices O(0,0), A(1,0), and B(x,y). Think of OA =
1 as the base of the triangle. To maximize the area of triangle OAB, the height,
|y| should be as great as possible. Thus OAB should be a right triangle with
the right angle at O. Its height and base will both be 1, so its area with be ½.

3/4 1/ 2  /4 1 2

738
What is the greatest prime factor of 417 - 228?

3 5 7 9 11

(factors and multiples, hard)

Writing this expression as a product of prime numbers:


417 - 228=234 - 228= 228(26 -1)= 228 (23 – 1)(23 + 1)= 228x7x32. The smallest
prime factor is 7.

3 5 7 9 11

739
What is the greatest prime factor of 2100 - 296?

2 3 5 7 11

(factors and multiples, hard)

Writing this expression as a product of prime numbers:

296 (24 – 1) =296(5)(3). Thus the three prime factors of this expression are 2, 3
and 5.

2 3 5 7 11

740
What is the hundredths digit of the decimal z?

(1) The tenths digit of 100z is 2


(2) The units digit of 1000z is 2

(exponents, medium)

Consider z=0.abc , where a, b and c are respectively the tenths, hundredths


and thousandths digit of z. We want to find the value of b.

(1) 100z = ab.c, so c=2. NOT SUFF


(2) 1000z= abc , so c=2. NOT SUFF
(T) (1) and (2) provide the same insufficient information. NOT SUFF

741
What is the hundreds digit of the integer z?

(1) 10z = 93,120


(2) z rounded to the nearest hundred is 9,300.

(inequalities, medium)

(1) Allows us to determine the exact value of z. SUFF


(2) Tells us that z is between 9, 249 and 9,350 NOT SUFF

742
What is the median number of employees assigned per project for the projects
at Company Z?

(1) 25 percent of the projects at Company Z have 4 or more employees


assigned to each project.
(2) 35 percent of the projects at Company Z have 2 or fewer employees
assigned to each project.

(statistics, medium)

(1) As 75% of the projects have less than 4 employees assigned to them,
the median number of employees assigned per project is less than 4.
NOT SUFF
(2) As 65% of the projects have more than than 2 employees assigned to
them, the median number of employees assigned per project is greater
than 2. NOT SUFF
(T) The median number is 3. SUFF

743
What is the product of all the solutions of x2 + 4x + 7 =|x+2| + 3 ?

-6 -2 2 6 12

(inequalities, hard)

Note that for w>0, |w|/w =w/|w| = 1, whereas for w < 0, |w|/w = - 1,
as |w| = - w.

We see that x2 + 4x + 7 = (x + 2)2 =|x + 2|. If x= - 2, we get 0=0

If x + 2 > 0, dividing both sides by x + 2, we get x + 2 = 1, i.e. x = - 1


On the other hand, if x + 2 < 0, we get x + 2 = - 1. i.e. x= - 3

Thus x can be either – 3, -2 , or -1, and the product of all of the solutions is – 6

-6 -2 2 6 12

744
What is the ratio of the average (arithmetic mean) height of students in class X
to the average height of students in class Y?

(1) The average height of the students in class X is 120 centimeters.


(2) The average height of the students in class X and class Y combined is
126 centimeters.

(ratios, medium)

(T) Without information about the ratio of the class sizes, the question
cannot be answered: if the classes were the same size, the average
height of the students in Y would be 132 centimeters, but Y had twice as
many students as X, the average height of the students in Y would be
129 centimeters. NOT SUFF

745
What is the remainder when positive integer n is divided by positive integer k,
where k > 1?

(1) n = (k+1)3
(2) k=5

(factors and multiples. hard)

(1) Note that n= k3 + 3k2 + 3k + 1, in other words, n is one greater than a


sum of multiples of k. Thus n is one greater than a multiple of k and thus
the remainder when n is divided by k is 1. SUFF
(2) The remainder can be 0, 1 , 2 , 3 or 4. NOT SUFF

746
What is the remainder when the positive integer n is divided by 2?

(1) When n is divided by 5, the remainder is an odd integer.


(2) When n is divided by 10, the remainder is an odd integer.

(factors and multiples, medium)

The remainder is 1 if n is odd and 0 if n is even.

(1) n is either 1 or 3 greater than a multiple of 5. If n is either 1 or 3 greater


than an odd multiple of 5, n is even. If n is either 1 or 3 greater than an
even multiple of 5, n is odd. NOT SUFF
(2) n = 10k + r, where k is an integer and r {1,3,5,7,9}. Since 10k is even
and r is odd, n is odd. SUFF

747
What is the remainder when the positive integer x is divided by 8?

(1) When x is divided by 12, the remainder is 5.


(2) When x is divided by 18, the remainder is 11.

(factors and multiples. hard)

As neither 12 nor 18 is a multiple of 8, neither (1) nor (2) is sufficient by itself.

The fastest way to combine (1) and (2) is to find two possible values of x, 29
and 65. They have different remainders when divided by 8.

We can say that x=12k + 5 = 18m + 11 for some integers m and k.


Thus 12k = 18m + 6, so 2k = 3m + 1. k = (3m+1)/2
For k to be an integer, m has to be odd (m=2n + 1 for some integer n).
Thus x= 29 + 36n for some integer n. However, as 36 is not a multiple of 8,
the remainder in question will vary. NOT SUFF

Alternatively, (1) reveals that when x is divided by 36, the remainder is either 5,
17 or 29. (2) implies that when x is divided by 36, the remainder is either 11 or
29. So the remainder when x is divided by 36, the remainder is always 29.
Therefore, when x is divided by 72, the remainder is 29 or 65. It follows that x
is either 29 or 65 greater than a multiple of 72=9(8). The remainder when x is
divided by 8 is either 5 or 1.

(T) NOT SUFF

748
What is the remainder when the positive integer x is divided by 3?

(1) When x is divided by 6, the remainder is 2.


(2) When x is divided by 15, the remainder is 2.

(factors and multiples. hard)

(1) Since 6 is a multiple of 3, x is 2 greater than a multiple of 3. Thus the


remainder when x is divided by 3 is 2. SUFF
(2) Since 15 is a multiple of 3, x is 2 greater than a multiple of 3. Thus the
remainder when x is divided by 3 is 2. SUFF

749
What is the remainder when the two-digit, positive integer x is divided by 3 ?

(1) The sum of the digits of x is 5.


(2) The remainder when x is divided by 9 is 5.

(factors and multiples, medium)

(1) If the sum of the two digits of x is 5, x must be either 50,41,32,23, or


14. In any case, the remainder will be 1. SUFF
(2) x =9k + 5, where k is a positive integer from 1 to 10.
Thus x = 9k + 3 + 2 =3(3k + 1) + 2.
In other words, x is 2 greater than a multiple of 3, so the remainder is 2.
SUFF

750
What is the result when X is rounded to the nearest hundredth?

(1) When X is rounded to the nearest thousandth the result is 0.455


(2) The thousandth digit of X is 5

(exponents, medium)

(1) 0.4545 < X < 0.4555. The result can be 0.45 or 0.46 NOT SUFF
(2) Obviously not sufficient on its own.
(T) 0.455 ≤ X < 0.4555 The result is 0.46 SUFF

751
What is the standard deviation of the weights of the 30 samples in a certain
experiment?

(1) The total weight of the 30 samples is 360 grams.


(2) Each of the 30 samples weighs 12 grams.

(statistics, medium)

(1) Allows to find the mean weight, but not the standard deviation.
NOT SUFF
(2) All of the weights are equal, so the standard deviation is 0. SUFF

752
What is the tens digit of the positive integer r ?

(1) The tens digit of r/10 is 3.


(2) The hundreds digit of 10r is 6.

(exponents, medium)

(1) The tens digit of r/10 is the the same as the hundreds digit of r.
NOT SUFF
(2) The hundreds digit of 10r is the the same as the tens digit of r.
SUFF

753
What is the total number of executives at Company P?

(1) The number of male executives is 3/5 the number of female executives.
(2) There are 4 more female executives than male executives.

(algebra, medium)

Suppose that there are m males and f females at Company P. We are asked for
the value of m + f .

(1) m =3f/5, so m+f = 8f/5. However, we not given a way to find f.


NOT SUFF
(2) f = m + 4 m = f – 4, so m + f = 2f – 4 NOT SUFF
(T) 8f/5 = 2f – 4. As we can find the value of f, we can find m + f SUFF

754
What is the value of a4 – b4 ?
(1) a2 – b2 = 16
(2) a+b=8

(algebra, medium)

Note that a4 – b4 = (a2 – b2)( a2 + b2)= (a + b)(a – b )( a2 + b2 )

(1) Although we are told the value of a2 – b2, no information is given about
the value of a2 + b2. For example if a=4 and b=0, a4 – b4= 44. On the other
hand, if a= 17 and b= 1 , a2 + b2= 18 and thus a4 – b4= 18 x 16 > 44.
NOT SUFF
(2) We are not told about the value of a – b or a2 – b2. For example, if a= 4
and b= 4, a – b =0 and so a4 – b4 = 0. On the other hand, if a= 8 and b= 0,
a4 – b4= 84. NOT SUFF

(T) Since a2 – b2 = (a + b)(a – b ), a – b = 2. (2) tells us that a + b = 8.


We have two linear equations with two variables, a and b, so we can
solve for each and thus find the value of a4 – b4. SUFF

755
k, n, 12, 6, 17

What is the value of n in the list above?

(1) k<n
(2) The median of the numbers in the list is 10.

(statistics, medium)

(1) It is clear that (1) is not sufficient.


(2) tells us that the median (middle number in value) is 10, so either k or n
is 10. It could be that k=n=10 or k < n = 10 or 10 = k < n NOT SUFF
(T) n= 10 SUFF

756
n , 8 , 3 , 15 , 17

What is the value of n in the list above?

(1) The median of the numbers in the list is 9.


(2) The difference between the greatest number in the list and the least
number is 14.

(statistics, medium)

(1) The median of a set of 5 numbers is the 3rd greatest number. Since
none of the other numbers is equal to 9, n must be 9. SUFF
(2) Excluding n, the range of the four numbers above is 14, so n can be any
number from 3 to 17. NOT SUFF

757
What is the value of 5x2 + 4x – 1?

(1) x(x + 2)= 0


(2) x= 0

(algebra, medium)

(1) If x=0, the polynomial equals -1 , whereas if x = - 2, the polynomial


equals 20 – 8 – 1 ≠ -2. NOT SUFF
(2) Obviously sufficient. SUFF

758
4
 4300  2600 
What is the value of p if 2 = 
p  ?
 2 

1200 2100 2400 4796 4799

(exponents, medium)

Since 4300 = (22)300 = 2600


2p = (2(2600)/2)4 = (2600)4=22400, so p= 2400

1200 2100 2400 4796 4799

759
What is the positive integer n?

(1) For every integer m, (m+n)!/(m – 1)! is divisible by 16


(2) n2- 9n + 20 = 0

(factors and multiples. hard)

(1) Note that (m+n)!/(m – 1)! = m(m+1)(m+2)...x (m+n) , the product of


the n + 1 integers from m to m+n, inclusive. The product is a multiple of 16 for
every integer n if and only if three of the terms are even, i.e. if the product
consists of at least 6 terms. Thus n + 1 ≥ 6 and n is at least 5. NOT SUFF

(2) n is either 4 or 5 NOT SUFF


(T) n=5 SUFF

760
What is the value of positive integer m?

(1) When m is divided by 6, the remainder is 3.


(2) When 15 is divided by m, the remainder is 6.

(factors and multiples, medium)

(1) m = 6k + 3 for non-negative integer k NOT SUFF


(2) 15 = xm + 6 for some integer x and m > 6
m = 9/x i.e. m is a divisor of 9
Since the only divisor of 9 that is greater than 6 is 9, m=9 SUFF

761
What is the total surface area of rectangular solid R ?

(1) The surface area of one of the faces of R is 48.


(2) The length of one of the edges of R is 3.

(geometry, medium)

(T) The total surface area of R is 2 ( xy + xz + yz ). Combining (1) and (2),


it could be that xy =48 and x=3. In that case, z could be any positive
real number, as could the total surface area of R. NOT SUFF

762
What was the percent increase in the population of City K from 1980 to 1990?

(1) In 1970 the population of City K was 160,000


(2) In 1980 the population of City K was 20 percent greater than it was in
1970, and in 1990 the population of City K was 30 percent greater that in
was in 1970.

(percents, medium)

(1) Obviously not sufficient NOT SUFF


(2) Suppose the population of City K was x in 1970. Then in 1980 it was 1.2x
and 1.3x in 1990. Since the ratio of the population in 1990 to the
population in 1980 is 1.3x/1.2z = 13/12, a constant, we can calculate the
percent increase from 1980 to 1990. SUFF

763
5
What is the value of x ?
2
(1) x =9
3
(2) x >9

(exponents, medium)

5
(1) |x|= 3. As x can be either positive or negative 3, x can be either -35 or
35. NOT SUFF
(2) x > the cube root of 9. As x cannot be determined , neither can x5. NOT
SUFF
(T) (2) tells us that x is positive, and so (1) tells us that x must be 35 SUFF

764
What is the value of v3 – k3?
(1) vk > 0
(2) v–k=6

(algebra, medium)

(T) v = 6 + k, so we are asked for the value of (6 + k)3 – k3 , whose value


depends on the value of k. For example, if k=1, we get 73 – 1 but if k=4
, we get 103 - 43 NOT SUFF

765
What is 35 percent of the sum of 1.4 and 1/5 ?

0.42 0.56 0.85 1.55 1.95

(percents, medium)

Since 1/5 = 0.2, we are asked for 35 % of 1.6 = 70% of 0.8 = 0.56

0.42 0.56 0.85 1.55 1.95

766
What was the range of the selling prices of the 30 wallets sold by a certain
store yesterday?

(1) 1/3 of the wallets had a selling price of $24 each.


(2) The lowest selling price of the wallets was 1/3 the highest selling price of
the wallets.

(statistics, medium)

(T) The lowst price could be $24 and the highest $72, yielding a range of 48.
Alternatively the lowest could be x and the highest 3x, where
x < 24 < 3x. NOT SUFF

767
What was the price at which a merchant sold a certain appliance?

(1) The merchant’s gross profit on the appliance was 20 percent of the price
at which the merchant sold the appliance.
(2) The price at which the merchant sold the appliance was $50 more than
the merchant’s cost of the appliance.

(algebra, medium)

(1) If s is the selling price and p is purchase price, s – p = 0.2s. Thus 0.8s =
p. The value of p is not known, so we cannot find the value of s.
NOT SUFF
(2) s = p + 50 NOT SUFF
(T) We have two linear equations involving two variables. SUFF

768
What is the value of x ?

(1) x2 + y2 = 25
(2) xy = 12

(algebra, coordinate geometry, medium)

(T) x2 + 144/x2 = 25, so x4 – 25x2 + 144 = 0


(x2 – 9)(x2 – 16) = 0
Thus x2 is either 9 or 16, and x can be -4 ,-3 , 3 , 4

Also, note that (1) describes a circle with center in the xy- plane with
center (0,0) and radius 5 and four possible 3-4-5 right triangles can
be drawn. NOT SUFF

769
When a certain tree was first planted, it was 4 feet tall, and the height of the
tree increased by a constant amount each year for the next 6 years. At the end
of the 6th year, the tree was 1/5 taller than it was at the end of the 4th year.
By how many feet did the height of the tree increase each year?

3/10 2/5 1/2 2/3 6/5

(algebra, hard)

Suppose the height increased x feet a year. At the end of the 6th year, it is (4 +
6x) feet tall and at the end of the 4th year , it is (4 + 4x) feet tall.

The difference between these two heights, 2x is 1/5 of 4 +4x, so x=2/3.

3/10 2/5 1/2 2/3 6/5

770
When positive integer n is divided by 5, the remainder is 1. When n is divided
by 7, the remainder is 3. What is the smallest positive integer k such that n + k
is a multiple of 35 ?

3 4 12 32 35

(factors and multiples, medium)

Note that 35 is a multiple of 7 and of 5, so if n is 1 greater than a multiple of 5,


its remainder when divided by 35 must be one of the following numbers:
1,6,11,16,21,26 and 31. Likewise, as n is 3 greater than a multiple of 7, its
remainder when divided by 35 must be one of the following numbers:
3,10,17,24,31. Thus n must be 31 greater than a multiple of 35 and thus 4 less
than a multiple of 35. k=4

3 4 12 32 35

771
When positive integer n is divided by 3, the remainder is 2 and when positive
integer t is divided by 5, the remainder is 3. What is the remainder when the
product of nt is divided by 15?

(1) n-2 is divisible by 5


(2) t is divisible by 3

(remainders, hard)

Note that n is 2 greater than a multiple of 3 and that t is 3 greater than a


multiple of 5. Thus n – 2 and t – 3 are multiples of 3 and 5 respectively.

(1) n – 2 is a multiple of 5 and thus of 15. We can say that n – 2 = 15x or


n = 15x + 2, where x is some integer. However, we do not know the
remainder when t is divided by 15- it could be 3, 8 or 13.

Suppose t= 15y + r where r is either 3, 8 or 13

nt = (15x + 2)(15y + r) , which is 2r greater than a multiple of 15


Thus nt is either 6, 16 or 26 greater than a multiple of 15.
Therefore the remainder in question is either 1,6 or 11.
NOT SUFF

Alternatively, since n – 2 is a multiple of 15, n could be 2. Since t is 3 greater


than a multiple of 5, t could be 3 or 8. Thus nt could be 6 or 16, and the
remainder in question could be 6 or 1.

(2) Similar to (1), n could be 2 or 5. t – 3 is a multiple of 3 and 5 and thus of


15. Thus t – 3 could be 15 or 30, i.e. t could be 18 or 33. nt could be 36
or 90, so the remainder in question could be 0 or 6- NOT SUFF

(T) n = 15x + 2 and t= 15y + 3, so nt =15(15xy + 2y + 3x) + 6, so the


remainder in question is 6. SUFF

Alternatively, look at consecutive possible values of n and t:


n
2 17

3 6 51
t
18 36 306

772
When positive integer x is divided by positive integer y, the remainder is 9. If
x/y = 96.12, what is the value of y?

96 75 48 25 12

(factors and multiples, hard)

Remember that x/y = q + r/y, where q is the quotient (96) and r is the
remainder when x is divided by y.

Thus 9/y = 0.12, so y=9/0.12 = 900/ 12 = 75

96 75 48 25 12

773
When a positive integer is divided by 4, the remainder is r; when divided by 9,
the remainder is R. What is the greatest possible value of r2+R?

23 21 17 13 11

(factors and multiples. hard)

The r must be an integer from 0 to 3 and R must be an integer from to 8. The


greatest possible value of r2+R= 32 + 8= 17.

23 21 17 13 11

774
When integer m is divided by 13, the quotient is q and the remainder is 2.
When m is divided by 17, the remainder is 2. What is the remainder when q is
divided by 17?

0 2 4 9 13

(factors and multiples, hard)

m=13q +2 but also m=17k + 2, where k is an integer.

So 13q=17k and thus integer k=13q/17. q is therefore a multiple of 17 and


thus the required remainder is 0.

0 2 4 9 13

775
Whenever Martin has a restaurant bill with an amount between $10 and $99,
he calculates the dollar amount of the tip as 2 times the tens digit of the
amount of his bill. If the amount of Martin's most recent restaurant bill was
between $10 and $99, was the tip calculated by Martin greater than 15% of the
amount of the bill?

(1) The amount of the bill was between $15 and $50
(2) The tip calculated by Martin was $8.

(inequalities, hard)

Using algebra, suppose his bill is 10t + u. Thus his tip will be 2t.
The question becomes: 2t > 0.15(10t +u) i.e. 50t > 15u ?
In other words, is u < 10t/3 ?

(1) If t=2, the answer will be yes if and only if u =5 or u=6.


(2) If 2t=8, t=4 , the question becomes u < 40/3. Since u < 10, SUFF

Note that the higher the tens digit of Martin’s bill, the closer to 20% will be the
percentage of his tip. Conversely, the higher the units digit of his bill, the lower
will be the percentage of his tip.

(1) The minimum tip in terms of percentage corresponds to a bill of $19. The
$2 tip is slightly more than 10% of the total bill. The maximum tip in
terms of percentage corresponds to bills of $20, $30, $40 and $50: for
any of these sums, the tip is 20% of the amount of his bill. NOT SUFF
(2) This tells us that the tens digit of Martin’s bill is 4. The lowest tip in terms
of percentage corresponds to a bill of $49.99. Since $8 is 16% of $50,
we know that the tip Martin calculated was greater than 15% of the
amount of the bill. SUFF

One could easily see that (2) but not (1) is sufficient by testing the extreme
values of the bill. For example, for (1), examine bills of 15 and 50. Note,
however, that this easy method is precarious: imagine that (1) said 18 instead
of 15. Would extreme values would you check?

776
When positive integer x is divided by 5, the remainder is 3; when x is divided by
7, the remainder is 4. When positive integer y is divided by 5, the remainder is
3 and when y is divided by 7, the remainder is 4. If x > y, which of the
following must be a factor of x – y ?

12 15 20 28 35

(factors and multiples, hard)

Note that x and y have the same remainder when divided by 5. Thus x – y has
a remainder of 0 when divided by 5, i.e. x – y is a multiple of 5. Also since and
y have the same remainder when divided by 7, x – y is a multiple of 7. Thus
x – y is a multiple of the least common multiple of 5 and 7, 35. This means that
35 is a factor of x – y.

12 15 20 28 35
777
When 20 is divided by the positive integer k, the remainder is k – 2. Which of
the following is a possible value of k ?

8 9 10 11 12

(factors and multiples , medium)

The most straightforward way of finding the answer is by trial and error.
Alternatively, if 20/k yields a quotient of q and a remainder of k – 2, 20 = qk
+ k – 2, so k(q+1) = 22. Therefore, k is a divisor of 22 that is greater than 2,
for example 11.

8 9 10 11 12

778
When 200 gallons of oil were removed from a tank, the volume of oil left in the
tank was 3/7 of the tank’s capacity. What was the tank’s capacity?

(1) Before the 200 gallons were removed, the volume of oil in the tank was
1/2 of the tank’s capacity.
(2) After the 200 gallons were removed, the volume of oil left in the tank
was 1,600 gallons less than the tank’s capacity.

(ratios, medium)

Suppose that the tank’s capacity is c gallons

(1) c/2 – 3c/7 = 200 SUFF


(2) 3c/7 = c – 1600 SUFF

779
Which of the following inequalities has a solution set that, when graphed on the
number line, is a single line segment of finite length?

x4  1 x 3  27 x 2  16 2 x 5 2  3x  4  6

(inequalities, hard)

x 4  1 | x | 1
x3  27  x  3
x 2  16 | x | 4
2  x  5  5  x  2 or 2 x5
2  3x  4  6  2  3x  2  2 / 3  x  2 / 3

The first and third are two line segments of infinite length.
The second is a single line segment of infinite length.
The fourth is two line segments of finite length.
The last is a single line segment of finite length.

x4  1 x 3  27 x 2  16 2  x  5 2  3x  4  6

780
Which of the following is an integer?

I 12!/6!

II 12!/8!

III 12!/7!5!

I only II only III only I and II I, II & III

(combinatronics, medium)

For non-negative integers x!/y! is an integer if x > y, so I and II are integers.


12!/7!5! = 12 x 11 x 10 x 9 x 8/ 5 x 4 x 3 x 2 x 1 = 11 x 9 x 8.
One can also think of III as C5 , which of course is an integer.
12

I only II only III only I and II I, II & III

781
Which of the following is closest in value to 99 – 92 ?

99 98 97 96 95

(exponents, medium)

Note that 99 is 9 times 98 , so 99 - 92 is much closer to 99 than to 98.

Alternatively, 99 – 92 = 9 (98 )– 98(1/96) = (9 - 1/96 ) 98 > 9/2 (98), the


average of 99 and 98.

99 98 97 96 95

782
Which of the following is equal to the value of 25 + 25 + 35 + 35 + 35?

56 135 26+36 27+38 45 + 95

(exponents, hard)

Note that 25 + 25= 2 x 25 = 25+1= 26. Similarly, 35 + 35 + 35= 36

56 135 26+36 27+38 45 + 95

783
Which of the following is equal to x18 for all positive values of x?

x9 + x9 (x2)9 (x9)9 (x3)15 x4/x22

(exponents, medium)

2 9
x9 + x9 = 2x9 (x ) =x18 (x9)9 = x81 (x3)15 = x45
x4/x22 = x-18

784
Which of the following is equal to 517 49?

2 × 1013 2 × 1017 2 × 1020 2 × 1026 2 × 1036

(exponents, medium)

Since 49 = (22)9 = 218 , 517 49 = 517 218 = 2(10)17

2 × 1013 2 × 1017 2 × 1020 2 × 1026 2 × 1036

785
38, 69, 22, 73, 31, 47, 13, 82

Which of the following numbers is greater than three-fourths of the numbers


but less than one-fourth of the numbers in the list above?

61 67 71 75 77

(ratios, medium)

Any number that is greater than 6 of the 8 measurments, i.e greater than 69
but less than 2 of the 8 measurements, i.e. less than 73 will do.

61 67 71 75 77

786
Which of the following statements can be inferred from the data above?

I. The Klein family’s annual income more than doubled from 1985 to 1995.
II. The Klein family’s annual income increased by a greater amount from
1985 to 1990 than from 1990 to 1995.
III. The Klein family’s average (arithmetic mean) annual income for the
period shown was greater than $40,000.

I II I and III only II and III only I, II and III

Since its income in thousands of dollars in 1985, 1990 and 1995 was 25, 38 and
64 respectively, I is clearly true and II is clearly false.

When 40 is subtracted from its annual income each year, the following set is
obtained:
{ -15, -14, -14, -5, -4, - 2 , -2 , 16, 24, 24, 24}, the sum of whose elements is
positive. Thus III is true.

I and III only

787
Working alone at its own constant rate, a machine seals k cartons in 8 hours,
and working alone at its own constant rate, a second machine seals k cartons in
4 hours. If the two machines, each working at its own constant rate and for the
same period of time, together sealed a certain number of cartons, what percent
of the cartons were sealed by the machine working at the faster rate?

25% 33 1/3% 50% 66 2/3% 75%

(combined work, medium)

The second machine is twice as fast as the first. Therefore, for every carton the
first seals, the second seals two. In other words, the faster machine sealed 2/3
of all the cartons sealed.

25% 33 1/3% 50% 66 2/3% 75%

788
x , y, z are three positive non-zero integers such that 3x=4y=7z. What is the
least possible value of x+y+z?

41 61 84 92 104

(ratios, hard)

x:y = 4:3 and y:z = 7:4, so x:z= 28:12. Thus x:y:z = 28:21:12
So x= 28k , y=21k and z=12k for some positive integer k.
Thus x+ y +z = 61k for some positive integer k. The least possible value of this
sum, 61, corresponds to k=1.

41 61 84 92 104

789
Working alone at its constant rate, machine K took 3 hours to produce
1/4 of the units produced last Friday. Then machine M started working and the
two machines, working simultaneously at their respective constant rates, took 6
hours to produce the rest of the units produced last Friday. How many hours
would it have taken machine M, working alone at its constant rate, to produce
all of the units produced last Friday?

8 12 16 24 30

(combined work, medium)

K worked for a total of 9 hours, and thus produced 3/4 of the units produced
last Friday. Thus the remaining 1/4 of the units were produced by M in the 6
hours it worked. Therefore to produce all the units alone at its constant rate, M
would have taken 24 hours.

8 12 16 24 30

790
Working independently at their respective constant rates, pumps X and Y took
48 minutes to fill an empty tank with water. What fraction of the water in the
full tank came from pump X ?

(1) Working alone at its constant rate, pump X would have taken 80 minutes
to fill the tank with water.
(2) Working alone at its constant rate, pump Y would have taken 120
minutes to fill the tank with water.

(combined work, medium)

It would be sufficient to know what fraction of the tank Y filled.

(1) X filled 48/80 of the tank. SUFF


(2) Y filled 48/120 of the tank. SUFF

791
Working independently, Tina can do a certain job in 12 hours. Working
independently, Ann can do the same job in 9 hours. If Tina works
independently at the job for 8 hours and then Ann works independently, how
many hours will it take Ann to complete the remainder of the job?

2 7/3 8/3 11/4 3

(combined work, medium)

If Tina works independently for 8 hours, she does 8/12 = 2/3 of the job, so Ann
has to do the remaining 1/3 of the job, a task that will take her 1/3 of the 9
hours it takes Ann to do the entire job on her own. Therefore, it will take Ann 3
hours to complete the job.

2 7/3 8/3 11/4 3

792
x and y are integers such that x < y < 0. What is the value of x ?

(1) (x + y )(x – y ) = 7
(2) xy = 12

(factors and multiples, hard)

(1) Note that (x + y )(x – y ) = x2 – y2 = 7. When we look at the squares of


consecutive integers 1,4,9,16, 25…, we see that x2 must be 16 and thus
x= -4 SUFF
(2) xy = 12, x could be -12 , -6 or -4 NOT SUFF

793
Xavier, Yvonne, and Zelda each try independently to solve a problem. If their
individual probabilities for success are ¼, ½, and 5/8, respectively, what is the
probability that Xavier and Yvonne, but not Zelda, will solve the problem?

11/8 7/8 9/64 5/64 3/64

(probability, medium)

We are asked to find Pr ( ) = Pr(X) Pr (Y ) ( 1 – Pr (Z)) = ¼ × ½ ×


3/8 = 3/64.

11/8 7/8 9/64 5/64 3/64


794
xn = xn+2 for any integer n. Is it true that x > 0 ?

(1) x = x2 – 2
(2) 2x < x5

(exponents, hard)

If xn = xn+2 , either x=0, or ,dividing both sides by xn , we see that x2 = 1, i.e


|x|=1.

Thus x is either – 1, 0, or 1.

(1) x2 – x – 2 = 0, so (x – 2)(x + 1)=0. Since x cannot be 2, x must be – 1


and the question can be answered. SUFF
(2) Substituting the possible values of x, we see that x can only be – 1
SUFF

Alternatively, assuming x > 0, x4 > 1, i.e. x > 1, which is impossible.


Thus x cannot be greater than 0

795
Yesterday, Diana spent a total of 240 minutes attending a training class,
responding to E-mails, and talking on the phone. If she did no two of these
three activities at the same time, how much time did she spend talking on the
phone?

(1) Yesterday the amount of time that Diana spent attending the training
class was 90 percent of the amount of time that she spent responding to
E-mails.
(2) Yesterday the amount of time that Diana spent attending the training
class was 60 percent of the total amount of time that she spent
responding to E-mails and talking on the phone.

(algebra, medium)

If c, e and p are the amount of time she spent in class, with e-mails and on the
phone, respectively. c + e + p = 240. We are asked for the value of p. Note
that p = 240 – (e + c)

(1) c =0.9e NOT SUFF


(2) c =0.6(e + p) We can find the value of e + p, but we don’t know
the value of e. NOT SUFF
(T) From (2), we can find c and then e and finally p. Also keep in mind that
we have 3 linear equations with three unknowns. SUFF

796
Yves can paint a certain fence in 1/2 the time it takes Miguel to paint the same
fence. If they work together, each at his own constant rate, how many hours
will it take them to paint the fence?

(1) Yves can paint the fence by himself in 3 hours.


(2) Working together, each at his own constant rate, they can paint the
fence in 1/3 the time it would take Miguel, working alone, to paint the
fence.

(combined work, medium)

(1) Yves paints twice as quickly as does Miguel, and (1) tells us that he can
paint 1/3 of the fence in 1 hour, so Miguel can paint 1/6 of the fence in
one hour. Since we can calculate the fraction of the fence they would
paint together in one hour, we can answer the question. SUFF
(2) This is simply a consequence of the fact that Yves paints twice as fast as
Miguel. NOT SUFF

797

You might also like